Download Abnormal Behavior - Keansburg School District

Document related concepts

E. Fuller Torrey wikipedia , lookup

Mechanisms of schizophrenia wikipedia , lookup

Transcript
Unit XII
Abnormal Behavior
PD
Unit Overview
”Normal” is a concept that is taken for granted. What is normal for
someone in Kiev might be considered abnormal for someone from
Kissimmee. “Normal” behavior can be defined in many ways and is
dependent on context, genetics, biology, thinking, and emotions. So,
if the definition of “normal” is so difficult to pin down, how can we
identify people who might be considered “disordered” in their behavior or thinking? Psychologists and psychiatrists use many tools to help
people whose behavior and thinking are different enough to cause
distress and disruption to their lives and the lives of those around
them. A major task of living a “normal” life is the ability to function
in daily activities and to adjust to changes as they arise. Psychologists
and psychiatrists have developed criteria for identifying what behaviors and thinking most likely disrupt daily life, and by identifying those
criteria, mental health professionals can then develop treatments to
help people live the “normal” lives they crave. In this unit, we will explore the criteria for defining psychological disorders and identifying
people who experience them. In the following unit, we will explore
the treatment options for people with psychological illness. After
reading this unit, students will be able to:
• Distinguish between normality and disorder.
• Analyze the controversy over the diagnosis of attention–deficit/
hyperactivity disorder.
•
•
•
•
•
•
Contrast the medical model with the biopsychosocial approach.
Explain how and why clinicians classify psychological disorders.
Analyze the arguments against using diagnostic labels.
Explore the prevalence of psychological disorders.
Analyze the link between poverty and serious mental illness.
• Describe obsessive-compulsive disorder.
• Describe posttraumatic stress disorder.
• Apply the learning and biological perspectives to anxiety
disorders, OCD, and PTSD.
• Identify mood disorders.
• Contrast major depressive disorder and bipolar disorder.
• Apply the biological and social-cognitive perspectives to mood
disorders.
• Analyze the factors involved in suicide and self-injury.
• Note important warning signs to watch for in order to prevent
suicide.
• Describe the symptoms of schizophrenia.
• Contrast acute and chronic schizophrenia.
• Analyze how brain abnormalities and viral infections contribute to
the incidence of schizophrenia.
•
•
•
•
•
•
Explore the evidence of genetic influence on schizophrenia.
Describe the early warning signs of schizophrenia in children.
Identify somatic symptoms and their related disorders.
Identify dissociative disorders.
Evaluate why dissociative disorders are controversial.
Identify psychological and genetic factors that lead to anorexia
nervosa, bulimia nervosa, and binge-eating disorder.
• Contrast the 3 clusters of personality disorders.
• Describe behaviors that characterize antisocial personality
disorder.
Identify different anxiety disorders.
Abnormal Behavior
MyersPsyAP_TE_2e_U12.indd 1
Unit XII
649a
3/5/14 11:02 AM
Alignment to AP® Course Description
Topic 12: Research Methods (8–10% of AP® Examination)
Module
Topic
Essential Questions
Module 65
Defining Psychological Disorders
• How do we define psychological disorders?
Understanding Psychological
Disorders
• How are psychological disorders different from “normal” behavior?
Classifying Psychological Disorders
• What are psychological disorders?
Labeling Psychological Disorders
• What are the implications of the label “psychological disorder” ?
Rates of Psychological Disorders
• Why are psychological disorders more prevalent in some countries or
cultures than in others?
Module 66
Module 67
Module 68
Module 69
3c
649b
Generalized Anxiety Disorder
• How does generalized anxiety disorder affect people?
Panic Disorder
• How does panic disorder affect people?
Phobias
• How do phobias affect people?
Obsessive-Compulsive Disorder
• How does obsessive-compulsive disorder affect people?
Posttraumatic Stress Disorder
• How does posttraumatic stress affect people?
Understanding Anxiety Disorders,
OCD, and PTSD
• What are the roots of anxiety disorders, OCD, and PTSD in people?
Major Depressive Disorder
• How does major depressive disorder affect people?
Bipolar Disorder
• How does bipolar disorder affect people?
Understanding Mood Disorders
• What are the roots of mood disorders in people?
Symptoms of Schizophrenia
• How can you tell someone has schizophrenia?
Onset and Development of
Schizophrenia
• When and how does schizophrenia develop?
Understanding Schizophrenia
• What are the roots of schizophrenia in people?
Somatic Symptoms and Related
Disorders
• How do psychological issues manifest themselves in our general
Dissociative Disorders
• What does it mean to dissociate?
Eating Disorders
• Why are eating disorders so difficult to treat and recover from?
Personality Disorders
• How do personality disorders affect people?
physical well-being?
Unit
Research
M≠≠ethods:
Thinking Critically With Psychological Science
UnitI XII
Abnormal
Behavior
MyersPsyAP_TE_2e_U12.indd 2
3/5/14 11:02 AM
Unit Resources
Module 65
Module 67
TEACHER DEMONSTRATION
STUDENT ACTIVITIES
• Diasthesis-Stress Model and Peanut Butter Sandwiches
•
•
•
•
•
STUDENT ACTIVITIES
• Fact or Falsehood?
• Utilizing Contradictions in Students’ Implicit Definitions of “Mental
Disorder”
Fact or Falsehood?
The Zung Self-Rating Depression Scale
Depression and Memory
Loneliness
The Automatic Thoughts Questionnaire
• Exploring Psychological Disorders on the Internet
• The Effects of Labeling
• Multiple Causation
Module 68
FLIP IT VIDEO
• Magical Ideation Scale
• The DSM and un-DSM: Defining Illness and Health
Module 66
STUDENT ACTIVITIES
•
•
•
•
•
TEACHER DEMONSTRATION
STUDENT ACTIVITY
• Fact or Falsehood?
FLIP IT VIDEO
• Schizophrenia: Positive and Negative Symptoms
Fact or Falsehood?
Taylor Manifest Anxiety Scale
Social Phobias
Fear Survey
Obsessive-Compulsive Disorder
FLIP IT VIDEO
• Classifying Panic
Module 69
TEACHER DEMONSTRATION
• Antisocial Personality Disorder
STUDENT ACTIVITY
•
•
•
•
Fact or Falsehood?
Questionnaire of Experiences of Dissociation
A Survey of Eating Habits
A Week of Food
Abnormal Behavior
MyersPsyAP_TE_2e_U12.indd 3
Unit XII
649c
3/5/14 11:02 AM
Pacing Guide
Module
Topic
Module 65
Defining Psychological Disorders
Understanding Psychological Disorders
Classifying Psychological Disorders
Labeling Psychological Disorders
Rates of Psychological Disorders
Module 66
Module 67
Standard Schedule Days
Generalized Anxiety Disorder
Panic Disorder
Phobias
Obsessive-Compulsive Disorder
Posttraumatic Stress Disorder
Understanding Anxiety Disorders, OCD, and PTSD
Major Depressive Disorder
Bipolar Disorder
Understanding Mood Disorders
Block Schedule Days
1
1
1
MyersAP_SE_2e
1/2
1/2
Module 68
Module 69
649d
Symptoms of Schizophrenia
Onset and Development of Schizophrenia
Understanding Schizophrenia
1/2
Somatic Symptoms and Related Disorders
Dissociative Disorders
Eating Disorders
Personality Disorders
1
Unit XII
MyersPsyAP_TE_2e_U12.indd 4
1/2
Abnormal Behavior
3/11/14 12:04 PM
Unit XII
Abnormal Behavior
Modules
65
Introduction to Psychological Disorders
66
Anxiety Disorders, Obsessive-Compulsive Disorder, and
Posttraumatic Stress Disorder
67
Mood Disorders
68
Schizophrenia
69
Other Disorders
I felt the need to clean my room at home in Indianapolis every Sunday and would
spend four to five hours at it. I would take every book out of the bookcase, dust and
put it back. . . . I couldn’t stop.
Marc, diagnosed with obsessive-compulsive disorder (from Summers, 1996)
Whenever I get depressed it’s because I’ve lost a sense of self. I can’t find reasons to
like myself. I think I’m ugly. I think no one likes me.
Greta, diagnosed with depression (from Thorne, 1993, p. 21)
Voices, like the roar of a crowd, came. I felt like Jesus; I was being crucified.
Stuart, diagnosed with schizophrenia (from Emmons et al., 1997)
People are fascinated by the exceptional, the unusual, the abnormal. “The sun
shines and warms and lights us and we have no curiosity to know why this is so,”
observed Ralph Waldo Emerson, “but we ask the reason of all evil, of pain, and
hunger, and [unusual] people.”
Why such fascination with disturbed people? Even when we are well, do we
see in them something of ourselves? At various moments, all of us feel, think, or
act the way disturbed people do much of the time. We, too, get anxious, depressed,
withdrawn, suspicious, or deluded, just less intensely and more briefly. No wonder studying psychological disorders sometimes evokes an eerie sense of selfrecognition, one that illuminates our own personality. “To study the abnormal is
the best way of understanding the normal,” proposed William James (1842–1910).
649
MyersAP_SE_2e_Mod65_B.indd 649
1/21/14 9:35 AM
Abnormal Behavior
MyersPsyAP_TE_2e_U12.indd 649
Unit XII
649
3/7/14 9:54 AM
650
Unit XII Abnormal Behavior
Another reason for our curiosity is that so many of us have felt, either personally
TEACH
TR M
TRM
or through friends or family, the bewilderment and pain of a psychological disorder
Discussion Starter
that may bring unexplained physical symptoms, irrational fears, or a feeling that
life is not worth living. Indeed, as members of the human family, most of us will at
Use the Module 65 Fact or Falsehood?
activity from the TRM to introduce the
concepts from this module.
some point encounter a person with a psychological disorder.
The World Health Organization (WHO, 2010) reports that, worldwide, some 450
million people suffer from mental or behavioral disorders. These disorders account
for 15.4 percent of the years of life lost due to death or disability—scoring slightly
TEACH
TR M
TRM
below cardiovascular conditions and slightly above cancer (Murray & Lopez, 1996).
Teaching Tip
Rates and symptoms of psychological disorders vary by culture, but two terrible
Before reading the unit, have students
form groups of 4 or 5 and develop a
definition for psychological disorder.
Instruct the students to be specific,
identifying criteria they would apply
to draw the line between normality
and abnormality. Have each group
report its definition to the class. Spend
the rest of the session considering the
difficulty of defining the term.
Use Student Activity: Utilizing
Contradictions in Students’ Implicit
Definitions of “Mental Disorder” from
the TRM to explore the difficulty in
defining mental illness.
TEACH
Flip It
Students can get additional help
understanding mental illness and
health by watching the Flip It Video:
The DSM and un-DSM: Defining Illness
and Health.
maladies appear more consistently worldwide: depression and schizophrenia.
Module 65
Introduction to Psychological Disorders
Module Learning Objectives
65-1
Discuss how we draw the line between normality and disorder.
65-2
Discuss the controversy over the diagnosis of attentiondeficit/hyperactivity disorder.
65-3
Contrast the medical model with the biopsychosocial approach to
psychological disorders.
65-4
Describe how and why clinicians classify psychological disorders.
65-5
Explain why some psychologists criticize the use of diagnostic labels.
65-6
chinaf
ace/G
etty Im
ages
Discuss the prevalence of psychological disorders, and summarize
the findings on the link between poverty and serious psychological
disorders.
M
ost people would agree that someone who is too depressed to get out of bed for weeks
at a time has a psychological disorder. But what about those who, having experienced
a loss, are unable to resume their usual social activities? Where should we draw the
line between sadness and depression? Between zany creativity and bizarre irrationality?
Between normality and abnormality? Let’s start with these questions:
MyersAP_SE_2e_Mod65_B.indd 650
650
Unit XII
MyersPsyAP_TE_2e_U12.indd 650
1/21/14 9:35 AM
Abnormal Behavior
3/5/14 11:02 AM
MyersAP_SE_2e
Introduction to Psychological Disorders
•
How should we define psychological disorders?
•
How should we understand disorders? How do underlying biological factors contribute
to disorder? How do troubling environments influence our well-being? And how do
these effects of nature and nurture interact?
•
How should we classify psychological disorders? And can we do so in a way that
allows us to help people without stigmatizing them with labels?
Module 65
651
ENGAGE
“Who in the rainbow can draw
the line where the violet tint ends
and the orange tint begins?
Distinctly we see the difference
of the colors, but where exactly
does the one first blendingly enter
into the other? So with sanity and
insanity.” -HERMAN MELVILLE, BILLY
BUDD, SAILOR, 1924
Active Learning
Have students investigate the behavior of people throughout history
that might be considered abnormal
according to today’s standards:
Defining Psychological Disorders
65-1
How should we draw the line between normality and disorder?
iStock/Thinkstock
Carol Beckwith
A psychological disorder is a syndrome marked by a “clinically significant disturbance
in an individual’s cognition, emotion regulation, or behavior” (American Psychiatric Association, 2013). Disturbed, or dysfunctional, behaviors are maladaptive—they interfere with
normal day-to-day life. An intense fear of spiders may be abnormal, but if it doesn’t interfere
with your life, it is not a disorder. Marc’s cleaning rituals (from this unit’s opening) did interfere with his work and leisure. If occasional sad moods persist and become disabling, they
may signal a psychological disorder. Distress often accompanies dysfunctional behaviors.
Marc, Greta, and Stuart were all distressed by their behaviors or emotions.
Over time, definitions of what makes for a “significant disturbance” have varied. From
1952 through December 9, 1973, homosexuality was classified as a mental illness. By day’s end
on December 10, it was not. The American Psychiatric Association had dropped homosexuality as a disorder because more and more of its members no longer viewed it as a psychological
problem. (Later research revealed that the stigma and stresses that often accompany homosexuality, however, increase the risk of mental health problems [Hatzenbuehler et al., 2009;
Meyer, 2003].) In this new century, controversy swirls over the frequent diagnosing of children
with attention-deficit/hyperactivity disorder (see Thinking Critically About: ADHD—Normal
High Energy or Disordered Behavior? on the next page).
Understanding Psychological Disorders
65-3
How do the medical model and the
biopsychosocial approach understand
psychological disorders?
To explain puzzling behavior, people in earlier times often presumed
the work of strange forces—the movements of the stars, godlike
powers, or evil spirits. Had you lived during the Middle Ages, you
might have said, “The devil made him do it,” and you might have
1/21/14 9:35 AM
MyersAP_SE_2e_Mod65_B.indd 651
What health practices were
popular in earlier times? (Consider
the use of leeches to rid patients
of “bad blood,” as well as a practice
by ancient cultures of cutting a hole
in the skull to release evil spirits
that “caused” migraines or epilepsy.)
What social practices were popular
in earlier times? (Consider that
bundling was a common practice
in colonial America. When a young
man came to visit his girlfriend and
had to stay overnight, the mother
would sew the suitor into the bed,
making it difficult for him to take
advantage of her daughter.)
psychological disorder
a syndrome marked by a clinically
significant disturbance in an
individual’s cognition, emotion
regulation, or behavior. (Adapted
from American Psychiatric
Association, 2013.)
Culture and normality
Young men of the West
African Wodaabe tribe
put on elaborate makeup
and costumes to attract
women. Young American
men may buy flashy cars
with loud stereos to do the
same. Each culture may
view the other’s behavior as
abnormal.
TEACH
Teaching Tip
Have students brainstorm about
behaviors that, when taken out of
context, could be considered abnormal. Here are some ideas to get them
started:
Yesterday’s “therapy” In other
times and places, psychologically
disordered people sometimes
received brutal treatments,
including the trephination
evident in this Stone Age skull.
Drilling skull holes like these may
have been an attempt to release
evil spirits and cure those with
mental disorders. Did this
patient survive the “cure”?
Arguing heatedly with someone
Praying aloud or chanting outside
of a religious service
Dancing in excitement over
receiving a good grade or hearing
good news
John W. Verano
1/21/14 9:35 AM
Introduction to Psychological Disorders
MyersPsyAP_TE_2e_U12.indd 651
Module 65
651
3/5/14 11:02 AM
Unit XII Abnormal Behavior
652
approved of a cure to rid the evil force by exorcising the demon. Until the last two centuries, “mad” people were sometimes caged in zoo-like conditions or given “therapies”
appropriate to a demon: beatings, burning, or castration. In other times, therapy included
pulling teeth, removing lengths of intestines, cauterizing the clitoris, or giving transfusions of animal blood (Farina, 1982).
TEACH
TR M
TRM
Teaching Tip
Some of your students may be identified as having attention–deficit/hyperactivity disorder (ADHD). Point out
to students that the DSM criteria do
not include a diagnosis of attention–
deficit disorder (ADD), only ADHD. If
students are willing, encourage them
to share how they process information
and external stimuli.
Use Student Activity: Exploring
Psychological Disorders on the Internet from the TRM to help students
learn more about psychological
disorders.
Thinking Critically About
ADHD—Normal High Energy or Disordered Behavior?
65-2
Why is there controversy over
attention-deficit/hyperactivity disorder?
Eight-year-old Todd has always been energetic. At home, he
chatters away and darts from one activity to the next, rarely
settling down to read a book or focus on a game. At play, he is
reckless and overreacts when playmates bump into him or take
one of his toys. At school, his exasperated teacher complains
that fidgety Todd doesn’t listen, follow instructions, or stay in his
seat and do his lessons. As he matures to adulthood, Todd’s
hyperactivity likely will subside, but his inattentiveness may persist (Kessler et al., 2010).
If taken for a psychological evaluation, Todd may be diagnosed with attention-deficit/hyperactivity disorder (ADHD),
as are some 11 percent of American 4- to 17-year-olds who display its key symptoms (extreme inattention, hyperactivity, and impulsivity) (Schwarz & Cohen, 2013). Studies also find 2.5 percent
of adults—though a diminishing number with age—exhibiting
ADHD symptoms (Simon et al., 2009). Psychiatry’s new diagnostic manual loosens the criteria for adult ADHD, leading critics to fear increased diagnosis and overuse of prescription drugs
(Frances, 2012).
To skeptics, being distractible, fidgety, and impulsive
sounds like a “disorder” caused by a single genetic variation:
a Y chromosome. And sure enough, ADHD is diagnosed three
times more often in boys than in girls. Does energetic child +
boring school = ADHD overdiagnosis? Is the label being applied
to healthy schoolchildren who, in more natural outdoor environments, would seem perfectly normal?
Skeptics think so. In the decade after 1987, they note, the
proportion of American children being treated for ADHD nearly
quadrupled (Olfson et al., 2003). How commonplace the diagnosis is depends in part on teacher referrals. Some teachers refer lots of kids for ADHD assessment, others none. ADHD rates
have varied by a factor of 10 in different counties of New York
State (Carlson, 2000). Although African-American youth display
more ADHD symptoms than do Caucasian youth, they less often
receive an ADHD diagnosis (Miller et al., 2009). Depending on
where they live, children who are “a persistent pain in the neck in
school” are often diagnosed with ADHD and given powerful prescription drugs, notes Peter Gray (2010). But the problem resides
less in the child, he argues, than in today’s abnormal environment
that forces children to do what evolution has not prepared them
to do—to sit for long hours in chairs.
On the other side of the debate are those who argue that
the more frequent diagnoses of ADHD today reflect increased
awareness of the disorder, especially in those areas where rates
are highest. They acknowledge that diagnoses can be subjective and sometimes inconsistent—ADHD is not as objectively
defined as is a broken arm. Nevertheless, declared the World
Federation for Mental Health (2005), “there is strong agreement
among the international scientific community that ADHD is a
real neurobiological disorder whose existence should no longer
be debated.” A consensus statement by 75 researchers noted
that in neuroimaging studies, ADHD has associations with abnormal brain activity patterns (Barkley et al., 2002).
What, then, is known about ADHD’s causes? It is not
caused by too much sugar or poor schools. There is mixed evidence suggesting that extensive TV watching and video gaming
are associated with reduced cognitive self-regulation and ADHD
(Bailey et al., 2011; Courage & Setliff, 2010; Ferguson, 2011).
ADHD often coexists with a learning disorder or with defiant
and temper-prone behavior. ADHD is heritable, and research
teams are sleuthing the culprit genes and abnormal neural pathways (Nikolas & Burt, 2010; Poelmans et al., 2011; Volkow et
al., 2009; Williams et al., 2010). It is treatable with medications
such as Ritalin and Adderall, which are considered stimulants
but help calm hyperactivity and increase the ability to sit and
focus on a task—and to progress normally in school (Barbaresi
et al., 2007). Psychological therapies, such as those focused
on shaping behaviors in the classroom and at home, have also
helped address the distress of ADHD (Fabiano et al., 2008).
The bottom line: Extreme inattention, hyperactivity, and impulsivity can derail social, academic, and vocational achievements, and these symptoms can be treated with medication
and other therapies. But the debate continues over whether
normal rambunctiousness is too often diagnosed as a psychiatric disorder, and whether there is a cost to the long-term use of
stimulant drugs in treating ADHD.
attention-deficit/hyperactivity disorder (ADHD)
a psychological disorder marked by the appearance by age 7
of one or more of three key symptoms: extreme inattention,
hyperactivity, and impulsivity.
MyersAP_SE_2e_Mod65_B.indd 652
1/21/14 9:35 AM
ENGAGE
Active Learning
Have students research the treatment of the
mentally ill throughout history. Use these ideas
as a starting point:
652
Unit XII
MyersPsyAP_TE_2e_U12.indd 652
When and where was the first hospital for
the mentally ill opened? (The Society of
Friends [Quakers] opened the first private
institution in 1813. The oldest governmentrun mental institution is St. Elizabeth’s in
Washington, D.C.)
For what reasons could a person be
committed in earlier times? (People could be
committed for any reason. Men who wanted a
divorce could have their wives committed, and
girls who became pregnant out of wedlock
were sometimes also institutionalized.)
Abnormal Behavior
3/5/14 11:02 AM
MyersAP_SE_2e
Introduction to Psychological Disorders
Module 65
653
The Medical Model
Dance in a Madhouse, 1917 (litho), Bellows, George Wesley (1882–1925)/
San Diego Museum of Art, USA/Museum Purchase/The Bridgeman Art Library
In opposition to brutal treatments, reformers, including Philippe Pinel (1745–1826) in
France, insisted that madness is not demon possession but a sickness of the mind caused by
severe stresses and inhumane conditions. For Pinel and others, “moral treatment” included
boosting patients’ morale by unchaining them and talking with them, and by replacing brutality with gentleness, isolation with activity, and filth with clean air and sunshine. While
such measures did not often cure patients, they were certainly more humane.
By the 1800s, the discovery that syphilis infects the brain and distorts the mind drove
further gradual reform. Hospitals replaced asylums, and the medical world began searching for physical causes and treatments of mental disorders. Today, this medical model is
recognizable in the terminology of the mental health movement: A mental illness (also called
a psychopathology) needs to be diagnosed on the basis of its symptoms and treated through
therapy, which may include time in a psychiatric hospital.
The medical perspective has gained credibility from recent discoveries that genetically influenced abnormalities in brain structure and biochemistry contribute to many disorders. But as
we will see, psychological factors, such as chronic or traumatic stress, also play an important role.
ENGAGE
Enrichment
Philippe Pinel, once the personal physician to Napoleon, devoted his life to
helping the mentally ill. He directed
the Paris men’s asylum, where some
patients had been chained to the
walls for decades. During his tenure,
the death rates for patients went from
60 to 10 percent.
medical model the concept that
diseases, in this case psychological
disorders, have physical causes
that can be diagnosed, treated, and,
in most cases, cured, often through
treatment in a hospital.
ENGAGE
“Moral treatment” Under Philippe
Pinel’s influence, hospitals sometimes
sponsored patient dances, often
called “lunatic balls,” depicted in this
painting by George Bellows (Dance in
a Madhouse).
Enrichment
Dorothea Dix began her crusade for
the humane treatment of mentally
ill patients when she started to tutor
inmates at a women’s prison. Men
and women who were mentally ill or
disabled were being housed there
with hardened female criminals. Dix
advocated for reforms in all existing
U.S. states and throughout Europe.
The Biopsychosocial Approach
Today’s psychologists contend that all behavior, whether called normal or disordered, arises
from the interaction of nature (genetic and physiological factors) and nurture (past and
present experiences). To presume that a person is “mentally ill,” they say, attributes the
condition to a “sickness” that must be identified and cured. But difficulty in the person’s
environment, the person’s current interpretations of events, or the person’s bad habits and
poor social skills may also be factors.
Evidence of such effects comes from links between specific disorders and cultures
(Beardsley, 1994; Castillo, 1997). Cultures differ in their sources of stress, and they produce
different ways of coping. The eating disorders anorexia nervosa and bulimia nervosa, for
example, have occurred mostly in Western cultures. In Malaysia, amok describes a sudden
outburst of violent behavior (thus the phrase “run amok”). Latin America lays claim to susto,
a condition marked by severe anxiety, restlessness, and a fear of black magic. Taijin-kyofusho,
social anxiety about one’s appearance combined with a readiness to blush and a fear of eye
contact, appears in Japan, as does the extreme withdrawal of hikikomori. Such disorders may
share an underlying dynamic (such as anxiety) while differing in the symptoms (an eating
problem or a type of fear) manifested in a particular culture.
But not all disorders are culture-bound. Depression and schizophrenia occur worldwide. From Asia to Africa and across the Americas, schizophrenia’s symptoms often include
irrationality and incoherent speech.
1/21/14 9:35 AM
MyersAP_SE_2e_Mod65_B.indd 653
FYI
Increasingly, North American
disorders, such as eating
disorders, are, along with
McDonald’s and MTV, spreading
across the globe (Watters, 2010).
1/21/14 9:35 AM
ENGAGE
Active Learning
The medical model of mental illness has led to
the development of several drugs for treating a
wide variety of mental disorders—from schizophrenia to depression to obsessive-compulsive
disorder. Have students research the drugs
used to treat different disorders, focusing on
the brain chemistry linked to each disorder:
Which drugs treat depression and bipolar
disorder most successfully? Which
neurotransmitters are linked to these
disorders?
Which drugs treat anxiety disorders most
successfully? Which neurotransmitters are
linked to these disorders?
Which drugs treat schizophrenia most
successfully? Which neurotransmitters are
linked to this disorder?
Introduction to Psychological Disorders
MyersPsyAP_TE_2e_U12.indd 653
Module 65
653
3/5/14 11:02 AM
654
Unit XII Abnormal Behavior
TEACH
Biological influences:
…evolution
…individual genes
…brain structure and chemistry
Diversity Connections
In Japan, hissing is a polite way to
show respect for superiors.
Among the Karaki of New Guinea,
a man is considered abnormal if he
has not engaged in homosexual
behavior before marriage.
Figure 65.1
The biopsychosocial approach to
psychological disorders Today’s
Respect for diversity sounds like a
wonderful ideal, but how should
we react when it clashes with some
deeply held value, such as gender
equality? Consider the following:
Should the U.S. government be
accepting of the genital mutilation
of women, a brutal practice that
is common in certain African
countries?
Female U.S. military personnel
in Saudi Arabia have repeatedly
protested regulations that require
them to wear a veil outside their
bases. Should these personnel be
forced to adhere to Saudi law, or
should they be allowed to follow
U.S. customs?
Should we as a nation object to
the law in Saudi Arabia that forbids
women to drive?
In Sudan, a woman may not
leave the country without the
permission of her husband, father,
or brother. Should we object to
such a practice?
What are the limits of tolerance?
What types of outside cultural
practices should we accept, and to
which ones should we object?
654
Unit XII
MyersPsyAP_TE_2e_U12.indd 654
To assess the whole set of influences—genetic predispositions
and physiological states, inner
psychological dynamics, and social and cultural circumstances—
the biopsychosocial model helps
(FIGURE 65.1). This approach
recognizes that mind and body
are inseparable. Negative emotions contribute to physical illness,
and physical abnormalities contribute to negative emotions. We
are mind embodied and socially
embedded.
Classifying Psychological Disorders
65-4
FYI
A book of case illustrations
accompanying the previous DSM
edition provides several examples
for this unit.
Critical Questions
Social-cultural influences:
…roles
…expectations
…definitions of normality and
disorder
psychology studies how biological,
psychological, and social-cultural
factors interact to produce specific
psychological disorders.
In Thailand, public displays of
affection between men and
women are unacceptable.
Interestingly, however, men
holding hands is a sign of
friendship. In addition, the use of
straws is considered vulgar.
ENGAGE
Psychological
P
Ps
all
disorder
© cultura/Corbis
Some cultures have very different
social practices that may be considered abnormal to people living in the
United States:
Psychological influences:
…stress
…trauma
…learned helplessness
…mood-related perceptions and
memories
DSM-5 the American Psychiatric
Association’s Diagnostic and
Statistical Manual of Mental
Disorders, Fifth Edition; a widely
used system for classifying
psychological disorders.
How and why do clinicians classify psychological disorders?
In biology and the other sciences, classification creates order. To classify an animal as a
“mammal” says a great deal—that it is warm-blooded, has hair or fur, and nourishes its
young with milk. In psychiatry and psychology, too, classification orders and describes
symptoms. To classify a person’s disorder as “schizophrenia” suggests that the person talks
incoherently; hallucinates or has delusions (bizarre beliefs); shows either little emotion or
inappropriate emotion; or is socially withdrawn. “Schizophrenia” provides a handy shorthand for describing a complex disorder.
In psychiatry and psychology, diagnostic classification aims not only to describe a
disorder but also to predict its future course, imply appropriate treatment, and stimulate
research into its causes. Indeed, to study a disorder we must first name and describe it.
The most common system for describing disorders and estimating how often they occur
is the American Psychiatric Association’s 2013 Diagnostic and Statistical Manual of Mental Disorders, now in its fifth edition (DSM-5). Physicians and mental health workers use
the detailed “diagnostic criteria and codes” in the DSM-5 to guide medical diagnoses
and define who is eligible for treatments, including medication. For example, a person
may be diagnosed with and treated for “insomnia disorder” if he or she meets all of the
following criteria:
•
Is dissatisfied with sleep quantity or quality (difficulty initiating, maintaining, or
returning to sleep).
•
Sleep disturbance causes distress or impairment in everyday functioning.
•
Occurs at least three nights per week.
•
Present for at least three months.
•
Occurs despite adequate opportunity for sleep.
•
Is not explained by another sleep disorder (such as narcolepsy).
•
Is not caused by substance use or abuse.
•
Is not caused by other mental disorders or medical conditions.
In this new DSM edition, some diagnostic labels have changed. For example, “autism”
and “Asperger’s syndrome” are no longer included; they have been combined into “autism
spectrum disorder.” “Mental retardation” has become “intellectual disability.” New categories include “hoarding disorder” and “binge-eating disorder.”
MyersAP_SE_2e_Mod65_B.indd 654
ENGAGE
1/21/14 9:35 AM
Online Activities
Have students further explore the new
changes in the DSM-5 by directing them to the
website http://www.dsm5. org/Documents/
changes%20from%20dsm-iv-tr%20to%20dsm5. pdf. Encourage them to make a chart outlining the changes to this important document.
Abnormal Behavior
3/5/14 11:02 AM
MyersAP_SE_2e
Introduction to Psychological Disorders
655
TEACH
Teaching Tip
ScienceCartoonsPlus.com
Some new or altered diagnoses are controversial. “Disruptive mood dysregulation disorder” is a new DSM-5 diagnosis for children “who exhibit persistent irritability and frequent episodes of behavior outbursts three or more times a week for more than a year.” Will
this diagnosis assist parents who struggle with unstable children, or will it “turn temper
tantrums into a mental disorder” and lead to overmedication, as the chair of the previous
DSM edition has warned (Frances, 2012)?
Critics have long faulted the DSM for casting too wide a net and bringing “almost any kind of behavior within the compass of psychiatry” (Eysenck et al., 1983).
They worry that the DSM-5 will extend the pathologizing of everyday life—for
example, by turning bereavement grief into depression and boyish rambunctiousness into ADHD (Frances, 2013). Others respond that depression and hyperactivity,
though needing careful definition, are genuine disorders even, for example, those triggered by a major life stress such as a death when the grief does not go away (Kendler,
2011; Kupfer, 2012).
Module 65
“I’m always like this, and my family
was wondering if you could prescribe
a mild depressant.”
Labeling Psychological Disorders
65-5
Why do some psychologists criticize the use of diagnostic labels?
The DSM has other critics who register a more fundamental complaint—that these labels
are at best arbitrary and at worst value judgments masquerading as science. Once we label
a person, we view that person differently (Farina, 1982). Labels create preconceptions that
guide our perceptions and our interpretations.
In a now-classic study of the biasing power of labels, David Rosenhan (1973) and seven
others went to hospital admissions offices, complaining of “hearing voices” saying empty,
hollow, and thud. Apart from this complaint and giving false names and occupations, they
answered questions truthfully. All eight normal people were misdiagnosed with disorders.
Should we be surprised? As one psychiatrist noted, if someone swallows blood, goes
to an emergency room, and spits it up, should we fault the doctor for diagnosing a bleeding
ulcer? Surely not. But what followed the diagnosis in the Rosenhan study was startling. Until being released an average of 19 days later, the “patients” exhibited no further symptoms
such as hearing voices. Yet after analyzing their (quite normal) life histories, clinicians were
able to “discover” the causes of their disorders, such as reacting with mixed emotions about
a parent. Even the routine behavior of taking notes was misinterpreted as a symptom.
Labels matter. When people in another experiment watched videotaped interviews,
those told the interviewees were job applicants perceived them as normal (Langer et al.,
1974, 1980). Those who thought they were watching psychiatric or cancer patients perceived
them as “different from most people.” Therapists who thought an interviewee was a psychiatric patient perceived him as “frightened of his own aggressive impulses,” a “passive,
dependent type,” and so forth. A label can, as Rosenhan discovered, have “a life and an
influence of its own.”
Surveys in Europe and North America have demonstrated the stigmatizing power of
labels (Page, 1977). Getting a job or finding a place to rent can be a challenge for those
known to be just released from prison—or a mental hospital. But as we are coming to
understand that many psychological disorders are diseases of the brain, not failures of
character, the stigma seems to be lifting (Solomon, 1996). Public figures are feeling freer
to “come out” and speak with candor about their struggles with disorders such as depression. And the more contact people have with individuals with disorders, the more accepting their attitudes are (Kolodziej & Johnson, 1996). People express greatest sympathy
for people whose disorders are gender atypical—for men suffering depression (which is
more common among women), or for women plagued by alcohol use disorder (Wirth &
Bodenhausen, 2009).
1/21/14 9:35 AM
MyersAP_SE_2e_Mod65_B.indd 655
ENGAGE
Enrichment
The DSM diagnostic criteria are important not only for doctors deciding
what type of mental illness a person
might have, but also for health care in
general for the mentally ill. Insurance
companies require that mental illness
diagnoses reference the DSM system
in order to pay for services to treat the
disorder. The use of the DSM by insurance providers makes any changes to
it very important.
“One of the unpardonable sins, in
the eyes of most people, is for a
man to go about unlabeled. The
world regards such a person as
the police do an unmuzzled dog,
not under proper control.” -T. H.
HUXLEY, EVOLUTION AND ETHICS, 1893
ENGAGE
TR M
TRM
Critical Questions
How might the issue of labeling affect teachers? Studies show that labeling of students
can bias a teacher’s treatment of a particular
student. However, knowing a student’s diagnosis can help a teacher get that student the
help he or she needs to succeed. How might a
balance be reached? Have students brainstorm
the ways in which teachers can be both made
aware of labels but also remain sensitive to the
limitations such labels may place on students:
What type of training should teachers
receive to help them become more
sensitive to the power of labels?
Should teachers and students be informed
of students’ labels? Why or why not?
How can teachers and students become
proactive about learning disabilities?
Calling someone a “schizophrenic” ?
Calling someone a “person with
schizophrenia” ?
Help students see that the first label
doesn’t recognize the person at all
and implies that the individual is the
condition. The second label acknowledges the person and places the
mental illness in the same categories
as other, less stigmatizing labels.
Use Student Activity: The Effects
of Labeling from the TRM to help students see how labels affect people’s
perceptions.
Introduction to Psychological Disorders
MyersPsyAP_TE_2e_U12.indd 655
Critical Questions
Ask students which of the following
they regard as less stigmatizing:
“My sister suffers from a bipolar
disorder and my nephew from
schizoaffective disorder. There has,
in fact, been a lot of depression
and alcoholism in my family and,
traditionally, no one ever spoke
about it. It just wasn’t done. The
stigma is toxic.” -ACTRESS GLENN
CLOSE, “MENTAL ILLNESS: THE STIGMA
OF SILENCE,” 2009
1/21/14 9:35 AM
ENGAGE
Divide your students into groups,
and give each one a different label
that is commonly used in schools,
such as learning disabled, emotionally
conflicted, gifted, or at-risk. Have the
students research what criteria people
must meet to be given such a label,
and consider how the label might
make them feel about themselves and
their school.
Module 65
655
3/5/14 11:02 AM
Accurate portrayal
ENGAGE
Active Learning
Have your students watch episodes of
popular television shows that depict
people with mental illness.
Ask them to compare the portrayal
with the DSM-5 diagnostic criteria
to see if they match.
Students can also evaluate
whether the portrayal promotes
sensitivity or further stigma with
regard to the particular mental
illness.
Recent films have offered
some realistic depictions
of psychological
disorders. Black Swan
(2010), shown here,
portrayed a main
character suffering a
delusional disorder.
Temple Grandin
(2010) dramatized a
lead character who
successfully copes with
autism spectrum disorder.
A Single Man (2009)
depicted depression.
A P ® E x a m Ti p
Notice that the term insanity comes
out of the legal system. It is not a
psychological or medical diagnosis
and does not appear in the DSM-5.
Nevertheless, stereotypes linger in media
portrayals of psychological disorders. Some are
reasonably accurate and sympathetic. But too often people with disorders are portrayed as objects
of humor or ridicule (As Good as It Gets), as homicidal maniacs (Hannibal Lecter in Silence of the
Lambs), or as freaks (Nairn, 2007). Apart from the
few who experience threatening delusions and
hallucinated voices that command a violent act,
and from those whose dysfunctionality includes
substance abuse, mental disorders seldom lead to
violence (Douglas et al., 2009; Elbogen & Johnson,
2009; Fazel et al., 2009, 2010). In real life, people
with disorders are more likely to be the victims of
violence than the perpetrators (Marley & Bulia, 2001). Indeed, reported the U.S. Surgeon
General’s Office (1999, p. 7), “There is very little risk of violence or harm to a stranger from
casual contact with an individual who has a mental disorder.” (Although most people with
psychological disorders are not violent, those who are create a moral dilemma for society.
For more on this topic, see Thinking Critically About: Insanity and Responsibility.)
Thinking Critically About
ENGAGE
Insanity and Responsibility
Enrichment
“My brain . . . my genes . . . my bad upbringing made me do it.”
Such defenses were anticipated by Shakespeare’s Hamlet. If I
wrong someone when not myself, he explained, “then Hamlet
does it not, Hamlet denies it. Who does it then? His madness.”
Such is the essence of a legal insanity defense. “Insanity” is a
legal rather than a psychological concept, and was created in
1843 after a deluded Scotsman tried to shoot the prime minister (who he thought was persecuting him) but killed an assistant by mistake. Like U.S. President Ronald Reagan’s nearassassin, John Hinckley, Scotsman Daniel M’Naghten was sent
to a mental hospital rather than to prison.
In both cases, the public was outraged. “Hinckley Insane,
Public Mad,” declared one headline. They were mad again when
a deranged Jeffrey Dahmer in 1991 admitted murdering 15 young
men and eating parts of their bodies. They were mad in 1998 when
15-year-old Kip Kinkel, driven by “those voices in my head,” killed his
parents and two fellow Springfield, Oregon, students and wounded
25 others. They were mad in 2002 when Andrea Yates, after being taken off her antipsychotic medication, was tried in Texas for
drowning her five children. And they were mad in 2011, when an
irrational Jared Loughner gunned down a crowd of people, including survivor Congresswoman Gabrielle Giffords, in an Arizona supermarket parking lot. Following their arrest, most of these people
were sent to jails, not hospitals. (Hinckley was sent to a psychiatric
hospital and later, after another trial, Yates was instead hospitalized.)
As Yates’ fate illustrates, 99 percent of those whose insanity defense is accepted are nonetheless institutionalized, often for as long
as those convicted of crimes (Lilienfeld & Arkowitz, 2011).
A landmark study conducted in
1962 by J. B. Calhoun demonstrated
that high population density causes
abnormal behavior in rat populations. Calhoun allowed his rats to live
in an overcrowded environment for
16 months. Some of the pathological
behaviors he observed included the
following:
Aggression: There was increased
fighting among male rats for
dominance, and periodic senseless
fighting with females, juveniles,
and less active males.
Submissiveness: Nondominant
male rats became unusually
submissive and acted as though
they were in a hypnotic trance.
Sexual deviance: Nondominant
males would not follow normal
rat “rules” for mating. This included
following females into burrows
and often eating the young found
inside.
Nesting abnormalities: The female
rats would eventually fail to build
nests, birthing their babies directly
on the floor.
High infant mortality: Rates ranged
from 80–96 percent.
MyersAP_SE_2e_Mod65_B.indd 656
HANDOUT/Reuters/Corbis
TR M
TRM
Unit XII Abnormal Behavior
Protozoa Pictures/Phoenix Pictures/The Kobal Collection
656
Jail or hospital? Jared Lee Loughner
was charged with the 2011 Tucson,
Arizona, shooting that killed six people and
left over a dozen others injured, including
U.S. Representative Gabrielle Giffords.
Loughner had a history of mental health
issues, including paranoid beliefs, and was
diagnosed with schizophrenia. Usually,
however, schizophrenia is only associated
with violence when accompanied by
substance abuse (Fazel et al., 2009).
Most people with psychological disorders are not violent.
But what should society do with those who are? What do we
do with disturbed individuals who mow down innocents at
movie theaters and schools? Sometimes there is nothing to
be done, as in the case of the 2012 Sandy Hook Elementary
School tragedy in Connecticut, where the shooter’s final fatal
shot was self-inflicted. Many people who have been executed
or are now on death row have been limited by intellectual disability or motivated by delusional voices. The State of Arkansas
forced one murderer with schizophrenia, Charles Singleton, to
take two anti-psychotic drugs—in order to make him mentally
competent, so that he could then be put to death.
Which of Yates’ two juries made the right decision? The first,
which decided that people who commit such rare but terrible
crimes should be held responsible? Or the second, which decided
to blame the “madness” that clouds their vision? As we come to
better understand the biological and environmental basis for all human behavior, from generosity to vandalism, when should we—
and should we not—hold people accountable for their actions?
1/21/14 9:35 AM
Use Student Activity: Multiple Causation from the TRM to help students
explore the different causes of mental
illness.
656
Unit XII
MyersPsyAP_TE_2e_U12.indd 656
Abnormal Behavior
3/5/14 11:02 AM
MyersAP_SE_2e
Introduction to Psychological Disorders
Module 65
657
Not only can labels bias perceptions, they can also change reality. When teachers are
told certain students are “gifted,” when students expect someone to be “hostile,” or when
interviewers check to see whether someone is “extraverted,” they may act in ways that elicit
the very behavior expected (Snyder, 1984). Someone who was led to think you are nasty
may treat you coldly, leading you to respond as a mean-spirited person would. Labels can
serve as self-fulfilling prophecies.
But let us remember the benefits of diagnostic labels. Mental health professionals use labels to communicate about their cases, to comprehend the underlying causes, and to discern
effective treatment programs. Diagnostic definitions also inform patient self-understandings.
And they are useful in research that explores the causes and treatments of disordered behavior.
ENGAGE
Critical Questions
Have students consider that statistically speaking, any behavior is atypical
if it is not exhibited by 68 percent of
the people in a particular group or culture. What common behaviors would
be viewed as abnormal if we followed
this definition of abnormality?
Rates of Psychological Disorders
65-6
How many people suffer, or have suffered, from a psychological
disorder? Is poverty a risk factor?
Who is most vulnerable to psychological disorders? At what times of life? To answer
such questions, various countries have conducted lengthy, structured interviews with
representative samples of thousands of their citizens. After asking hundreds of questions that probed for symptoms—“Has there ever been a period of two weeks or
more when you felt like you wanted to die?”—the researchers have estimated the
current, prior-year, and lifetime prevalence of various disorders.
United States
How many people have, or have had, a
psychological disorder? More than most of
Ukraine
us suppose:
Table 65.1
•
•
1/21/14 9:35 AM
The U.S. National Institute of Mental
Health (2008, based on Kessler et
al., 2005) estimates that 26 percent
of adult Americans “suffer from a
diagnosable mental disorder in a
given year” (TABLE 65.1).
A large-scale World Health
Organization (2004a) study—based
on 90-minute interviews of 60,463
people—estimated the number of
prior-year mental disorders in 20
countries. As FIGURE 65.2 displays,
the lowest rate of reported mental
disorders was in Shanghai, the highest
rate in the United States. Moreover,
immigrants to the United States from
Mexico, Africa, and Asia average
better mental health than their native
U.S. counterparts (Breslau et al., 2007;
Maldonado-Molina et al., 2011).
For example, compared with people
who have recently immigrated from
Mexico, Mexican-Americans born in
the United States are at greater risk
of mental disorder—a phenomenon
known as the immigrant paradox
(Schwartz et al., 2010).
MyersAP_SE_2e_Mod65_B.indd 657
Percentage
of Americans Reporting
Selected Psychological
Disorders in the Past Year
Psychological
Disorder
Percentage
3.1
Social anxiety
disorder
6.8
Phobia of specific
object or situation
8.7
Who would suffer discrimination if
this standard were enforced?
What religious practices would
be considered abnormal for your
community?
What ways of life would be
abnormal? Single-parent
households? Two-parent
households?
What ethnic groups would be
considered abnormal?
What styles of dress? What music
preferences? What post–high
school choices?
France
Colombia
Lebanon
Netherlands
Mexico
Generalized anxiety
Belgium
Spain
Germany
ENGAGE
Beijing
Mood disorder
9.5
Obsessivecompulsive disorder
(OCD)
1.0
Schizophrenia
1.1
Posttraumatic stress
disorder (PTSD)
3.5
TR M
TRM
Japan
Italy
Nigeria
Shanghai
0%
Attention-deficit/
hyperactivity disorder
(ADHD)
4.1
Any mental disorder
26.2
Source: National Institute of Mental Health, 2008.
10%
20%
30%
Any mental disorder
Serious mental disorder
Figure 65.2
Prior-year prevalence of disorders
in selected areas From World Health
Organization (WHO, 2004a) interviews in
20 countries.
1/21/14 9:35 AM
Introduction to Psychological Disorders
MyersPsyAP_TE_2e_U12.indd 657
Critical Questions
Have students look at Figure 65.2 and
speculate about why major mental illness is more prevalent in the
United States than in other nations or
cultures. Could awareness of mental illness lead to more diagnoses?
Or do environmental conditions in
the United States contribute to the
increased incidence of mental illness?
If so, what might those conditions be?
Help students see that multiple factors contribute to incidence rates of
mental illness.
Use Teacher Demonstration:
Diasthesis-Stress Model and Peanut
Butter Sandwiches from the TRM
to help students see the effects of
genetic predisposition and stress on
performance.
Module 65
657
3/5/14 11:02 AM
658
Unit XII Abnormal Behavior
Who is most vulnerable to mental disorders? As we have seen, the answer varies with
the disorder. One predictor of mental disorder, poverty, crosses ethnic and gender lines. The
incidence of serious psychological disorders has been doubly high among those below the
poverty line (CDC, 1992). Like so many other correlations, the poverty-disorder association
raises a chicken-and-egg question: Does poverty cause disorders? Or do disorders cause
poverty? It is both, though the answer varies with the disorder. Schizophrenia understandably leads to poverty. Yet the stresses and demoralization of poverty can also precipitate disorders, especially depression in women and substance use disorder in men (Dohrenwend
et al., 1992). In one natural experiment on the poverty-pathology link, researchers tracked
rates of behavior problems in North Carolina Native American children as economic development enabled a dramatic reduction in their community’s poverty rate. As the study began, children of poverty exhibited more deviant and aggressive behaviors. After four years,
children whose families had moved above the poverty line exhibited a 40 percent decrease
in the behavior problems, while those who continued in their previous positions below or
above the poverty line exhibited no change (Costello et al., 2003).
As TABLE 65.2 indicates, there is a wide range of risk and protective factors for mental
disorders. At what times of life do disorders strike? Usually by early adulthood. “Over 75
percent of our sample with any disorder had experienced its first symptoms by age 24,”
reported Lee Robins and Darrel Regier (1991, p. 331). The symptoms of antisocial personality disorder and of phobias are among the earliest to appear, at a median age of 8 and
10, respectively. Symptoms of alcohol use disorder, obsessive-compulsive disorder, bipolar
disorder, and schizophrenia appear at a median age near 20. Major depression often hits
somewhat later, at a median age of 25. Such findings make clear the need for research and
treatment to help the growing number of people, especially teenagers and young adults,
who suffer the bewilderment and pain of a psychological disorder.
ENGAGE
Critical Questions
Some psychologists have called for a
classification of strengths and virtues
that characterize those who are mentally healthy. They call this effort to
create such a classification the
“un-DSM.” Have students consider the
following about the “un-DSM” :
In what ways could researchers
validate that these strengths are
universally held?
Are people born with these
strengths and virtues, or can such
qualities be developed?
Some of these strengths are not
demonstrated unless a crisis occurs
(for example, courage). How can
a person know if he or she has
this strength if a situation has not
occurred that would lead to its
demonstration?
Table 65.2 Risk and Protective Factors for Mental Disorders
Risk Factors
Academic failure
Birth complications
Caring for chronically ill or patients with
neurocognitive disorder
Child abuse and neglect
Chronic insomnia
Chronic pain
Family disorganization or conflict
Low birth weight
Low socioeconomic status
Medical illness
Neurochemical imbalance
Parental mental illness
Parental substance abuse
Personal loss and bereavement
Poor work skills and habits
Reading disabilities
Sensory disabilities
Social incompetence
Stressful life events
Substance abuse
Trauma experiences
Are some of the strengths and
virtues more desirable than others?
Why or why not?
ENGAGE
Critical Questions
Ask your students if psychological well-being simply refers to the
absence of disorder. Are people mentally sound if they do not suffer from
anxiety, depression, or other forms of
psychological symptomology? What
characteristics mark psychological
well-being? Carol D. Ryff is one of
many psychologists who argue that
we must define mental health in
terms of the positive. She identifies
the following 6 core dimensions of
well-being:
Self-acceptance. The person not
only has a positive attitude toward
his or her self but also accepts
multiple aspects of the self—good
and bad, past, present, and future.
Positive relations with other
people. Healthy people have
warm, satisfying, and trusting
interpersonal relationships.
Protective Factors
Aerobic exercise
Community offering empowerment,
opportunity, and security
Economic independence
Effective parenting
Feelings of mastery and control
Feelings of security
Literacy
Positive attachment and early bonding
Positive parent-child relationships
Problem-solving skills
Resilient coping with stress and adversity
Self-esteem
Social and work skills
Social support from family and friends
Source: World Health Organization (WHO, 2004b,c).
MyersAP_SE_2e_Mod65_B.indd
Environmental658mastery. The
healthy are
able to choose or create contexts that are
supportive of personal needs or values.
Purpose in life. The person has both goals
and a sense of directedness.
Personal growth. Healthy people see
themselves as growing and expanding.
1/21/14 9:35 AM
Ryff, C. D. (1995). Psychological well-being in adult
life. Current Directions in Psychological Science, 4,
99–103.
Autonomy. The person is
independent, self-determining,
and self-controlled.
658
Unit XII
MyersPsyAP_TE_2e_U12.indd 658
Abnormal Behavior
3/5/14 11:02 AM
MyersAP_SE_2e
Introduction to Psychological Disorders
Module 65
659
Although mindful of the pain, we can also be encouraged by the many successful people—including Leonardo da Vinci, Isaac Newton, and Leo Tolstoy—who pursued brilliant
careers while enduring psychological difficulties. So have 18 U.S. presidents, including the
periodically depressed Abraham Lincoln, according to one psychiatric analysis of their biographies (Davidson et al., 2006). The bewilderment, fear, and sorrow caused by psychological
disorders are real. But, as Unit XIII shows, hope, too, is real.
Before You Move On
䉴 ASK YOURSELF
How would you draw the line between sending disturbed criminals to prisons or to mental
hospitals? Would the person’s history (for example, having suffered child abuse) influence
your decisions?
CLOSE & ASSESS
Exit Assessment
䉴 TEST YOURSELF
What is the biopsychosocial approach, and why is it important in our understanding of
psychological disorders?
Answers to the Test Yourself questions can be found in Appendix E at the end of the book.
Module 65 Review
65-1
•
According to psychologists and psychiatrists, a psychological
disorder is a syndrome marked by a clinically significant
disturbance in an individual’s cognition, emotion regulation, or behavior.
65-2
•
•
1/21/14 9:35 AM
•
Why is there some controversy over
attention-deficit/hyperactivity disorder?
A child who by age 7 displays extreme inattention,
hyperactivity, and impulsivity may be diagnosed with
attention-deficit/hyperactivity disorder (ADHD) and treated
with medication and other therapy.
How do the medical model and the
biopsychosocial approach understand
psychological disorders?
The biopsychosocial approach assumes that three sets of
influences—biological (evolution, genetics, brain structure
and chemistry), psychological (stress, trauma, learned
helplessness, mood-related perceptions and memories),
and social-cultural (roles, expectations, definitions of
“normality” and “disorder”)—interact to produce specific
psychological disorders.
65-4
How and why do clinicians classify
psychological disorders?
•
The American Psychiatric Association’s DSM-5 (Diagnostic
and Statistical Manual of Mental Disorders, Fifth Edition)
contains diagnostic labels and descriptions that
provide a common language and shared concepts for
communication and research.
•
Some critics believe the DSM editions have become too
detailed and extensive.
The controversy centers on whether the growing number
of ADHD cases reflects overdiagnosis or increased
awareness of the disorder. Long-term effects of stimulantdrug treatment for ADHD are not yet known.
65-3
•
How should we draw the line between
normality and disorder?
Have students compare and contrast
the medical model and the biopsychosocial model of understanding
mental illness. Key factors to look for
include the medical model focusing
on symptoms and treatment and the
biopsychosocial model exploring
physical, psychological, and socialcultural factors that influence the
explanations of mental illness.
The medical model assumes that psychological disorders
are mental illnesses with physical causes that can be
diagnosed, treated, and, in most cases, cured through
therapy, sometimes in a hospital.
MyersAP_SE_2e_Mod65_B.indd 659
1/21/14 9:35 AM
Introduction to Psychological Disorders
MyersPsyAP_TE_2e_U12.indd 659
Module 65
659
3/5/14 11:03 AM
Unit XII Abnormal Behavior
660
65-5
•
•
Answers to Multiple-Choice
Questions
1. e
2. a
Why do some psychologists criticize the
use of diagnostic labels?
Other critics view DSM diagnoses as arbitrary labels
that create preconceptions which bias perceptions of
the labeled person’s past and present behavior. The
legal label, “insanity,” raises moral and ethical questions
about whether society should hold people with disorders
responsible for their violent actions.
Most people with disorders are nonviolent and are more
likely to be victims than attackers.
How many people suffer, or have suffered,
from a psychological disorder? Is poverty a
risk factor?
65-6
•
Psychological disorder rates vary, depending on the time
and place of the survey. In one multinational survey,
rates for any disorder ranged from less than 5 percent
(Shanghai) to more than 25 percent (the United States).
•
Poverty is a risk factor: Conditions and experiences
associated with poverty contribute to the development
of psychological disorders. But some disorders, such as
schizophrenia, can drive people into poverty.
Multiple-Choice Questions
1. Which of the following describes the idea that
3. b
3. Which of the following disorders do Americans report
psychological disorders can be diagnosed and treated?
most frequently?
a.
b.
c.
d.
e.
a.
b.
c.
d.
e.
Taijin-kyofusho
The DSM
The biopsychosocial approach
Amok
The medical model
Schizophrenia
Mood disorders
Posttraumatic stress disorder (PTSD)
Obsessive-compulsive disorder (OCD)
Attention-deficit/hyperactivity disorder (ADHD)
2. Which of the following is the primary purpose of the
DSM?
a. Diagnosis of mental disorders
b. Selection of appropriate psychological therapies for
mental disorders
c. Placement of mental disorders in appropriate cultural
context
d. Selection of appropriate medicines to treat mental
disorders
e. Understanding the causes of mental disorders
Answer to Practice FRQ 2
Answers may vary, but the 2 criticisms
emphasized in the text appear below.
1 point: The system is too broad and
identifies too many widespread behaviors as possibly abnormal.
1 point: The labels used are arbitrary
and not based on science.
Practice FRQs
1. Name and describe the two major approaches to
understanding psychological disorders.
Answer
2 points: The medical model, which is an attempt to first
diagnose and then treat psychological disorders.
2 points: The biopsychosocial approach, which is an attempt
to understand psychological disorders as an interaction of
biological, psychological, and social-cultural factors.
MyersAP_SE_2e_Mod65_B.indd 660
660
Unit XII
MyersPsyAP_TE_2e_U12.indd 660
2. Explain two criticisms of the DSM.
(2 points)
1/21/14 9:35 AM
Abnormal Behavior
3/5/14 11:03 AM
MyersAP_SE_2e
Anxiety Disorders, Obsessive-Compulsive Disorder, and Posttraumatic Stress Disorder
Module 66
661
TEACH
Module 66
es
ty Imag
via Get
ferns
ge/Red
w Ben
TEACH
TR M
TRM
Identify the different anxiety disorders.
66-2
Describe obsessive-compulsive disorder.
66-3
Describe posttraumatic stress disorder.
66-4
Describe how the learning and biological perspectives explain anxiety
disorders, OCD, and PTSD.
66-1
What are the different anxiety disorders?
Anxiety is part of life. Speaking in front of a class, peering down from a ladder, or waiting to
play in a big game, any one of us might feel anxious (even seasoned performers like Green
Day’s Billie Joe Armstrong, whose anxiety and substance abuse resulted in cancelled concerts in 2012 and 2013). At times we may feel enough anxiety to avoid making eye contact
or talking with someone—“shyness,” we call it. Fortunately for most of us, our uneasiness
is not intense and persistent.
Some of us, however, are more prone to notice and remember threats (Mitte,
2008). This tendency may place us at risk for one of the anxiety disorders, marked
by distressing, persistent anxiety or dysfunctional anxiety-reducing behaviors. We
will consider these three:
•
Generalized anxiety disorder, in which a person is unexplainably and continually
tense and uneasy
•
Panic disorder, in which a person experiences sudden episodes of intense dread
•
Phobias, in which a person is intensely and irrationally afraid of a specific
object or situation
Teaching Tip
Have students provide a list of
anxiety-producing situations they
have encountered in their lives.
Encourage them to choose the situation that produces the most anxiety.
Ask how they cope with that type of
situation and what they might do to
overcome the associated anxiety.
Use Student Activity: Taylor Manifest Anxiety Scale from the TRM to
help students assess their own levels
of anxiety.
Module Learning Objectives
66-1
Discussion Starter
Use the Module 66 Discussion Starter:
Fact or Falsehood? activity from the
TRM to introduce the concepts from
this module.
Andre
Anxiety Disorders, Obsessive-Compulsive
Disorder, and Posttraumatic Stress
Disorder
TR M
TRM
anxiety disorders psychological
disorders characterized by distressing,
persistent anxiety or maladaptive
behaviors that reduce anxiety.
1/21/14 9:35 AM
•
Obsessive-compulsive disorder, in which a person is troubled by repetitive
thoughts or actions
•
Posttraumatic stress disorder, in which a person has lingering memories, nightmares,
and other symptoms for weeks after a severely threatening, uncontrollable event
MyersAP_SE_2e_Mod66_B.indd 661
© Jason Love
Two other disorders involve anxiety, though the DSM-5 now classifies them separately:
Obsessing about obsessive-compulsive
disorder
1/21/14 9:35 AM
Anxiety Disorders, Obsessive-Compulsive Disorder, and Posttraumatic Stress Disorder
MyersPsyAP_TE_2e_U12.indd 661
Module 66
661
3/5/14 11:03 AM
662
Unit XII Abnormal Behavior
A P ® E x a m Ti p
ENGAGE
The way disorders are classified
can be confusing, so it’s worth
taking some time to keep the
organization straight. Sometimes,
there is a broad classification
that includes more specific
disorders—the broad category of
anxiety disorders, for example,
includes generalized anxiety
disorder, panic disorder, and
phobia. Other times, there is
just one level of classification.
Obsessive-compulsive disorder
and posttraumatic stress disorder
do not fit into broader categories.
Enrichment
Patients who experience generalized
anxiety disorder (GAD) often also have
major depression, an illness discussed
later in this unit. Effexor®, a drug
manufactured by Wyeth-Ayerst, has
been shown to be effective in treating both GAD and major depression.
Paxil®, an antidepressant drug in the
same class as Prozac® and Zoloft®, is
a serotonin reuptake inhibitor and is
also approved for use to treat GAD
as well as social phobia and panic
disorder.
For the past two years, Tom, a 27-year-old electrician, has been bothered by dizziness,
sweating palms, heart palpitations, and ringing in his ears. He feels edgy and sometimes
finds himself shaking. With reasonable success, he hides his symptoms from his family and
co-workers. But he allows himself few other social contacts, and occasionally he has to leave
work. His family doctor and a neurologist can find no physical problem.
Tom’s unfocused, out-of-control, agitated feelings suggest a generalized anxiety disorder, which is marked by pathological worry. The symptoms of this disorder are commonplace; their persistence, for six months or more, is not. People with this condition—
two-thirds are women (McLean & Anderson, 2009)—worry continually, and they are often
jittery, agitated, and sleep-deprived. Concentration is difficult as attention switches from
worry to worry, and their tension and apprehension may leak out through furrowed brows,
twitching eyelids, trembling, perspiration, or fidgeting.
One of generalized anxiety disorder’s worst characteristics is that the person may not
be able to identify, and therefore deal with or avoid, its cause. To use Sigmund Freud’s term,
the anxiety is free-floating. Generalized anxiety disorder is often accompanied by depressed
mood, but even without depression it tends to be disabling (Hunt et al., 2004; Moffitt et al.,
2007b). Moreover, it may lead to physical problems, such as high blood pressure.
Many people with generalized anxiety disorder were maltreated and inhibited as children (Moffitt et al., 2007a). As time passes, however, emotions tend to mellow, and by age
50, generalized anxiety disorder becomes fairly rare (Rubio & López-Ibor, 2007).
Panic Disorder
TEACH
Flip It
Students can get additional help
understanding panic disorder by
watching the Flip It Video: Classifying
Panic.
generalized anxiety disorder
an anxiety disorder in which
a person is continually tense,
apprehensive, and in a state of
autonomic nervous system arousal.
TEACH
TR M
TRM
Generalized Anxiety Disorder
Teaching Tip
Agoraphobia, or the fear of being in
open spaces or in public, often accompanies panic disorder. Since people
experience panic attacks at uncontrollable times and in uncontrollable situations, they will often develop a fear
of being in public and having a panic
attack. This fear leaves them suffering
in their homes, afraid to even go out.
Use Student Activity: Social Phobias from the TRM to help students
assess their own levels of social
phobia.
panic disorder an anxiety
disorder marked by unpredictable,
minutes-long episodes of intense
dread in which a person experiences
terror and accompanying chest
pain, choking, or other frightening
sensations. Often followed by worry
over a possible next attack.
phobia an anxiety disorder marked
by a persistent, irrational fear and
avoidance of a specific object,
activity, or situation.
social anxiety disorder intense
fear of social situations, leading to
avoidance of such. (Formerly called
social phobia.)
Panic disorder entails an anxiety tornado. Panic strikes suddenly, wreaks havoc, and disappears. For the 1 person in 75 with this disorder, anxiety suddenly escalates into a terrifying panic attack—a minutes-long episode of intense fear that something horrible is about to
happen. Heart palpitations, shortness of breath, choking sensations, trembling, or dizziness
typically accompany the panic, which may be misperceived as a heart attack or other serious physical ailment. Smokers have at least a doubled risk of panic disorder (Zvolensky &
Bernstein, 2005). Because nicotine is a stimulant, lighting up doesn’t lighten up.
One woman recalled suddenly feeling “hot and as though I couldn’t breathe. My heart
was racing and I started to sweat and tremble and I was sure I was going to faint. Then my
fingers started to feel numb and tingly and things seemed unreal. It was so bad I wondered
if I was dying and asked my husband to take me to the emergency room. By the time we got
there (about 10 minutes) the worst of the attack was over and I just felt washed out” (Greist
et al., 1986).
Phobias
Phobias are anxiety disorders in which an irrational fear causes the person to avoid some
object, activity, or situation. Many people accept their phobias and live with them, but others
are incapacitated by their efforts to avoid the feared situation. Marilyn, an otherwise healthy
and happy 28-year-old, fears thunderstorms so intensely that she feels anxious as soon as a
weather forecaster mentions possible storms later in the week. If her husband is away and a
storm is forecast, she may stay with a close relative. During a storm, she hides from windows
and buries her head to avoid seeing the lightning.
Other specific phobias may focus on animals, insects, heights, blood, or enclosed spaces
(FIGURE 66.1). People avoid the stimulus that arouses the fear, hiding during thunderstorms or avoiding high places.
Not all phobias have such specific triggers. Social anxiety disorder (formerly
called social phobia) is shyness taken to an extreme. Those with social anxiety disorder,
MyersAP_SE_2e_Mod66_B.indd 662
1/21/14 9:35 AM
ENGAGE
TR M
TRM
Critical Questions
Phobias are one of the most successfully
treated disorders around, yet few people with
phobias seek treatment for them. Have students contemplate the reasons for this:
662
Unit XII
MyersPsyAP_TE_2e_U12.indd 662
What types of fears are more crippling than
others? Why?
Use Student Activity: Fear Survey from the TRM
to help students assess their own fears.
Why would people be reluctant to get
treatment for their everyday fears?
(Everyday fears might not be crippling or
disruptive. Also, people may successfully
avoid their fear object, not seeing the need to
rid themselves of the underlying fear.)
Abnormal Behavior
3/5/14 11:03 AM
MyersAP_SE_2e
Anxiety Disorders, Obsessive-Compulsive Disorder, and Posttraumatic Stress Disorder
25%
Percentage
of people
surveyed
Module 66
663
TEACH
Figure 66.1
Some common and uncommon
specific fears This Dutch national
20
Diversity Connections
interview study identified the
commonality of various specific fears.
A strong fear becomes a phobia if it
provokes a compelling but irrational
desire to avoid the dreaded object or
situation. (From Delpa et al., 2008.)
15
10
5
Have students explore whether the
things feared most by people in the
United States are similar to those
things feared most in other countries.
0
Being
alone
Flying
Fear
Storms
Water
Blood
Enclosed
spaces
Animals
Height
Phobia
an intense fear of being scrutinized by others, avoid potentially embarrassing social situations, such as speaking up,
eating out, or going to parties—or will sweat or tremble
when doing so.
Much as fretting over insomnia may, ironically, cause insomnia, so worries about anxiety—perhaps fearing another
Martin Harvey/
Jupiterimages
panic attack, or fearing anxiety-caused sweating in public—
can amplify anxiety symptoms (Olatunji & Wolitzky-Taylor, 2009). People who have experienced several panic attacks may come to avoid situations where the panic has struck before.
If the fear is intense enough, it may become agoraphobia, fear or avoidance of situations
in which escape might be difficult or help unavailable when panic strikes. Given such fear,
people may avoid being outside the home, in a crowd, on a bus, or on an elevator.
After spending five years sailing the world, Charles Darwin began suffering panic disorder at age 28. Because of the attacks, he moved to the country, avoided social gatherings,
and traveled only in his wife’s company. But the relative seclusion did free him to focus on
developing his evolutionary theory. “Even ill health,” he reflected, “has saved me from the
distraction of society and its amusements” (quoted in Ma, 1997).
agoraphobia fear or avoidance of
situations, such as crowds or wide
open places, where one has felt loss
of control and panic.
1/21/14 9:35 AM
TEACH
MyersAP_SE_2e_Mod66_B.indd 663
Soccer star David Beckham has
openly discussed his obsessivecompulsive tendencies, which have
driven him to line up objects in pairs or
to spend hours straightening furniture
(Adams, 2011).
Common Pitfalls
OCD is an anxiety disorder characterized by
obsessive thoughts with corresponding
compulsions. People with OCD are likely
to lead lives where strict routines become
essential, repeating tasks over and over
again to find relief from their anxieties.
Are the incidences of certain
anxiety disorders higher among
people in countries that have
experienced war? Which kinds of
disorders are most prevalent?
Ask your students to explore the Internet to see how many different types of
phobias they can find.
1/21/14 9:35 AM
Help students differentiate between obsessivecompulsive disorder and obsessive-compulsive
personality disorder:
Do fears differ among people of
different socioeconomic groups?
Why or why not?
Online Activities
Stephen Dunn/Getty Images
As with generalized anxiety and phobias, we can see aspects of obsessive-compulsive
disorder (OCD) in our everyday behavior. We all may at times be obsessed with senseless
or offensive thoughts that will not go away. Or we may engage
in compulsive behaviors, perhaps lining up books and pencils
“just so” before studying.
Obsessive thoughts and compulsive behaviors cross the fine
line between normality and disorder when they persistently interfere with everyday living and cause distress. Checking to see
you locked the door is normal; checking 10 times is not. Washing
your hands is normal; washing so often that your skin becomes
raw is not. (TABLE 66.1 on the next page offers more examples.)
At some time during their lives, often during their late teens or
twenties, 2 to 3 percent of people cross that line from normal
preoccupations and fussiness to debilitating disorder (Karno
et al., 1988). Although the person knows them to be irrational,
the anxiety-fueled obsessive thoughts become so haunting, the
compulsive rituals so senselessly time-consuming, that effective
functioning becomes impossible.
ENGAGE
Making everything perfect
What is obsessive-compulsive disorder?
Do people in other cultures fear
public speaking as much as
Americans seem to?
obsessive-compulsive disorder
(OCD) a disorder characterized
by unwanted repetitive thoughts
(obsessions) and/or actions
(compulsions).
Obsessive-Compulsive Disorder
66-2
What is the most obscure phobia?
What is the most common phobia?
ENGAGE
Enrichment
Former Nickelodeon game show host
Marc Summers suffers from obsessivecompulsive disorder. He hosted the
classic shows Double Dare and Family
Double Dare, game shows that daily
subjected him and contestants to
getting splattered with slime and goo.
He was often consumed by anxiety as
he struggled to put on a cheerful face
as the shows’ host. Currently, he hosts
several shows and has worked with
with Freedom from Fear, a national
anxiety and depression resource
organization.
OCPD is a personality disorder characterized
by an obsessive need for neatness, order,
and symmetry. People with OCPD are
more likely to be called “neat freaks”
or “anal-retentive.”
Anxiety Disorders, Obsessive-Compulsive Disorder, and Posttraumatic Stress Disorder
MyersPsyAP_TE_2e_U12.indd 663
Module 66
663
3/5/14 11:03 AM
664
Unit XII Abnormal Behavior
TEACH
TR M
TRM
Table 66.1 Common Obsessions and Compulsions Among
Children and Adolescents With Obsessive-Compulsive Disorder
Teaching Tip
Hoarders collect things and have a
hard time throwing anything away
because they are plagued by fears
of losing something or needing
something that they’ve discarded.
So, they keep everything, including
leftover food, years-old paperwork,
and broken items.
posttraumatic stress disorder
(PTSD) a disorder characterized
by haunting memories, nightmares,
social withdrawal, jumpy anxiety,
numbness of feeling, and/or
insomnia that lingers for four weeks
or more after a traumatic experience.
Checkers will recheck actions they
have performed over and over
again. For example, they might
fear they did not turn off the oven
after preparing a meal, so they will
recheck it many times before their
anxiety lessens. The number of
checks needed to relieve anxiety
increases with each experience.
Counters will count everything,
from steps they take to words
people say to them. They can
become so preoccupied with
counting something seemingly
unimportant that it disrupts their
daily lives.
Cleaners will clean excessively.
Often, this cleaning will need to be
done a certain number of times in
order to relieve anxiety.
Obsessions (repetitive thoughts)
Concern with dirt, germs, or toxins
Something terrible happening (fire, death, illness)
Symmetry, order, or exactness
40
24
17
Compulsions (repetitive behaviors)
Excessive hand washing, bathing, toothbrushing, or grooming
Repeating rituals (in/out of a door, up/down from a chair)
Checking doors, locks, appliances, car brakes, homework
85
51
46
Source: Adapted from Rapoport, 1989.
OCD is more common among teens and young adults than among older people (Samuels & Nestadt, 1997). A 40-year follow-up study of 144 Swedish people diagnosed with
the disorder found that, for most, the obsessions and compulsions had gradually lessened,
though only 1 in 5 had completely recovered (Skoog & Skoog, 1999).
Posttraumatic Stress Disorder
66-3
Bringing the war home Nearly
a quarter of a million Iraq and
Afghanistan war veterans have been
diagnosed with PTSD or traumatic
brain injury (TBI). Many vets participate
in an intensive recovery program using
deep breathing, massage, and group
and individual discussion techniques to
treat their PTSD or TBI.
Lynn Johnson/National Geographic Society/Corbis
Percentage
Reporting Symptom
Thought or Behavior
Compulsions manifest themselves in
several different ways:
Use Student Activity: ObsessiveCompulsive Disorder from the TRM to
help students assess their tendencies
toward this disorder.
What is posttraumatic stress disorder?
As an Iraq war soldier, Jesse “saw the murder of children, women. It was just horrible for
anyone to experience.” After calling in a helicopter strike on one house where he had seen
ammunition crates carried in, he heard the screams of children from within. “I didn’t know
there were kids there,” he recalls. Back home in Texas, he suffered “real bad flashbacks”
(Welch, 2005).
Our memories exist in part to protect us in the future. So there is biological wisdom in not being able to forget our most emotional or traumatic experiences—our greatest embarrassments, our worst accidents, our most horrid experiences. But sometimes,
for some of us, the unforgettable takes over our lives. The complaints of battle-scarred
veterans such as Jesse—recurring haunting memories and
nightmares, a numbed social withdrawal, jumpy anxiety, insomnia—are typical of what once was called “shellshock”
or “battle fatigue” and now is called posttraumatic stress
disorder (PTSD) (Babson & Feldner, 2010; Yufik & Simms,
2010). What defines and explains PTSD is less the event itself than the severity and persistence of the trauma memory
(Rubin et al., 2008).
PTSD symptoms have also been reported by survivors of
accidents, disasters, and violent and sexual assaults (including an estimated two-thirds of prostitutes) (Brewin et al., 1999;
Farley et al., 1998; Taylor et al., 1998). A month after the 9/11
terrorist attacks, a survey of Manhattan residents indicated that
8.5 percent were suffering PTSD, most as a result of the attack
(Galea et al., 2002). Among those living near the World Trade
Center, 20 percent reported such telltale signs as nightmares,
severe anxiety, and fear of public places (Susser et al., 2002).
MyersAP_SE_2e_Mod66_B.indd 664
1/21/14 9:35 AM
ENGAGE
Critical Questions
Today, 24-hour news channels air up-to-theminute coverage of war and disaster. People
can watch violence as it happens. While this
represents an advance in TV journalism, it
exposes people to everyday experiences that
could lead to posttraumatic stress disorder
(PTSD).
664
Unit XII
MyersPsyAP_TE_2e_U12.indd 664
How does viewing trauma on TV affect
people who haven’t experienced the
tragedy or conflict firsthand?
Can people develop PTSD vicariously? How
different are the experiences of those who
develop PTSD vicariously compared with
people who develop it through firsthand
experience?
How did coverage of the September 11,
2001, terrorist attacks and Hurricane Katrina
affect the viewing public?
Were lessons learned from coverage of the
first Gulf War applied to coverage of the Iraq
War? Why or why not?
Abnormal Behavior
3/5/14 11:03 AM
MyersAP_SE_2e
Anxiety Disorders, Obsessive-Compulsive Disorder, and Posttraumatic Stress Disorder
To pin down the frequency of this disorder, the U.S. Centers for Disease Control (1988)
compared 7000 Vietnam combat veterans with 7000 noncombat veterans who served during
the same years. On average, according to a reanalysis, 19 percent of all Vietnam veterans reported PTSD symptoms. The rate varied from 10 percent among those who had never seen
combat to 32 percent among those who had experienced heavy combat (Dohrenwend et al.,
2006). Similar variations in rates have been found among more recent combat veterans and
among people who have experienced a natural disaster or have been kidnapped, held captive,
tortured, or raped (Brewin et al., 2000; Brody, 2000; Kessler, 2000; Stone, 2005; Yaffe et al., 2010).
The toll seems at least as high for veterans of the Iraq war, where 1 in 6 U.S. combat
infantry personnel has reported symptoms of PTSD, depression, or severe anxiety in the
months after returning home (Hoge et al., 2006, 2007). In one study of 103,788 veterans
returning from Iraq and Afghanistan, 1 in 4 was diagnosed with a psychological disorder,
most frequently PTSD (Seal et al., 2007).
So what determines whether a person suffers PTSD after a traumatic event? Research
indicates that the greater one’s emotional distress during a trauma, the higher the risk for
posttraumatic symptoms (Ozer et al., 2003). Among New Yorkers who witnessed the 9/11
attacks, PTSD was doubled for survivors who were inside rather than outside the World
Trade Center (Bonanno et al., 2006). And the more frequent an assault experience, the more
adverse the long-term outcomes tend to be (Golding, 1999). In the 30 years after the Vietnam war, veterans who came home with a PTSD diagnosis had twice the normal likelihood
of dying (Crawford et al., 2009).
A sensitive limbic system seems to increase vulnerability, by flooding the body with
stress hormones again and again as images of the traumatic experience erupt into consciousness (Kosslyn, 2005; Ozer & Weiss, 2004). Brain scans of PTSD patients suffering memory
flashbacks reveal an aberrant and persistent right temporal lobe activation (Engdahl et al.,
2010). Genes may also play a role. In one study, combat-exposed men had identical twins
who did not experience combat. But these nonexposed co-twins still tended to share their
brother’s risk for cognitive difficulties, such as unfocused attention. Such findings suggest
that some PTSD symptoms may actually be genetically predisposed (Gilbertson et al., 2006).
Some psychologists believe that PTSD has been overdiagnosed, due partly to a broadening definition of trauma (Dobbs, 2009; McNally, 2003). PTSD is actually infrequent, say those
critics, and well-intentioned attempts to have people relive the trauma may exacerbate their
emotions and pathologize normal stress reactions (Wakefield & Spitzer, 2002). “Debriefing”
survivors right after a trauma by getting them to revisit the experience and vent emotions has
actually proven generally ineffective and sometimes harmful (Bonanno et al., 2010).
Researchers have noted the impressive survivor resiliency of those who do not develop
PTSD (Bonanno et al., 2010). About half of adults experience at least one traumatic event in
their lifetime, but only about 1 in 10 women and 1 in 20 men develop PTSD (Olff et al., 2007;
Ozer & Weiss, 2004; Tolin & Foa, 2006). More than 9 in 10 New Yorkers, although stunned
and grief-stricken by 9/11, did not respond pathologically. By the following January, the stress
symptoms of the rest had mostly subsided (Galea et al., 2002). Similarly, most combat-stressed
veterans and most political dissidents who survive dozens of episodes of torture do not later exhibit PTSD (Mineka & Zinbarg, 1996). Likewise, the Holocaust survivors in 71 studies
“showed remarkable resilience.” Despite some lingering stress symptoms, most experienced
essentially normal physical health and cognitive functioning (Barel et al., 2010).
Psychologist Peter Suedfeld (1998, 2000; Cassel & Suedfeld, 2006), who as a boy survived the Holocaust under deprived conditions while his mother died in Auschwitz, has
documented the resilience of Holocaust survivors, most of whom have lived productive lives.
“It is not always true that ‘What doesn’t kill you makes you stronger,’ but it is often true,”
he reports. And “what doesn’t kill you may reveal to you just how strong you really are.”
Indeed, suffering can lead to “benefit finding” (Aspinwall & Tedeschi, 2010a,b; Helgeson
et al., 2006), and to what Richard Tedeschi and Lawrence Calhoun (2004) call posttraumatic
growth. Tedeschi and Calhoun have found that the struggle with challenging crises, such as
1/21/14 9:35 AM
MyersAP_SE_2e_Mod66_B.indd 665
Module 66
665
TEACH
Concept Connections
Link PTSD to flashbulb memories.
Have students recall a powerful
memory of a traumatic event that
they’ve experienced.
Do they remember vividly where
they were when learning of the
death of someone famous?
Do their parents remember vividly
the 9/11 attacks, the Challenger
explosion, the attempted
assassination of President Ronald
Reagan, or the John Lennon
murder?
ENGAGE
Active Learning
Have students interview people who
have experienced trauma firsthand to
see how they coped with the stress
associated with a particularly tragic
event. Some people they may consider interviewing:
FYI
A $125 million, five-year U.S. Army
program is currently assessing
the well-being of 800,000 soldiers
and training them in emotional
resilience (Stix, 2011).
posttraumatic growth positive
psychological changes as a result of
struggling with extremely challenging
circumstances and life crises.
Survivors of the Holocaust or World
War II veterans
Soldiers who served in the Korean,
Vietnam, or Iraq wars
People who remember the
assassinations of the 1960s (for
example, the 2 Kennedy brothers,
Martin Luther King, Jr.)
Survivors of or witnesses to the
September 11, 2001, terrorist
attacks
Evacuees of Hurricane Katrina
1/21/14 9:35 AM
Anxiety Disorders, Obsessive-Compulsive Disorder, and Posttraumatic Stress Disorder
MyersPsyAP_TE_2e_U12.indd 665
Module 66
665
3/5/14 11:03 AM
666
Unit XII Abnormal Behavior
facing cancer, often leads people later to report an increased appreciation for life, more meaningful relationships, increased personal strength, changed priorities, and a richer spiritual life. This
idea—that suffering has transformative power—is also found in Judaism, Christianity, Hinduism, Buddhism, and Islam. The idea is confirmed by research with ordinary people. Compared
with those with traumatic life histories and with those unchallenged by any significant adversity,
people whose life history includes some adversity tend to enjoy better mental health and wellbeing (Seery et al., 2010). Out of even our worst experiences some good can come. Like the body,
the mind has great recuperative powers and may grow stronger with exertion.
Understanding Anxiety Disorders, OCD, and PTSD
66-4
How do the learning and biological perspectives explain anxiety
disorders, OCD, and PTSD?
Anxiety is both a feeling and a cognition, a doubt-laden appraisal of one’s safety or social
skill. How do these anxious feelings and cognitions arise? Freud’s psychoanalytic theory proposed that, beginning in childhood, people repress intolerable impulses, ideas, and feelings
and that this submerged mental energy sometimes produces mystifying symptoms, such
as anxiety. Today’s psychologists have instead turned to two contemporary perspectives—
learning and biological.
The Learning Perspective
TEACH
Concept Connections
Review the components of classical
conditioning that led to Little Albert
developing a fear of white, furry
objects (Unit VI):
Unconditioned stimulus (US)—
loud “bang” that occurred when
lead pipes were struck
Unconditioned response (UR)—
fear of the loud “bang”
Conditioned stimulus (CS)—white,
furry objects
Conditioned response (CR)—fear,
but this time in response to the
white, furry object
Have students recall that the fear
response John Watson cultivated so
easily was reversed in other patients
by his student Mary Cover Jones. She
used classical conditioning to remove
a fear response from individuals who
had developed a fear of a certain
object. She used a pleasant US, like
candy or food, so patients could
associate the pleasant feeling with the
object they had come to fear.
666
Unit XII
MyersPsyAP_TE_2e_U12.indd 666
A P ® E x a m Ti p
This is a good time to return to
Unit VI and review the principles
of classical and operant
conditioning.
CLASSICAL AND OPERANT CONDITIONING
When bad events happen unpredictably and uncontrollably, anxiety or other disorders
often develop (Field, 2006b; Mineka & Oehlberg, 2008). Recall from Unit VI that dogs
learn to fear neutral stimuli associated with shock and that infants come to fear furry
objects associated with frightening noises. Using classical conditioning, researchers have
also created chronically anxious, ulcer-prone rats by giving them unpredictable electric
shocks (Schwartz, 1984). Like assault victims who report feeling anxious when returning
to the scene of the crime, the rats become apprehensive in their lab environment. This
link between conditioned fear and general anxiety helps explain why anxious or traumatized people are hyperattentive to possible threats, and how panic-prone people come
to associate anxiety with certain cues (Bar-Haim et al., 2007; Bouton et al., 2001). In one
survey, 58 percent of those with social anxiety disorder experienced their disorder after a
traumatic event (Ost & Hugdahl, 1981).
Through conditioning, the short list of naturally painful and frightening events can
multiply into a long list of human fears. My car was once struck by another whose driver
missed a stop sign. For months afterward, I felt a twinge of unease when any car approached
from a side street. Marilyn’s phobia of thunderstorms may have been similarly conditioned
during a terrifying or painful experience associated with a thunderstorm.
Two specific learning processes can contribute to these disorders. The first, stimulus generalization, occurs, for example, when a person attacked by a fierce dog later develops a
fear of all dogs. The second learning process, reinforcement, helps maintain our phobias and
compulsions after they arise. Avoiding or escaping the feared situation reduces anxiety, thus
reinforcing the phobic behavior. Feeling anxious or fearing a panic attack, a person may go
inside and be reinforced by feeling calmer (Antony et al., 1992). Compulsive behaviors operate similarly. If washing your hands relieves your feelings of anxiety, you may wash your
hands again when those feelings return.
OBSERVATIONAL LEARNING
We may also learn fear through observational learning—by observing others’ fears. Susan
Mineka (1985, 2002) sought to explain why nearly all monkeys reared in the wild fear snakes,
yet lab-reared monkeys do not. Surely, most wild monkeys do not actually suffer snake bites.
MyersAP_SE_2e_Mod66_B.indd 666
1/23/14 11:16 AM
Abnormal Behavior
3/5/14 11:03 AM
MyersAP_SE_2e
Anxiety Disorders, Obsessive-Compulsive Disorder, and Posttraumatic Stress Disorder
Module 66
667
Do they learn their fear through observation? To find out, Mineka experimented with six monkeys reared in the wild (all strongly fearful of snakes) and their lab-reared offspring (virtually
none of which feared snakes). After repeatedly observing their parents or peers refusing to
reach for food in the presence of a snake, the younger monkeys developed a similar strong fear
of snakes. When retested three months later, their learned fear persisted. Humans likewise
learn fears by observing others (Olsson et al., 2007).
COGNITION
Observational learning is not the only cognitive influence on feelings of anxiety. As the next
unit’s discussion of cognitive-behavioral therapy illustrates, our interpretations and irrational beliefs can also cause feelings of anxiety. Whether we interpret the creaky sound in the
old house simply as the wind or as a possible knife-wielding intruder determines whether
we panic. People with anxiety disorder also tend to be hypervigilant. A pounding heart becomes a sign of a heart attack. A lone spider near the bed becomes a likely infestation.
An everyday disagreement with a friend or boss spells possible doom for the relationship.
Anxiety is especially common when people cannot switch off such intrusive thoughts and
perceive a loss of control and sense of helplessness (Franklin & Foa, 2011).
The Biological Perspective
TEACH
There is, however, more to anxiety, OCD, and PTSD than conditioning, observational learning, and cognition. The biological perspective can help us understand why few people develop lasting phobias after suffering traumas, why we learn some fears more readily, and
why some individuals are more vulnerable.
Teaching Tip
Have your students name fears that
have evolved to help further our species. Ask them to ponder how some
common fears might have developed:
NATURAL SELECTION
We humans seem biologically prepared to fear threats faced by our ancestors. Our phobias focus on such specific fears: spiders, snakes, and other animals; enclosed spaces and
heights; storms and darkness. (Those fearless about these occasional threats were less likely
to survive and leave descendants.) Thus, even in Britain, with only one poisonous snake
species, people often fear snakes. And preschool children more speedily detect snakes in a
scene than flowers, caterpillars, or frogs (LoBue & DeLoache, 2008). It is easy to condition
and hard to extinguish fears of such “evolutionarily relevant” stimuli (Coelho & Purkis,
2009; Davey, 1995; Öhman, 2009).
Our modern fears can also have an evolutionary explanation. For example, a fear of
flying may come from our biological predisposition to fear confinement and heights. Moreover, consider what people tend not to learn to fear. World War II air raids produced remarkably few lasting phobias. As the air blitzes continued, the British, Japanese, and German
populations became not more panicked, but rather more indifferent to planes outside their
immediate neighborhoods (Mineka & Zinbarg, 1996). Evolution has not prepared us to fear
bombs dropping from the sky.
Just as our phobias focus on dangers faced by our ancestors, our compulsive acts typically exaggerate behaviors that contributed to our species’ survival. Grooming gone wild becomes hair pulling. Washing up becomes ritual hand washing. Checking territorial boundaries becomes rechecking an already locked door (Rapoport, 1989).
MyersAP_SE_2e_Mod66_B.indd 667
Public speaking
Flying
Germs
Failure
TEACH
Concept Connections
Remind students that studies of identical twins provide insight into which
behaviors are genetically predisposed
and which behaviors are environmentally influenced. Identical twins share
100 percent of their DNA, so comparisons of their behaviors can shed light
on what might be passed on in our
genetic code.
GENES
Some people are more anxious than others. Genes matter. Pair a traumatic event with a
sensitive, high-strung temperament and the result may be a new phobia (Belsky & Pluess,
2009). Some of us have genes that make us like orchids—fragile, yet capable of beauty under favorable circumstances. Others of us are like dandelions—hardy, and able to thrive in
varied circumstances (Ellis & Boyce, 2008).
Among monkeys, fearfulness runs in families. Individual monkeys react more
strongly to stress if their close biological relatives are anxiously reactive (Suomi, 1986).
1/23/14 11:16 AM
1/21/14 9:35 AM
Anxiety Disorders, Obsessive-Compulsive Disorder, and Posttraumatic Stress Disorder
MyersPsyAP_TE_2e_U12.indd 667
Module 66
667
3/5/14 11:03 AM
Unit XII Abnormal Behavior
Reuters/Mike Blake/Landov
668
Fearless The biological perspective
helps us understand why most people
would be too afraid to try U.S. Olympic
snowboarder Shaun White’s tricks.
White is less vulnerable to a fear of
heights than most of us!
THE BRAIN
TEACH
Remind students that the frontal
lobes are responsible for judgment
and decision making. If people with
OCD have overactive frontal lobes,
then they are controlled by overzealous decision making. They cannot
attend to the decisions they should be
making, insofar as their behaviors are
ruled by repetitive and overbearing
thoughts.
ENGAGE
Figure 66.2
An obsessive-compulsive
brain Neuroscientists Nicholas
Critical Questions
Maltby, David Tolin, and their
colleagues (2005) used functional
MRI scans to compare the brains
of those with and without OCD
as they engaged in a challenging
cognitive task. The scans of
those with OCD showed elevated
activity in the anterior cingulate
cortex in the brain’s frontal area
(indicated by the yellow area on
the far right).
What would life be like without fear?
Have students offer scenarios where
fear is detrimental or useful to daily
life. If fear weren’t an issue, how would
those situations be different?
MyersAP_SE_2e_Mod66_B.indd 668
Unit XII
MyersPsyAP_TE_2e_U12.indd 668
Reprinted from NeuroImage, 24, Maltby, N., Tolin, D . F., Worhunsky, P., O’Keefe,
T. M., & Kiehl, K. A. Dysfunctional action monitoring hyperactivates frontal-striatal
circuits in obsessive-compulsive disorder: An event-related fMRI study, 495–503,
2005, with permission from Elsevier.
Generalized anxiety, panic attacks, PTSD, and even obsessions and compulsions are manifested biologically as an overarousal of brain areas involved in impulse control and habitual
behaviors. When the disordered brain detects that something is amiss, it seems to generate a mental hiccup of repeating thoughts or actions (Gehring et al., 2000). Brain scans
of people with OCD reveal elevated activity in specific brain areas during behaviors such
as compulsive hand washing, checking, ordering, or hoarding (Insel, 2010; Mataix-Cols et
al., 2004, 2005). As FIGURE 66.2 shows, the anterior cingulate cortex, a brain region that
monitors our actions and checks for errors, seems especially likely to be hyperactive in those
with OCD (Maltby et al., 2005). Fear-learning experiences that traumatize the brain can
also create fear circuits within the amygdala (Etkin & Wager, 2007; Kolassa & Elbert, 2007;
Maren, 2007). Some antidepressant drugs dampen this fear-circuit activity and its associated
obsessive-compulsive behavior.
Fears can also be blunted by giving people drugs, such as propranolol or D-Cycloserine,
as they recall and then rerecord (“reconsolidate”) a traumatic experience (Kindt et al., 2009;
Norberg, et al., 2008). Although they don’t forget the experience, the associated emotion is
largely erased.
Concept Connections
668
In humans, vulnerability to anxiety disorders rises when an
afflicted relative is an identical twin (Hettema et al., 2001;
Kendler et al., 1992, 1999, 2002a,b). Identical twins also
may develop similar phobias, even when raised separately
(Carey, 1990; Eckert et al., 1981). One pair of 35-year-old
female identical twins independently became so afraid of
water that each would wade in the ocean backward and
only up to the knees.
Given the genetic contribution to anxiety disorders, researchers are now sleuthing the culprit genes. One research
team has identified 17 genes that appear to be expressed with
typical anxiety disorder symptoms (Hovatta et al., 2005). Other teams have found genes associated specifically with OCD
(Dodman et al., 2010; Hu et al., 2006).
Genes influence disorders by regulating neurotransmitters. Some studies point to an
anxiety gene that affects brain levels of serotonin, a neurotransmitter that influences sleep
and mood (Canli, 2008). Other studies implicate genes that regulate the neurotransmitter
glutamate (Lafleur et al., 2006; Welch et al., 2007). With too much glutamate, the brain’s
alarm centers become overactive.
1/21/14 9:35 AM
Abnormal Behavior
3/5/14 11:03 AM
MyersAP_SE_2e
Anxiety Disorders, Obsessive-Compulsive Disorder, and Posttraumatic Stress Disorder
Module 66
669
Before You Move On
CLOSE & ASSESS
䉴 ASK YOURSELF
Exit Assessment
Can you recall a fear that you have learned? What role, if any, was played by fear
conditioning and by observational learning?
Have students write the definition of
each of the disorders in this module.
Be sure they include the major diagnostic criteria for each.
䉴 TEST YOURSELF
How do generalized anxiety disorder, panic disorder, phobias, obsessive-compulsive
disorder, and posttraumatic stress disorder differ?
Answers to the Test Yourself questions can be found in Appendix E at the end of the book.
Module 66 Review
66-1
•
Anxious feelings and behaviors are classified as an anxiety
disorder only when they form a pattern of distressing,
persistent anxiety or maladaptive behaviors that reduce
anxiety.
•
People with generalized anxiety disorder feel persistently
and uncontrollably tense and apprehensive, for no
apparent reason.
•
In the more extreme panic disorder, anxiety escalates into
periodic episodes of intense dread.
•
Those with a phobia may be irrationally afraid of a specific
object, activity, or situation.
•
Two other disorders (obsessive-compulsive disorder and
posttraumatic stress disorder) involve anxiety (though
they are classified separately from the anxiety disorders).
66-2
•
1/21/14 9:35 AM
What are the different anxiety disorders?
What is obsessive-compulsive disorder?
Persistent and repetitive thoughts (obsessions) and
actions (compulsions) characterize obsessive-compulsive
disorder (OCD).
MyersAP_SE_2e_Mod66_B.indd 669
66-3
•
What is posttraumatic stress disorder?
Symptoms of posttraumatic stress disorder (PTSD) include
four or more weeks of haunting memories, nightmares,
social withdrawal, jumpy anxiety, and sleep problems
following some traumatic experience.
66-4
How do the learning and biological
perspectives explain anxiety disorders,
OCD, and PTSD?
•
The learning perspective views anxiety disorders, OCD,
and PTSD as products of fear conditioning, stimulus
generalization, fearful-behavior reinforcement, and
observational learning of others’ fears and cognitions
(interpretations, irrational beliefs, and hypervigilance).
•
The biological perspective considers the role that fears of
life-threatening animals, objects, or situations played in
natural selection and evolution; genetic predispositions for
high levels of emotional reactivity and neurotransmitter
production; and abnormal responses in the brain’s fear
circuits.
1/21/14 9:35 AM
Anxiety Disorders, Obsessive-Compulsive Disorder, and Posttraumatic Stress Disorder
MyersPsyAP_TE_2e_U12.indd 669
Module 66
669
3/5/14 11:03 AM
Unit XII Abnormal Behavior
670
Answers to Multiple-Choice
Questions
1. b
2. d
Multiple-Choice Questions
1. What do we call an anxiety disorder marked by a
persistent, irrational fear and avoidance of a specific
object, activity, or situation?
3. c
a.
b.
c.
d.
e.
Obsessive-compulsive disorder
Phobia
Panic disorder
Generalized anxiety disorder
Posttraumatic stress disorder
3. The key difference between obsessions and compulsions
is that compulsions involve repetitive
a.
b.
c.
d.
e.
thoughts.
experiences.
behaviors.
memories.
concerns.
2. A person troubled by repetitive thoughts or actions is
most likely experiencing which of the following?
a.
b.
c.
d.
e.
Answer to Practice FRQ 2
2 points each (up to 4 points total) for
correctly naming and describing any of
the following 3 disorders:
Generalized anxiety disorder:
A person is unexplainably and
continually tense and uneasy.
Panic disorder: A person
experiences sudden episodes of
intense dread.
Phobias: A person is intensely
and irrationally afraid of a specific
object or situation.
Generalized anxiety disorder
Posttraumatic stress disorder
Panic disorder
Obsessive-compulsive disorder
Fear conditioning
Practice FRQs
1. Name the two contemporary perspectives used by
psychologists to understand anxiety disorders. Then
explain how or what psychologists study within each
perspective.
Answer
1 point: The learning perspective
1 point: Psychologists using the learning perspective
study fear conditioning, observational learning, or cognitive
processes.
1 point: The biological perspective
1 point: Psychologists using the biological perspective study
natural selection, genes, or the brain.
MyersAP_SE_2e_Mod66_B.indd 670
670
Unit XII
MyersPsyAP_TE_2e_U12.indd 670
2. Name and describe two anxiety disorders.
(4 points)
1/21/14 9:35 AM
Abnormal Behavior
3/5/14 11:03 AM
MyersAP_SE_2e
Mood Disorders
Module 67
671
TEACH
Module 67
bis
67-1
Define mood disorders, and contrast major depressive disorder and
bipolar disorder.
67-2
Describe how the biological and social-cognitive perspectives explain
mood disorders.
67-3
Discuss the factors that affect suicide and self-injury, and identify
important warning signs to watch for in suicide-prevention efforts.
67-1
ng Sto
ck/Cor
Module Learning Objectives
ENGAGE
Active Learning
There are several types of mood disorders that are not classified in the DSM.
Have students research the following
2 examples, concentrating on why
they are not included:
Seasonal affective disorder
Postpartum depression
TEACH
mood disorders psychological
disorders characterized by
emotional extremes. See major
depressive disorder, mania, and
bipolar disorder.
TR M
TRM
If you are like most high school students, at some time during this year—more likely the
dark months of winter than the bright days of summer—you will probably experience some
of depression’s symptoms. You may feel deeply discouraged about the future, dissatisfied
with your life, or socially isolated.You may lack the energy to get things done or even to force
yourself out of bed; be unable to concentrate, eat, or sleep normally; or even wonder if you
would be better off dead. Perhaps academic success came easily to you in middle school, and
now you find that disappointing grades jeopardize your goals. Maybe social stresses, such as
feeling you don’t belong or experiencing the end of a romance, have plunged you into despair. And maybe brooding has at times only worsened your self-torment. Likely you think
you are more alone in having such negative feelings than you really are (Jordan et al., 2011).
In one survey of American high school students, 29 percent “felt so sad or hopeless almost
every day for 2 or more weeks in a row that they stopped doing some usual activities” (CDC,
2008). In another national survey, of American collegians, 31 percent agreed when asked if
in the past year they had at some time “felt so depressed that it was difficult to function”
(ACHA, 2009). Misery has more company than most suppose.
Although phobias are more common, depression is the number-one reason people seek
mental health services. At some point during their lifetime, depression plagues 12 percent of
Canadian adults and 17 percent of U.S. adults (Holden, 2010; Patten et al., 2006). Moreover, it is
the leading cause of disability worldwide (WHO, 2002). In any given year, a depressive episode
plagues 5.8 percent of men and 9.5 percent of women, reports the World Health Organization.
MyersAP_SE_2e_Mod67_B.indd 671
“My life had come to a sudden
stop. I was able to breathe, to
eat, to drink, to sleep. I could
not, indeed, help doing so; but
there was no real life in me.” -LEO
TOLSTOY, MY CONFESSION, 1887
1/21/14 9:35 AM
Mood Disorders
MyersPsyAP_TE_2e_U12.indd 671
Teaching Tip
National Depression Screening Day,
created by Harvard psychiatrist Douglas G. Jacobs in 1991, has since been
repeated every year in early October.
Each year, the number of sites staffed
by mental health professionals has
grown. The free screening includes
completion of a self-rating depression
scale; a 20-minute talk on the causes,
symptoms, and treatment of the disorder, during which participants may ask
questions; and a 5-minute private session with a mental health professional.
No diagnosis or treatment is provided,
but based on the scale scores and the
clinician’s probing, participants learn if
they need more evaluation.
Use Student Activity: The Zung
Self-Rating Depression Scale from the
TRM to help students assess their own
levels of depression.
Major Depressive Disorder
1/21/14 9:35 AM
What are mood disorders? How does major depressive disorder differ
from bipolar disorder?
The emotional extremes of mood disorders come in two principal forms: (1) major depressive disorder, with its prolonged hopelessness and lethargy, and (2) bipolar disorder (formerly
called manic-depressive disorder), in which a person alternates between depression and mania, an overexcited, hyperactive state.
Discussion Starter
Use the Module 67 Fact or Falsehood?
activity from the TRM to introduce the
concepts from this module.
Laughi
Mood Disorders
TR M
TRM
Module 67
671
3/5/14 11:03 AM
672
TEACH
Unit XII Abnormal Behavior
“Depression . . . is well adapted
to make a creature guard itself
against any great or sudden evil.”
-CHARLES DARWIN, THE LIFE AND
LETTERS OF CHARLES DARWIN, 1887
Common Pitfalls
Help students see that depression
following a traumatic event (death of
a loved one, a major failure, a serious
physical injury, and so on) is considered normal. Depression without a
known stressor or causal event may
be due to biological or psychological
reasons.
“If someone offered you a pill that
would make you permanently
happy, you would be well advised
to run fast and run far. Emotion
is a compass that tells us what
to do, and a compass that is
perpetually stuck on NORTH is
worthless.” -DANIEL GILBERT, “THE
SCIENCE OF HAPPINESS,” 2006
ENGAGE
Enrichment
Students may be interested in dysthymic disorder, a depressive state
lasting more than 2 years. In children
and adolescents, the depressive state
need only last 1 year. Patients must
also present 2 or more of the following symptoms:
major depressive disorder
a mood disorder in which a person
experiences, in the absence of drugs
or another medical condition, two
or more weeks with five or more
symptoms, at least one of which
must be either (1) depressed mood
or (2) loss of interest or pleasure.
As anxiety is a response to the threat of future loss, depressed mood is often a response
to past and current loss. About one in four people diagnosed with depression is debilitated
by a significant loss, such as a loved one’s death, a ruptured marriage, or a lost job (Wakefield et al., 2007). To feel bad in reaction to profoundly sad events is to be in touch with
reality. In such times, there is an up side to being down. Sadness is like a car’s low-fuel
light—a signal that warns us to stop and take appropriate measures. Recall that, biologically
speaking, life’s purpose is not happiness but survival and reproduction. Coughing, vomiting,
swelling, and pain protect the body from dangerous toxins. Similarly, depression is a sort of
psychic hibernation: It slows us down, defuses aggression, helps us let go of unattainable
goals, and restrains risk taking (Andrews & Thomson, 2009a,b; Wrosch & Miller, 2009). To
grind temporarily to a halt and ruminate, as depressed people do, is to reassess one’s life
when feeling threatened, and to redirect energy in more promising ways (Watkins, 2008).
Even mild sadness can improve people’s recall, make them more discerning, and help them
make complex decisions (Forgas, 2009). There is sense to suffering.
But when does this response become seriously maladaptive? Joy, contentment, sadness,
and despair are different points on a continuum, points at which any of us may be found at any
given moment. The difference between a blue mood after bad news and a mood disorder is like
the difference between gasping for breath after a hard run and being chronically short of breath.
Major depressive disorder occurs when at least five signs of depression last two or
more weeks (TABLE 67.1). To sense what major depression feels like, suggest some clinicians, imagine combining the anguish of grief with the sluggishness of bad jet lag.
Adults diagnosed with persistent depressive disorder (also called dysthymia) experience
a mildly depressed mood more often than not for at least two years (American Psychiatric
Association, 2013). They also display at least two of the following symptoms:
1.
2.
3.
4.
5.
6.
Problems regulating appetite
Problems regulating sleep
Low energy
Low self-esteem
Difficulty concentrating and making decisions
Feelings of hopelessness
Table 67.1 Diagnosing Major Depressive Disorder
poor appetite or overeating
insomnia (lack of sleeping) or
hypersomnia (too much sleep)
low energy or fatigue
• Depressed mood most of the day
low self-esteem
• Markedly diminished interest or pleasure in activities most of the day
poor concentration or difficulty
making decisions
feelings of hopelessness
The DSM-5 classifies major depressive disorder as the presence of at least five of the
following symptoms over a two-week period of time (including depressed mood or loss of
interest or pleasure). The symptoms must cause near-daily distress or impairment and not be
attributable to substance use or another medical or mental illness.
• Significant weight loss or gain when not dieting, or significant decrease or increase in
appetite
• Insomnia or sleeping too much
• Physical agitation or lethargy
• Fatigue or loss of energy
• Feeling worthless, or excessive or inappropriate guilt
• Problems in thinking, concentrating, or making decisions
• Recurrent thoughts of death and suicide
MyersAP_SE_2e_Mod67_B.indd 672
672
Unit XII
MyersPsyAP_TE_2e_U12.indd 672
1/21/14 9:35 AM
Abnormal Behavior
3/5/14 11:03 AM
MyersAP_SE_2e
Mood Disorders
Bipolar Disorder
WireImage/Getty Images
1/21/14 9:35 AM
Humorist
Samuel Clemens (Mark Twain)
The Granger Collection
Writer
Virginia Woolf
Movie Producer
Tim Burton
George C. Beresford/Hulton Getty Pictures Library
Jemal Countess/Getty Images
MyersAP_SE_2e_Mod67_B.indd 673
673
FYI
With or without therapy, episodes of major depression usually end, and people temporarily
or permanently return to their previous behavior patterns. However, some people rebound
to, or sometimes start with, the opposite emotional extreme—the euphoric, hyperactive,
wildly optimistic state of mania. If depression is living in slow motion, mania is fast forward. Alternating between depression and mania (week to week, and not day to day or
moment to moment) signals bipolar disorder.
Adolescent mood swings, from rage to bubbly, can, when prolonged, produce a bipolar
diagnosis. Between 1994 and 2003, U.S. National Center for Health Statistics annual physician
surveys revealed an astonishing 40-fold increase in diagnoses of bipolar disorder in those 19
and under—from an estimated 20,000 to 800,000 (Carey, 2007; Flora & Bobby, 2008; Moreno
et al., 2007). The new popularity of the diagnosis, given in two-thirds of the cases to boys, has
been a boon to companies whose drugs are prescribed to lessen mood swings. The DSM-5
will likely reduce the number of child and adolescent bipolar diagnoses, by classifying as disruptive mood dysregulation disorder some of those with emotional volatility (Miller, 2010).
During the manic phase, people with bipolar disorder are typically overtalkative, overactive, and elated (though easily irritated); have little need for sleep; and show fewer sexual inhibitions. Speech is loud, flighty, and hard to interrupt. They find advice irritating. Yet they need
protection from their own poor judgment, which may lead to reckless spending or unsafe sex.
In milder forms, mania’s energy and free-flowing thinking does fuel creativity. George
Frideric Handel, who may have suffered from a mild form of bipolar disorder, composed
his nearly four-hour-long Messiah (1742) during three weeks of intense, creative energy
(Keynes, 1980). Robert Schumann composed 51 musical works during two years of mania
(1840 and 1849) and none during 1844, when he was severely depressed (Slater & Meyer,
1959). Those who rely on precision and logic, such as architects, designers, and journalists,
suffer bipolar disorder less often than do those who rely on emotional expression and vivid
imagery (Ludwig, 1995). Composers, artists, poets, novelists, and entertainers seem especially prone (Jamison, 1993, 1995; Kaufman & Baer, 2002; Ludwig, 1995).
Actress
Catherine Zeta-Jones
Module 67
ENGAGE
For some people suffering
depressive disorders or bipolar
disorder, symptoms may have
a seasonal pattern. Depression
may regularly return each fall or
winter, and mania (or a reprieve
from depression) may dependably
arrive with spring. For many
others, winter darkness simply
means more blue moods. When
asked “Have you cried today?”
Americans have agreed more
often in the winter.
Percentage who cried
Men
Women
August
4%
7%
December 8%
21%
Enrichment
There are 2 types of bipolar disorder:
Bipolar Disorder I is the classic
diagnosis of this disorder. Patients
experience periods of inflated
mood followed by depressive
episodes occurring in cycles.
Bipolar Disorder II is a milder form
of this disorder in which patients
experience at least one episode of
hypomania (a period of elevated
mood, but without psychosis)
and at least one major depressive
episode.
Source: Time/CNN survey, 1994.
mania a mood disorder marked by
a hyperactive, wildly optimistic state.
bipolar disorder a mood disorder
in which a person alternates
between the hopelessness and
lethargy of depression and the
overexcited state of mania. (Formerly
called manic-depressive disorder.)
ENGAGE
Enrichment
Give students an idea of mania by
asking them to read the following
account:
When I start going into a high, I no
longer feel like an ordinary housewife. Instead, I feel organized and
accomplished, and I begin to feel
I am my most creative self. I can
write poetry easily. I can compose
melodies without effort. I can
paint. . . . I see myself as being able to
accomplish a great deal for the good
of people. . . . I feel able to accomplish a great deal for the good of my
family and others. I feel pleasure, a
sense of euphoria or elation. I want
it to last forever. I don’t seem to need
much sleep. I’ve lost weight and feel
healthy, and I like myself. I’ve just
bought six new dresses, in fact, and
they look quite good on me. . . . I feel
capable of speaking and doing good
in politics. I would like to help people
with problems similar to mine so
they won’t feel hopeless.
Creativity and bipolar
disorder There are many creative
artists, composers, writers, and
musical performers with bipolar
disorder. Madeleine L’Engle wrote in A
Circle of Quiet (1972): “All the people
in history, literature, art, whom I most
admire: Mozart, Shakespeare, Homer,
El Greco, St. John, Chekhov, Gregory
of Nyssa, Dostoevsky, Emily Brontë:
not one of them would qualify for a
mental-health certificate.”
1/21/14 9:35 AM
Fieve, R. R. (1975). Mood swing (p. 17). New
York: Morrow.
Mood Disorders
MyersPsyAP_TE_2e_U12.indd 673
Module 67
673
3/5/14 11:03 AM
TEACH
TR M
TRM
Concept Connections
Unit XII Abnormal Behavior
Jennifer Graylock
674
It is as true of emotions as of everything else: What goes up comes down. Before long,
the elated mood either returns to normal or plunges into a depression. Though bipolar disorder is much less common than major depressive disorder, it is often more dysfunctional,
claiming twice as many lost workdays yearly (Kessler et al., 2006). Among adults, it afflicts
men and women about equally.
Remind students how healthy people
think and behave. (See, for example,
the discussion of the self-serving bias
in Unit X.) This will help them understand the types of behaviors and
patterns of thinking that characterize
depression.
Use Student Activity: Depression
and Memory from the TRM to help
students explore the role of memory
in depression.
Understanding Mood Disorders
67-2
In thousands of studies, psychologists have been accumulating evidence to help explain
mood disorders and suggest more effective ways to treat and prevent them. Researcher Peter Lewinsohn and his colleagues (1985, 1998, 2003) summarized the facts that any theory
of depression must explain, including the following:
•
Many behavioral and cognitive changes accompany depression. People trapped in
a depressed mood are inactive and feel unmotivated. They are sensitive to negative
happenings (Peckham et al., 2010). They more often recall negative information. They
expect negative outcomes (my team will lose, my grades will fall, my love will fail). When
the mood lifts, these behavioral and cognitive accompaniments disappear. Nearly half
the time, people also exhibit symptoms of another disorder, such as anxiety or substance
use disorder.
•
Depression is widespread. Its commonality suggests that its causes, too, must be common.
•
Women’s risk of major depression is nearly double men’s. When Gallup in 2009 asked
more than a quarter-million Americans if they had ever been diagnosed with depression,
13 percent of men and 22 percent of women said they had (Pelham, 2009). This gender
gap has been found worldwide (FIGURE 67.1). The trend begins in adolescence;
preadolescent girls are not more depression-prone than are boys (Hyde et al., 2008).
Life after depression Writer J. K.
ENGAGE
Critical Questions
Divide your students into small groups
and have them create a continuum
of the different types of mood, with
depression on one extreme and mania
on the other. Have them come up
with at least 5 different levels of mood
between the 2 extremes.
Could the groups reach a
consensus about what the
different types of mood were
called? Why or why not?
Rowling reports suffering acute
depression—a “dark time,” with
suicidal thoughts—between ages 25
and 28. It was, she said, a “terrible
place” that did, however, form a
foundation that allowed her “to come
back stronger” (McLaughlin, 2010).
Figure 67.1
Gender and major
depression Interviews with
89,037 adults in 18 countries
(10 shown here) confirm what
many smaller studies have
found: Women’s risk of major
depression is nearly double
that of men’s.
15%
Percentage
of adults
experiencing
major depression
in previous
12 months
5
Males
Israel
Italy
Japan Netherlands
New
Zealand
Spain
USA
Females
The factors that put women at risk for depression (genetic predispositions, child abuse,
low self-esteem, marital problems, and so forth) similarly put men at risk (Kendler et al.,
2006). Yet women are more vulnerable to disorders involving internalized states, such as
depression, anxiety, and inhibited sexual desire. Men’s disorders tend to be more external—
alcohol use disorder, antisocial conduct, lack of impulse control. When women get sad, they
often get sadder than men do. When men get mad, they often get madder than women do.
MyersAP_SE_2e_Mod67_B.indd 674
MyersPsyAP_TE_2e_U12.indd 674
10
Belgium France Germany
•
Unit XII
Around the world, women
are more susceptible to
depression
0
What were the different names
they came up with for each type of
mood?
674
How do the biological and social-cognitive perspectives explain
mood disorders?
Most major depressive episodes self-terminate. Although therapy often helps and
tends to speed recovery, most people suffering major depression eventually return
to normal even without professional help. The plague of depression comes and, a
few weeks or months later, it goes, though for about half of people it eventually
1/21/14 9:35 AM
Abnormal Behavior
3/5/14 11:03 AM
MyersAP_SE_2e
Mood Disorders
•
•
675
TEACH
Teaching Tip
Invite a psychologist who specializes
in treating mood disorders to address
your class.
Paula Niedenthal
recurs (Burcusa & Iacono, 2007; Curry et al.,
2011; Hardeveld et al., 2010). For only about 20
percent is the condition chronic (Klein, 2010). On
average, patients with major depression today will
spend about three-fourths of the next decade in
a normal, nondepressed state (Furukawa et al.,
2009). Recovery is more likely to be permanent
the later the first episode strikes, the longer the
person stays well, the fewer the previous episodes,
the less stress experienced, and the more social
support received (Belsher & Costello, 1988;
Fergusson & Woodward, 2002; Kendler et al.,
2001).
Module 67
What method or methods of
treatment does he or she prefer?
What is the current research on
mood disorders?
What are the rates of mood
disorders among high school
students?
The emotional lives of men and women?
Stressful events related to work, marriage, and
close relationships often precede depression.
A family member’s death, a job loss, a marital crisis, or a physical assault increase
one’s risk of depression (Kendler et al., 2008; Monroe & Reid, 2009; Orth et al., 2009).
If stress-related anxiety is a “crackling, menacing brushfire,” notes biologist Robert
Sapolsky (2003), “depression is a suffocating heavy blanket thrown on top of it.” One
long-term study (Kendler, 1998) tracked rates of depression in 2000 people. The risk
of depression ranged from less than 1 percent among those who had experienced
no stressful life event in the preceding month to 24 percent among those who had
experienced three such events in that month.
With each new generation, depression is striking earlier (now often in the late
teens) and affecting more people, with the highest rates in developed countries
among young adults. This is true in Canada, the United States, England, France,
Germany, Italy, Lebanon, New Zealand, Puerto Rico, and Taiwan (Collishaw et al.,
2007; Cross-National Collaborative Group, 1992; Kessler et al., 2010; Twenge et
al., 2008). In one study, 12 percent of Australian adolescents reported symptoms of
depression (Sawyer et al., 2000). Most hid it from their parents; almost 90 percent of
those parents perceived their depressed teen as not suffering depression. In North
America, today’s young adults are three times more likely than their grandparents to
report having recently—or ever—suffered depression (despite the grandparents’ many
more years of being at risk). The increase appears partly authentic, but it may also
reflect today’s young adults’ greater willingness to disclose depression.
“I see depression as the plague
of the modern era.” -LEWIS JUDD,
FORMER CHIEF, NATIONAL INSTITUTE OF
MENTAL HEALTH, 2000
Today’s researchers propose biological and cognitive explanations of depression, often
combined in a biopsychosocial approach.
TEACH
The Biological Perspective
1/21/14 9:35 AM
GENETIC INFLUENCES
Concept Connections
Mood disorders run in families. As one researcher noted, emotions are “postcards from
our genes” (Plotkin, 1994). The risk of major depression and bipolar disorder increases if
you have a parent or sibling with the disorder (Sullivan et al., 2000). If one identical twin is
diagnosed with major depressive disorder, the chances are about 1 in 2 that at some time
the other twin will be, too. If one identical twin has bipolar disorder, the chances are 7 in 10
that the other twin will at some point be diagnosed similarly. Among fraternal twins, the
corresponding odds are just under 2 in 10 (Tsuang & Faraone, 1990). The greater similarity
among identical twins holds even among twins raised apart (DiLalla et al., 1996). Summarizing the major twin studies, one research team estimated the heritability (extent to which
individual differences are attributable to genes) of major depression at 37 percent (FIGURE
67.2 on the next page).
Remind students that being genetically predisposed to a condition such
as depression does not mean one is
guaranteed to get the disease. Mitigating environmental circumstances
can help a person who is genetically
predisposed to depression avoid the
symptoms or incidence of disease.
MyersAP_SE_2e_Mod67_B.indd 675
1/21/14 9:35 AM
Mood Disorders
MyersPsyAP_TE_2e_U12.indd 675
Module 67
675
3/5/14 11:03 AM
Unit XII Abnormal Behavior
676
TEACH
Figure 67.2
The heritability of various
psychological disorders
Teaching Tip
Researchers Joseph Bienvenu, Dimitry
Davydow, and Kenneth Kendler (2011)
aggregated data from studies of
identical and fraternal twins to estimate
the heritability of bipolar disorder,
schizophrenia, anorexia nervosa, major
depressive disorder, and generalized
anxiety disorder.
Suicide may be a sensitive subject,
especially if students have known
someone who has attempted or committed suicide. Some students may
also have attempted suicide and not
shared that fact with others. Teens and
young adults generally struggle with
questions on life’s purpose, and these
feelings are normal. Remind students
that if there is any indication that a
friend or fellow student is contemplating suicide, this should be taken
seriously.
90%
80
70
60
Bipolar disorder
Schizophrenia
50
Anorexia nervosa
40
Major depressive disorder
30
Generalized anxiety disorder
20
10
0
Heritability estimates
Moreover, adopted people who suffer a mood disorder often have close biological relatives who suffer mood disorders, develop alcohol use disorder, or commit suicide (Wender
et al., 1986). (Close-up: Suicide and Self-Injury reports other research findings.)
Close-up
Suicide and Self-Injury
ENGAGE
TR M
TRM
67-3
Enrichment
What reasons do people give for
being lonely? One survey sorted them
into 5 major categories:
Each year nearly 1 million despairing people worldwide will elect
a permanent solution to what might have been a temporary
problem. Comparing the suicide rates of different groups, researchers have found
2. Alienation. Being misunderstood
and feeling different; not being
needed and having no close
friends
3. Being alone. Coming home to an
empty house
4. Forced isolation. Being hospitalized or housebound; having no
transportation
5. Dislocation. Being away from
home; starting a new job or school;
traveling often
676
Unit XII
MyersPsyAP_TE_2e_U12.indd 676
•
other group differences: Suicide rates are much
higher among the rich, the nonreligious, and those who
are single, widowed, or divorced (Hoyer & Lund, 1993;
Stack, 1992; Stengel, 1981). When facing an unsupportive
environment, including family or peer rejection, gay and
lesbian youth are at increased risk of attempting suicide
(Goldfried, 2001; Haas et al., 2011; Hatzenbuehler, 2011).
•
day of the week differences: 25 percent of suicides
occur on Wednesdays (Kposowa & D’Auria, 2009).
““But life, being weary of these worldly bars, / Never lacks power
to dismiss itself.” -WILLIAM SHAKESPEARE, JULIUS CAESAR, 1599
1. Being unattached. Having no
spouse or partner, particularly
breaking up with a spouse or
significant other
Use Student Activity: Loneliness from
the TRM to help students further
explore this topic.
What factors affect suicide and selfinjury, and what are some of the
important warning signs to watch for in
suicide prevention efforts?
•
national differences: Britain’s, Italy’s, and Spain’s suicide
rates are little more than half those of Canada, Australia,
and the United States. Austria’s and Finland’s are about
double (WHO, 2011). Within Europe, people in the most
suicide-prone country (Belarus) have been 16 times more
likely to kill themselves than those in the least (Georgia).
•
racial differences: Within the United States, Whites kill
themselves twice as often as Blacks (AAS, 2010).
•
gender differences: Women are much more likely than
men to attempt suicide (WHO, 2011). But men are two
to four times more likely (depending on the country) to
actually end their lives. Men use more lethal methods,
such as firing a bullet into the head, the method of choice
in 6 of 10 U.S. suicides.
•
age differences and trends: In late adulthood, rates
increase, peaking in middle age and beyond. In the last
half of the twentieth century, the global rate of annual
suicide deaths nearly doubled (WHO, 2008).
MyersAP_SE_2e_Mod67_B.indd 676
The risk of suicide is at least five times greater for those
who have been depressed than for the general population
(Bostwick & Pankratz, 2000). People seldom commit suicide
while in the depths of depression, when energy and initiative
are lacking. The risk increases when they begin to rebound
and become capable of following through. Among people
with alcohol use disorder, 3 percent die by suicide. This rate
is roughly 100 times greater than the rate for people without
alcohol use disorder (Murphy & Wetzel, 1990; Sher, 2006).
Because suicide is so often an impulsive act, environmental
barriers (such as jump barriers on high bridges and the unavailability of loaded guns) can reduce suicides (Anderson, 2008).
Although common sense might suggest that a determined person would simply find another way to complete the act, such
restrictions give time for self-destructive impulses to subside.
Social suggestion may trigger suicide. Following highly publicized suicides and TV programs featuring suicide, known suicides
increase. So do fatal auto and private airplane “accidents.” One
six-year study tracked suicide cases among all 1.2 million people
who lived in metropolitan Stockholm at any time during the 1990s
(continued)
1/21/14 9:35 AM
Abnormal Behavior
3/7/14 9:54 AM
MyersAP_SE_2e
Mood Disorders
Close-up
Module 67
677
TEACH
(continued)
(Hedström et al., 2008). Men exposed to a family suicide were 8
times more likely to commit suicide than were nonexposed men.
Although that phenomenon may be partly attributable to family
genes, shared genetic predispositions do not explain why men
exposed to a co-worker’s suicide were 3.5 times more likely to
commit suicide, compared with nonexposed men.
Suicide is not necessarily an act of hostility or revenge. The elderly sometimes choose death as an alternative to current or future
suffering. In people of all ages, suicide may be a way of switching
off unendurable pain and relieving a perceived burden on family
members. “People desire death when two fundamental needs are
frustrated to the point of extinction,” notes Thomas Joiner (2006, p.
47): “The need to belong with or connect to others, and the need
to feel effective with or to influence others.” Suicidal urges typically
arise when people feel disconnected from others, and a burden
to them (Joiner, 2010), or when they feel defeated and trapped by
an inescapable situation (Taylor et al., 2011). Thus, suicide rates
increase a bit during economic recessions (Luo et al., 2011). Suicidal thoughts also may increase when people are driven to reach
a goal or standard—to become thin or straight or rich—and find it
unattainable (Chatard & Selimbegović, 2011).
In hindsight, families and friends may recall signs they believe
should have forewarned them—verbal hints, giving possessions
away, or withdrawal and preoccupation with death. To judge from
surveys of 84,850 people across 17 nations, about 9 percent of
people at some point in their lives have thought seriously of suicide.
About 30 percent of these (3 percent of people) actually attempt
it (Nock et al., 2008). For only about 1 in 25 does the attempt become their final act (AAS, 2009). Of those who die, one-third had
tried to kill themselves previously. Most discussed it beforehand.
So, if a friend talks suicide to you, it’s important to listen and to direct the person to professional help. Anyone who threatens suicide
is at least sending a signal of feeling desperate or despondent.
Active Learning
NONSUICIDAL SELF-INJURY
Suicide is not the only way to send a message or deal with
distress. Some people, especially adolescents and young
adults, may engage in nonsuicidal self-injury (NSSI) (FIGURE
67.3). Such behavior includes cutting or burning the skin, hitting oneself, pulling hair out, inserting objects under the nails
or skin, and self-administered tattooing (Fikke et al., 2011).
Why do people hurt themselves? Those who do so tend
to be less able to tolerate emotional distress, are extremely
self-critical, and often have poor communication and problemsolving skills (Nock, 2010). They engage in NSSI to
•
gain relief from intense negative thoughts through the
distraction of pain,
•
ask for help and gain attention,
•
relieve guilt by self-punishment
•
get others to change their negative behavior (bullying,
criticism), or
•
to fit in with a peer group.
Divide students into groups, and ask
them to come up with several serious,
helpful responses they might offer to
a friend, family member, or acquaintance who is communicating suicidal
thoughts.
Does NSSI lead to suicide? Usually not. Those who engage in NSSI are typically suicide gesturers, not suicide attempters (Nock & Kessler, 2006). Suicide gesturers engage
in NSSI as a desperate but non-life-threatening form of communication or when they are feeling overwhelmed. But NSSI
has been shown to be a risk factor for future suicide attempts
(Wilkinson & Goodyer, 2011). If people do not get help, their
nonsuicidal behavior may escalate to suicidal ideation and finally, to attempted suicide.
How would they respond to
someone who seems preoccupied
with death?
How would they respond to
someone who tries to give them a
prized possession?
What would they say to someone
who reveals that he or she is
contemplating suicide?
ENGAGE
Enrichment
How do people cope with loneliness?
Carin Rubenstein and Phillip Shaver
have identified 4 major strategies.
Figure 67.3
Rates of nonfatal self-injury in the U.S. Self-injury rates peak
higher for females than for males (CDC, 2009).
Sad passivity includes sleeping,
drinking, overeating, and watching
TV.
Social contact may involve calling a
friend or visiting someone.
Active solitude takes the form of
studying, reading, exercising, or
going to a movie.
Distractions include spending
money and shopping.
450
Injury rate 400
per 100,000
people 350
Females
300
250
200
150
100
Males
50
0
0–4
5–9
10–14 15–19 20–24 25–29 30–34 35–39 40–44 45–49 50–54 55–59 60–64 65–69 70–74 75–79 80–84
Rubenstein, C. M., & Shaver, P. (1982). In
search of intimacy. New York: Delacorte
Press.
85>
Age group (years)
1/21/14 9:35 AM
MyersAP_SE_2e_Mod67_B.indd 677
1/21/14 9:35 AM
Mood Disorders
MyersPsyAP_TE_2e_U12.indd 677
Module 67
677
3/5/14 11:03 AM
678
Unit XII Abnormal Behavior
To tease out the genes that put people at risk for depression, some researchers have turned
to linkage analysis. After finding families in which the disorder appears across several generations, geneticists examine DNA from affected and unaffected family members, looking for differences. Linkage analysis points us to a chromosome neighborhood, note behavior genetics
researchers Robert Plomin and Peter McGuffin (2003); “a house-to-house search is then needed
to find the culprit gene.” Such studies are reinforcing the view that depression is a complex condition. Many genes work together, producing a mosaic of small effects that interact with other
factors to put some people at greater risk. If the culprit gene variations can be identified—with
chromosome 3 genes implicated in separate British and American studies (Breen et al., 2011;
Pergadia et al., 2011)—they may open the door to more effective drug therapy.
Concept Connections
Have students research the neurotransmitters most related to mood disorders. Extend the research to include
the types of drugs used to treat these
conditions. Have students discover
how the drugs act on the neural sites
to affect behavior.
A P ® E x a m Ti p
You can review brain scanning
techniques, neurotransmitters,
and brain structures in Unit III.
TEACH
Teaching Tip
Research shows that patients with
mood disorders respond best to treatments that use both medical therapies
(that is, drugs) and cognitive/ behavioral therapy. Each alone does not
seem as effective as both combined.
THE DEPRESSED BRAIN
Using modern technology, researchers are also gaining insight into brain activity during
depressed and manic states, and into the effects of certain neurotransmitters during these
states. One study gave 13 elite Canadian swimmers the wrenching experience of watching
a video of the swim in which they failed to make the Olympic team or failed at the Olympic
games (Davis et al., 2008). Functional MRI scans showed the disappointed swimmers experiencing brain activity patterns akin to those of patients with depressed moods.
Many studies have found diminished brain activity during slowed-down depressive
states, and more activity during periods of mania (FIGURE 67.4). The left frontal lobe and
an adjacent brain reward center are active during positive emotions, but less active during
depressed states (Davidson et al., 2002; Heller et al., 2009). In one study of people with
severe depression, MRI scans also found their frontal lobes 7 percent smaller than normal
(Coffey et al., 1993). Other studies show that the hippocampus, the memory-processing
center linked with the brain’s emotional circuitry, is vulnerable to stress-related damage.
Bipolar disorder likewise correlates with brain structure. Neuroscientists have found
structural differences, such as decreased axonal white matter or enlarged fluid-filled ventricles,
in the brains of people with bipolar disorder (Kempton et al., 2008; van der Schot et al., 2009).
Neurotransmitter systems influence mood disorders. Norepinephrine, which increases arousal and boosts mood, is scarce during depression and overabundant during mania.
(Drugs that alleviate mania reduce norepinephrine.) Many people with a history of depression also have a history of habitual smoking, and smoking increases one’s risk for future
depression (Pasco et al. 2008). This may indicate an attempt to self-medicate with inhaled
nicotine, which can temporarily increase norepinephrine and boost mood (HMHL, 2002b).
Researchers are also exploring a second neurotransmitter, serotonin (Carver et al.,
2008). One well-publicized study of New Zealand young adults found that the recipe for
depression combined two necessary ingredients—significant life stress plus a variation on
a serotonin-controlling gene (Caspi et al., 2003; Moffitt et al., 2006). Depression arose from
the interaction of an adverse environment plus a genetic susceptibility, but not from either
alone. But stay tuned: The story of gene-environment interactions is still being written, as
other researchers debate the reliability of this result (Caspi et al., 2010; Karg et al., 2011;
Munafò et al., 2009; Risch et al., 2009; Uher & McGuffin, 2010).
Courtesy of Drs. Lewis Baxter and Michael E.
Phelps, UCLA School of Medicine
TEACH
Figure 67.4
The ups and downs of bipolar
disorder These top-facing PET
scans show that brain energy
consumption rises and falls with the
patient’s emotional switches. Areas
where the brain rapidly consumes
glucose are shown in red in these
images.
Depressed state
(May 17)
MyersAP_SE_2e_Mod67_B.indd 678
678
Unit XII
MyersPsyAP_TE_2e_U12.indd 678
Manic state
(May 18)
Depressed state
(May 27)
1/23/14 11:16 AM
Abnormal Behavior
3/5/14 11:03 AM
MyersAP_SE_2e
Mood Disorders
Module 67
679
Drugs that relieve depression tend to increase norepinephrine or serotonin supplies
by blocking either their reuptake (as Prozac, Zoloft, and Paxil do with serotonin) or their
chemical breakdown. Repetitive physical exercise, such as jogging, reduces depression as it
increases serotonin (Ilardi, 2009; Jacobs, 1994). Boosting serotonin may promote recovery
from depression by stimulating hippocampus neuron growth (Airan et al., 2007; Jacobs et
al., 2000).
What’s good for the heart is also good for the brain and mind. People who eat a hearthealthy “Mediterranean diet” (heavy on vegetables, fish, and olive oil) have a comparatively low risk of developing heart disease, late-life cognitive decline, and depression—all of
which are associated with inflammation (Dowlati et al., 2010; Sánchez-Villegas et al., 2009;
Tangney et al., 2011). Excessive alcohol use also correlates with depression—mostly because
alcohol misuse leads to depression (Fergusson et al., 2009).
The Social-Cognitive Perspective
ENGAGE
Depression is a whole-body disorder. Biological influences contribute to depression but don’t
fully explain it. The social-cognitive perspective explores the roles of thinking and acting.
Depressed people view life through the dark glasses of low self-esteem (Orth et al.,
2009). Their intensely negative assumptions about themselves, their situation, and their future lead them to magnify bad experiences and minimize good ones. Listen to Norman, a
Canadian college professor, recalling his depression:
Michael Marsland
TR M
TRM
I [despaired] of ever being human again. I honestly felt subhuman, lower than the
lowest vermin. Furthermore, I was self-deprecatory and could not understand why
anyone would want to associate with me, let alone love me. . . . I was positive that
I was a fraud and a phony and that I didn’t deserve my Ph.D. I didn’t deserve to
have tenure; I didn’t deserve to be a Full Professor. . . . I didn’t deserve the research
grants I had been awarded; I couldn’t understand how I had written books and
journal articles. . . . I must have conned a lot of people. (Endler, 1982, pp. 45–49)
Research reveals how self-defeating beliefs and a negative explanatory style feed depression’s
vicious cycle.
NEGATIVE THOUGHTS AND NEGATIVE MOODS INTERACT
Self-defeating beliefs may arise from learned helplessness. As we saw in Module 29, both
dogs and humans act depressed, passive, and withdrawn after experiencing uncontrollable
painful events. Learned helplessness is more common in women than in men, and women may respond more strongly to stress (Hankin & Abramson, 2001; Mazure et al., 2002;
Nolen-Hoeksema, 2001, 2003). For example, 38 percent of women and 17 percent of men
entering U. S. colleges and universities report feeling at least occasionally “overwhelmed
by all I have to do” (Pryor et al., 2006). (Men report spending more of their time in “light
anxiety” activities such as sports, TV watching, and partying, possibly avoiding activities that
might make them feel overwhelmed.) This may help explain why, beginning in their early
teens, women are nearly twice as vulnerable to depression. Susan Nolen-Hoeksema (2003)
believed women’s higher risk of depression relates to what she described as their tendency
to overthink, to ruminate. Rumination—staying focused on a problem (thanks to the continuous firing of a frontal lobe area that sustains attention)—can be adaptive (Altamirano et
al., 2010; Andrews & Thomson, 2009a,b). But when it is relentless, self-focused rumination
diverts us from thinking about other life tasks and produces a negative emotional inertia
(Kuppens et al., 2010).
But why do life’s unavoidable failures lead only some people to become depressed?
The answer lies partly in their explanatory style—who or what they blame for their failures
(or credit for their successes). Think of how you might feel if you failed a test. If you can externalize the blame (“What an unfair test!”), you are more likely to feel angry. If you blame
yourself, you probably will feel stupid and depressed.
1/23/14 11:16 AM
MyersAP_SE_2e_Mod67_B.indd 679
Explanatory style and learned helplessness help explain depression, but
what about explanations for mania?
Have students brainstorm on different
psychological explanations for mania:
䊉
How might learning theorists
explain mania?
䊉
What attributions might a person
in a manic state make about his or
her behavior? Would the person be
optimistic or pessimistic?
Susan Nolen-Hoeksema
(1959–2013) “This epidemic of
morbid meditation is a disease that
women suffer much more than men.
Women can ruminate about anything
and everything—our appearance,
our families, our career, our health.”
(Women Who Think Too Much: How
to Break Free of Overthinking and
Reclaim Your Life, 2003)
Use Student Activity: The Automatic
Thoughts Questionnaire from the
TRM to help students assess how they
think in relation to mood disorders by
measuring the frequency of automatic
negative thoughts.
TEACH
rumination compulsive fretting;
overthinking about our problems
and their causes.
Concept Connections
“I have learned to accept my
mistakes by referring them to a
personal history which was not of
my making.” -B. F. SKINNER (1983)
1/21/14 9:35 AM
Remind students that learned helplessness, an early experimental basis
for explanatory style, has its roots in
classical conditioning. Using classical conditioning techniques, Martin
Seligman and Steven Maier found that
dogs learned to be helpless in the face
of uncontrollable stimuli. This led the
2 men to wonder if depression has its
roots in learning theory.
Mood Disorders
MyersPsyAP_TE_2e_U12.indd 679
Critical Questions
Module 67
679
3/5/14 11:03 AM
Unit XII Abnormal Behavior
680
ENGAGE
Active Learning
Stable
“I’ll never get over this.”
Global
“Without my boyfriend/girlfriend,
I can’t seem to do
anything
ything
g right.”
righ
Internal
“Our breakup was all my fault.”
Does the cause you named reflect
more on you (internal) rather than
other people or circumstances
(external)?
Is the cause something that is
permanent/stable (likely to be
present in the future), or is it
temporary?
Depression
Figure 67.5
Explanatory style and
depression After a negative
experience, a depression-prone
person may respond with a negative
explanatory style.
Is the cause something that
influences other areas of your
life (global) or only your account
balance (specific)?
Ask for volunteers to share their
answers and reiterate that attributions
for events that are internal, stable, and
global are most likely to be associated
with depression.
PEANUTS
Might Charlie
Brown be
helped by an
optimismtraining
program?
TEACH
Concept Connections
Peter Barnett and Ian Gotlib point out
that certain types of thinking may be
related to depression, but they might
not be the causes of depression. Further experimental evidence is needed
to determine the exact causes of the
illness. Have students suggest ways
this experimentation might occur
using their knowledge of the scientific
method.
680
Unit XII
MyersPsyAP_TE_2e_U12.indd 680
Permisson of United Features
Syndicate, Inc.
Ask students to imagine receiving
a notice that their bank account is
overdrawn. Have them write down in
a sentence or 2 what they believe to
be the single most important cause.
Then, in considering their written
responses, have them answer the following questions:
So it is with depressed people, who tend to explain bad events in terms that are stable (“It’s going to
last forever”), global (“It’s going to affect everything I
do”), and internal (“It’s all my fault”) (FIGURE 67.5).
Depression-prone people respond to bad events in
Temporary
“This is hard to take, but I
an especially self-focused, self-blaming way (Mor &
will get through this.”
Winquist, 2002; Pyszczynski et al., 1991; Wood et al.,
1990a,b). Their self-esteem fluctuates more rapidly
up with boosts and down with threats (Butler et al.,
1994).
Specific
“I miss my boyfriend/girlfriend,
The result of these pessimistic, overgeneralized,
but thankfully I have family
self-blaming attributions may be a depressing sense
and other frie
friends.”
of hopelessness (Abramson et al., 1989; Panzarella et
al., 2006). As Martin Seligman has noted, “A recipe
for severe depression is preexisting pessimism enExternal
countering failure” (1991, p. 78). What then might we
“It takes two to make a relationship
work and it wasn’t
wasn t meant
me
to be.”
expect of new college students who are not depressed
but do exhibit a pessimistic explanatory style? Lauren
Alloy and her collaborators (1999) monitored Temple
University and University of Wisconsin students evSuccessful coping
ery 6 weeks for 2.5 years. Among those identified as
having a pessimistic thinking style, 17 percent had a
first episode of major depression, as did only 1 percent of those who began college with an
optimistic thinking style.
Seligman (1991, 1995) has contended that depression is common among young Westerners because the rise of individualism and the decline of commitment to religion and
family have forced young people to take personal responsibility for failure or rejection. In
non-Western cultures, where close-knit relationships and cooperation are the norm, major
depression is less common and less tied to self-blame over personal failure (WHO, 2004a).
In Japan, for example, depressed people instead tend to report feeling shame over letting
others down (Draguns, 1990a).
Breakup with a boyfriend/girlfriend
“Man never reasons so much
and becomes so introspective
as when he suffers, since he is
anxious to get at the cause of his
sufferings.” -LUIGI PIRANDELLO,
SIX CHARACTERS IN SEARCH OF AN
AUTHOR, 1922
MyersAP_SE_2e_Mod67_B.indd 680
There is, however, a chicken-and-egg problem with the social-cognitive explanation
of depression. Self-defeating beliefs, negative attributions, and self-blame coincide with a
depressed mood and are indicators of depression. But do they cause depression, any more
than a speedometer’s reading causes a car’s speed? Before or after being depressed, people’s thoughts are less negative. Perhaps this is because, as we noted in our discussion of
state-dependent memory (Module 32), a depressed mood triggers negative thoughts. If you
temporarily put people in a bad or sad mood, their memories, judgments, and expectations
suddenly become more pessimistic.
DEPRESSION’S VICIOUS CYCLE
Depression, as we have seen, is often brought on by stressful experiences—losing a job,
getting divorced or rejected, suffering physical trauma—by anything that disrupts our
sense of who we are and why we are worthy human beings. This disruption in turn leads
1/21/14 9:35 AM
Abnormal Behavior
3/5/14 11:03 AM
MyersAP_SE_2e
Mood Disorders
Module 67
681
to brooding, which amplifies negative feelings.
Figure 67.6
1
But being withdrawn, self-focused, and comThe vicious cycle of
Stressful
depressed thinking
experiences
plaining can by itself elicit rejection (Furr &
Cognitive therapists
Funder, 1998; Gotlib & Hammen, 1992). In one
attempt to break this
study, researchers Stephen Strack and James
cycle, as we will see in
4
2
Module 71, by changing
Coyne (1983) noted that “depressed persons inCognitive and
Negative
the way depressed
duced hostility, depression, and anxiety in others
behavioral changes
explanatory
p
y style
people process events.
and got rejected. Their guesses that they were not
Psychiatrists attempt
to alter with medication
accepted were not a matter of cognitive distorthe biological roots of
tion.” Indeed, people in the throes of depression
3
persistently depressed
Depressed
are at high risk for divorce, job loss, and other
moods.
mood
ss
stressful life events. Weary of the person’s fatigue, hopeless
attitude, and lethargy, a spouse may threaten to leave or a
boss may begin to question the person’s competence. (This provides another example of
genetic-environmental interaction: People genetically predisposed to depression more often
experience depressing events.) The losses and stress only serve to compound the original
“Some cause happiness wherever
depression. Rejection and depression feed each other. Misery may love another’s company,
they go; others, whenever they
go.” -IRISH WRITER OSCAR WILDE
but company does not love another’s misery.
(1854–1900)
We can now assemble some of the pieces of the depression puzzle (FIGURE 67.6):
(1) Negative, stressful events interpreted through (2) a ruminating, pessimistic explanatory
style create (3) a hopeless, depressed state that (4) hampers the way the person thinks and
acts. This, in turn, fuels (1) negative, stressful experiences such as rejection.
None of us is immune to the dejection, diminished self-esteem, and negative thinking
brought on by rejection or defeat. As Edward Hirt and his colleagues (1992) demonstrated,
even small losses can temporarily sour our thinking. They studied some avid Indiana University basketball fans who seemed to regard the team as an extension of themselves. After
the fans watched their team lose or win, the researchers asked them to predict the team’s
future performance and their own. After a loss, the morose fans offered bleaker assessments
not only of the team’s future but also of their own likely performance at throwing darts,
solving anagrams, and getting a date. When things aren’t going our way, it may seem as
though they never will.
It is a cycle we can all recognize. Bad moods feed on themselves: When we feel down,
we think negatively and remember bad experiences. On the brighter side, we can break
the cycle of depression at any of these points—by moving to a different environment, by
reversing our self-blame and negative attributions, by turning our attention outward, or by
engaging in more pleasant activities and more competent behavior.
Winston Churchill called depression a “black dog” that periodically hounded him. Poet
Emily Dickinson was so afraid of bursting into tears in public that she spent much of her
adult life in seclusion (Patterson, 1951). As each of these lives reminds us, people can and do
struggle through depression. Most regain their capacity to love, to work, and even to succeed at the highest levels.
ENGAGE
Enrichment
Barbara Frederickson at the University
of North Carolina, Chapel Hill, has
proposed a theory of positive emotions. While negative emotions make
us selfish and closed to the world
around us, positive emotions broaden
and build our lives. When experiencing positive emotions, people tend
to be helpful and spread their good
feelings around. Frederickson calls this
the “broaden and build” theory.
CLOSE & ASSESS
Exit Assessment
Have students write the definition of
each of the disorders in this module.
Be sure they include the major diagnostic criteria for each.
Before You Move On
䉴 ASK YOURSELF
Has your high school experience been a challenging time for you? What advice would you
have for other students about to enter high school?
䉴 TEST YOURSELF
What is the most common psychological disorder? What is the disorder for which people most
often seek treatment?
Answers to the Test Yourself questions can be found in Appendix E at the end of the book.
1/21/14 9:35 AM
MyersAP_SE_2e_Mod67_B.indd 681
1/21/14 9:35 AM
Mood Disorders
MyersPsyAP_TE_2e_U12.indd 681
Module 67
681
3/5/14 11:03 AM
682
Unit XII Abnormal Behavior
Module 67 Review
67-1
•
•
Mood disorders are characterized by emotional extremes.
•
A person with the less common condition of bipolar
disorder experiences not only depression but also mania—
episodes of hyperactive and wildly optimistic, impulsive
behavior.
A person with major depressive disorder experiences two or
more weeks of seriously depressed moods and feelings of
worthlessness, and takes little interest in, and derives little
pleasure from, most activities.
67-2
•
•
Answers to Multiple-Choice
Questions
1. b
2. e
What are mood disorders? How does major
depressive disorder differ from bipolar
disorder?
How do the biological and social-cognitive
perspectives explain mood disorders?
The biological perspective on depression focuses on
genetic predispositions and on abnormalities in brain
structures and function (including those found in
neurotransmitter systems).
The social-cognitive perspective views depression as an
ongoing cycle of stressful experiences (interpreted through
negative beliefs, attributions, and memories) leading to
negative moods and actions and fueling new stressful
experiences.
1. Which of the following is NOT a symptom of major
depressive disorder?
3. d
a.
b.
c.
d.
e.
Weight gain or loss
Auditory hallucinations
Sleep disturbance
Inappropriate guilt
Problems concentrating
a. Depression usually develops during middle age.
b. Depression usually happens without major cognitive
or behavioral changes.
c. A major depressive episode usually gets worse and
worse unless it’s treated.
d. True depression is usually not related to stress in
one’s work or relationships.
e. Compared with men, nearly twice as many women
have been diagnosed with depression.
MyersAP_SE_2e_Mod67_B.indd 682
Unit XII
MyersPsyAP_TE_2e_U12.indd 682
What factors affect suicide and self-injury,
and what are some of the important warning
signs to watch for in suicide-prevention
efforts?
•
Suicide rates differ by nation, race, gender, age group,
income, religious involvement, marital status, and (for gay
and lesbian youth) social support structure.
•
Those with depression are more at risk for suicide than
others are, but social suggestion, health status, and
economic and social frustration are also contributing
factors.
•
Environmental barriers (such as jump barriers) are
effective in preventing suicides.
•
Forewarnings of suicide may include verbal hints, giving
away possessions, withdrawal, preoccupation with death,
and discussing one’s own suicide.
•
Nonsuicidal self-injury (NSSI) does not usually lead to
suicide but may escalate to suicidal thoughts and acts if
untreated.
•
People who engage in NSSI do not tolerate stress well
and tend to be self-critical, with poor communication and
problem-solving skills.
Multiple-Choice Questions
2. Which of the following is true of depression?
682
67-3
3. Which of the following is true of suicide?
a. Marijuana use is related to suicide, but alcohol use is
not.
b. Women are more likely to end their lives than men.
c. Suicide is a bigger problem among the poor than the
rich.
d. In the United States, suicide is more common among
Whites than Blacks.
e. Married individuals are more likely to commit suicide
than single people.
1/21/14 9:35 AM
Abnormal Behavior
3/5/14 11:03 AM
MyersAP_SE_2e
Mood Disorders
4. Based on brain scans, which of the following is true of
brain function and mood?
a. The brain is more active during manic episodes and
less active during depressive episodes.
b. The brain is less active during manic episodes and
more active during depressive episodes.
c. There is no consistent relationship between brain
activity and mood.
d. The brain is more active than normal during both
manic and depressive episodes.
e. The brain is less active than normal during both
manic and depressive episodes.
Module 67
683
5. Xavier, who has a negative explanatory style, is most
4. a
5. e
likely to get depressed after failing a math test if he
believes that he failed because
a.
b.
c.
d.
he is not good at math and never will be.
his teacher made it impossible to learn the material.
he was sick on the day he took the test.
his parents have been putting too much pressure on
him and he panicked on the test.
e. the testing room was very hot and stuffy.
Answer to Practice FRQ 2
Practice FRQs
1. Christina became depressed after being laid off from her
job. Her therapist thinks it’s because she has a stable,
global, and internal explanatory style. Illustrate each
of these three attributes by writing a possible thought
Christina might have for each one.
2. Identify and describe the two major symptoms of bipolar
disorder.
(4 points)
Answer
1 point: Stable thought (For example, “I have always had
trouble holding down a job”).
2 points: Depression: students can
name any one of the 9 symptoms of
depression from Table 67. 1 on page
672 (for example, depressed mood or
diminished interest in activities).
2 points: Mania: a euphoric, hyperactive, or wildly optimistic state.
1 point: Global thought (For example, “Everything in my life
is messed up”).
1 point: Internal thought (For example, “It’s all my fault I
lost this job”).
1/21/14 9:35 AM
MyersAP_SE_2e_Mod67_B.indd 683
1/21/14 9:35 AM
Mood Disorders
MyersPsyAP_TE_2e_U12.indd 683
Module 67
683
3/5/14 11:03 AM
684
TEACH
Module Learning Objectives
Teaching Tip
Arrange a field trip to a local mental
health care facility to learn more
about the most common disorders
among patients in mental institutions. Students should tour the facility
and meet with staff psychologists
or psychiatrists to learn more about
how schizophrenia is treated. You
may also want to ask your students to
bring donations of clothing, personal
hygiene products, and games for the
patients.
TEACH
Teaching Tip
Children as young as 12 have developed symptoms of schizophrenia. The
onset of the disorder typically occurs
during the late teen and early adult
years. Full-blown psychotic episodes
(where patients lose touch with reality) may not occur until the patient is
living on his or her own, apart from
family and friends who have provided
support in the past.
68-1
Describe the patterns of thinking, perceiving, and feeling
that characterize schizophrenia.
68-2
Contrast chronic and acute schizophrenia.
68-3
Discuss how brain abnormalities and viral infections help explain
schizophrenia.
68-4
Discuss the evidence for genetic influences on schizophrenia,
and describe some factors that may be early warning signs of
schizophrenia in children.
schizophrenia a psychological
disorder characterized by delusions,
hallucinations, disorganized speech,
and/or diminished or inappropriate
emotional expression.
psychosis a psychological disorder
in which a person loses contact with
reality, experiencing irrational ideas
and distorted perceptions.
delusions false beliefs, often of
persecution or grandeur, that may
accompany psychotic disorders.
Unit XII
MyersPsyAP_TE_2e_U12.indd 684
I
magine trying to communicate with Maxine, a young woman with schizophrenia whose
thoughts spill out in no logical order. Her biographer, Susan Sheehan (1982, p. 25), observed her saying aloud to no one in particular, “This morning, when I was at Hillside
[Hospital], I was making a movie. I was surrounded by movie stars. . . . I’m Mary Poppins.
Is this room painted blue to get me upset? My grandmother died four weeks after my eighteenth birthday.”
Nearly 1 in 100 people (about 60 percent men) develop schizophrenia, with an estimated 24 million across the world suffering from this dreaded disorder (Abel et al., 2010;
WHO, 2011).
Symptoms of Schizophrenia
68-1
A P ® E x a m Ti p
It is common for the AP® exam
to measure your awareness
of various “media myths”
about psychology. One of
the most common of these
myths is that schizophrenia
means “split personality” or
“multiple personality.” Read this
section carefully to achieve an
accurate understanding of what
schizophrenia is—and isn’t.
MyersAP_SE_2e_Mod68_B.indd 684
684
es
Schizophrenia
ty Imag
Use the Module 68 Fact or Falsehood?
activity from the TRM to introduce the
concepts from this module.
Module 68
via Get
Discussion Starter
r Post
TR M
TRM
Denve
TEACH
Unit XII Abnormal Behavior
What patterns of thinking, perceiving, and feeling characterize
schizophrenia?
Literally translated, schizophrenia means “split mind.” It refers not to a multiplepersonality split but rather to a split from reality that shows itself in disturbed perceptions,
disorganized thinking and speech, and diminished, inappropriate emotions. As such, it is
the chief example of a psychosis, a psychotic disorder marked by irrationality and lost contact with reality.
Disorganized Thinking and Disturbed Perceptions
As Maxine’s strange monologue illustrates, the thinking of a person with schizophrenia is
fragmented, bizarre, and often distorted by false beliefs called delusions (“I’m Mary Poppins”). Those with paranoid tendencies are particularly prone to delusions of persecution.
Even within sentences, jumbled ideas may create what is called word salad. One young man
1/21/14 9:35 AM
Abnormal Behavior
3/5/14 11:03 AM
MyersAP_SE_2e
Schizophrenia
begged for “a little more allegro in the treatment,” and suggested that “liberationary movement with a view to the widening of the horizon” will “ergo extort some wit in lectures.”
A person with schizophrenia may have hallucinations
(sensory experiences without sensory stimulation). They may
see, feel, taste, or smell things that are not there. Most often,
however, the hallucinations are auditory, frequently voices
making insulting remarks or giving orders. The voices may tell
the patient that she is bad or that she must burn herself with
a cigarette lighter. Imagine your own reaction if a dream broke
into your waking consciousness. When the unreal seems real,
the resulting perceptions are at best bizarre, at worst terrifying.
Disorganized thoughts may result from a breakdown in
selective attention. Recall from Module 16 that we normally
have a remarkable capacity for giving our undivided attention
to one set of sensory stimuli while filtering out others. Those
with schizophrenia cannot do this. Thus, irrelevant, minute
stimuli, such as the grooves on a brick or the inflections of a voice, may distract their attention from a bigger event or a speaker’s meaning. As one former patient recalled, “What had
happened to me . . . was a breakdown in the filter, and a hodge-podge of unrelated stimuli
were distracting me from things which should have had my undivided attention” (MacDonald, 1960, p. 218). This selective-attention difficulty is but one of dozens of cognitive differences associated with schizophrenia (Reichenberg & Harvey, 2007).
Diminished and Inappropriate Emotions
The expressed emotions of schizophrenia are often utterly inappropriate, split off from reality (Kring & Caponigro, 2010). Maxine laughed after recalling her grandmother’s death. On
other occasions, she cried when others laughed, or became angry for no apparent reason.
Others with schizophrenia lapse into an emotionless state of flat affect. Most also have difficulty perceiving facial emotions and reading others’ states of mind (Green & Horan, 2010;
Kohler et al., 2010).
Motor behavior may also be inappropriate. Some perform senseless, compulsive acts,
such as continually rocking or rubbing an arm. Others, who exhibit catatonia, may remain
motionless for hours and then become agitated.
As you can imagine, such disorganized thinking, disturbed perceptions, and inappropriate emotions profoundly disrupt social relationships and make it difficult to hold a job.
Even those with dissociative identity disorder, which we’ll discuss later in this unit, may continue to function in everyday life, but less so those with schizophrenia. During their most
severe periods, those with schizophrenia live in a private inner world, preoccupied with
illogical ideas and unreal images. Given a supportive environment and medication, over 40
percent of schizophrenia patients will have periods of a year or more of normal life experience (Jobe & Harrow, 2010). Many others remain socially withdrawn and isolated or rejected
throughout much of their lives (Hooley, 2010).
Onset and Development of Schizophrenia
68-2
Module 68
685
© Craig Geiser
TEACH
Common Pitfalls
Help students remember the difference between delusions and
hallucinations:
Art by someone diagnosed
with schizophrenia
Commenting on the kind of
artwork shown here (from Craig
Geiser’s 2010 art exhibit in
Michigan), poet and art critic
John Ashbery wrote: “The lure
of the work is strong, but so is
the terror of the unanswerable
riddles it proposes.”
A P ® E x a m Ti p
Delusions are false beliefs. These
are a dysfunction of our cognitive
systems.
Hallucinations are false
perceptions. These are a
dysfunction of our perceptual
systems.
ENGAGE
Are you clear about the
difference between delusions and
hallucinations? Delusions are false
thoughts. Hallucinations are false
sensory experiences.
Online Activities
Access a free computer simulation
available through the University
of California, Davis, that simulates
the hallucinations that people with
schizophrenia might experience.
Students will need to set up a free
account with Second Life, the online
virtual world: www.ucdmc.ucdavis.
edu/ais/virtualhallucinations/.
“When someone asks me to
explain schizophrenia I tell them,
you know how sometimes in your
dreams you are in them yourself
and some of them feel like real
nightmares? My schizophrenia
was like I was walking through a
dream. But everything around me
was real. At times, today’s world
seems so boring and I wonder
if I would like to step back into
the schizophrenic dream, but
then I remember all the scary and
horrifying experiences.” -STUART
EMMONS, WITH CRAIG GEISER, KALMAN
J. KAPLAN, AND MARTIN HARROW,
LIVING WITH SCHIZOPHRENIA, 1997
hallucination false sensory
experience, such as seeing
something in the absence of an
external visual stimulus.
How do chronic and acute schizophrenia differ?
Schizophrenia typically strikes as young people are maturing into adulthood. Although it
only afflicts 1 in 100 people, it knows no national boundaries, and it affects both males
and females—though men tend to be struck earlier, more severely, and slightly more often
(Aleman et al., 2003; Picchioni & Murray, 2007).
1/21/14 9:35 AM
TEACH
MyersAP_SE_2e_Mod68_B.indd 685
1/21/14 9:35 AM
Concept Connections
Extend the discussion of schizophrenia to
include autism. Social withdrawal is prominent
in both autism and schizophrenia. Autism is
diagnosed at an early age (always by age 3).
The diagnosis of schizophrenia occurs between
15 and 30 years. The incidence of schizophrenia in males and females is about equal, while
autism occurs mostly in males. James Kalat
identifies 9 behaviors of the autistic child:
1. Social isolation
2. Stereotyped behaviors, such as rocking
back and forth, biting hands, and staring
3. Resistance to any change in routine
4. Abnormal, often extreme, responses to
sensory stimuli
5. Insensitivity to pain
6. Inappropriate emotional expression
7. Disturbances of movement (i.e., abnormal
gait, rocking, grimacing, etc.)
8. Poor development of speech
9. Specific, limited intellectual
problems
Have students discuss the possible
links between schizophrenia and
autism. What behaviors are similar?
Different?
Kalat, J. (1998). Biological psychology
(9th ed.). Pacific Grove, CA: Brooks/Cole.
Schizophrenia
MyersPsyAP_TE_2e_U12.indd 685
Module 68
685
3/5/14 11:03 AM
686
Unit XII Abnormal Behavior
For some, schizophrenia will appear suddenly, seemingly as a reaction to stress. For
others, as was the case with Maxine, schizophrenia develops gradually, emerging from a
long history of social inadequacy and poor school performance (MacCabe et al., 2008). No
wonder those predisposed to schizophrenia often end up in the lower socioeconomic levels,
or even homeless.
We have thus far described schizophrenia as if it were a single disorder. Actually, it
varies. Schizophrenia patients with positive symptoms may experience hallucinations, talk
in disorganized and deluded ways, and exhibit inappropriate laughter, tears, or rage. Those
with negative symptoms have toneless voices, expressionless faces, or mute and rigid bodies.
Thus, positive symptoms are the presence of inappropriate behaviors, and negative symptoms are the absence of appropriate behaviors.
When schizophrenia is a slow-developing process (called chronic, or process, schizophrenia), recovery is doubtful (WHO, 1979). Those with chronic schizophrenia often exhibit the
persistent and incapacitating negative symptom of social withdrawal (Kirkpatrick et al.,
2006). Men, whose schizophrenia develops on average four years earlier than women’s,
more often exhibit negative symptoms and chronic schizophrenia (Räsänen et al., 2000).
When previously well-adjusted people develop schizophrenia rapidly (called acute, or reactive, schizophrenia) following particular life stresses, recovery is much more likely. They
more often have the positive symptoms that are responsive to drug therapy (Fenton &
McGlashan, 1991, 1994; Fowles, 1992).
ENGAGE
TR M
TRM
Enrichment
People with schizophrenia will exhibit
other striking symptoms of the
disorder:
Loose associations occur in
patients with disorganized
schizophrenia as they link events
and memories that don’t seem to
fit together logically.
Neologisms are words that patients
create, usually as part of the “word
salad” symptom. They will conjure
up words that make no logical
sense.
Understanding Schizophrenia
Use Teacher Demonstration: Magical Ideation Scale from the TRM to
learn more about other symptoms of
schizophrenia.
Schizophrenia is not only the most dreaded psychological disorder but also one of the
most heavily researched. Most of the new research studies link it with brain abnormalities
and genetic predispositions. Schizophrenia is a disease of the brain manifest in symptoms
of the mind.
Brain Abnormalities
TEACH
68-3
How do brain abnormalities and viral infections help explain
schizophrenia?
Common Pitfalls
Students may get the positive and
negative symptoms of schizophrenia confused. Remind them that
positive and negative in psychology
typically do not designate “good”
and “bad,” but rather “addition”
and “subtraction.”
© mikeledray/Shutterstock
DOPAMINE OVERACTIVITY
Positive symptoms refer to those
that are excessive or in addition
to normal behaviors. Outlandish
behavior such as paranoid
delusions, hallucinations, and
erratic emotions or behaviors are
typical of positive symptoms.
Negative symptoms refer to those
that are deficient or less than
normal behaviors. Flat affect, social
withdrawal, and catatonia are
common negative symptoms.
FYI
Most schizophrenia patients
smoke, often heavily. Nicotine
apparently stimulates certain
brain receptors, which helps
focus attention (Diaz et al., 2008;
Javitt & Coyle, 2004).
Researchers discovered one such key when they examined schizophrenia patients’ brains
after death and found an excess of receptors for dopamine—a sixfold excess for the so-called
D4 dopamine receptor (Seeman et al., 1993; Wong et al., 1986). They now speculate that
such a hyper-responsive dopamine system may intensify brain signals in schizophrenia,
creating positive symptoms such as hallucinations and paranoia (Grace, 2010). As we might
therefore expect, drugs that block dopamine receptors often lessen these symptoms; drugs
that increase dopamine levels, such as amphetamines and cocaine, sometimes intensify
them (Seeman, 2007; Swerdlow & Koob, 1987).
TEACH
MyersAP_SE_2e_Mod68_B.indd 686
Flip It
Students can get additional help understanding schizophrenia and its symptoms by watching the Flip It Video: Schizophrenia: Positive
and Negative Symptoms.
686
Unit XII
MyersPsyAP_TE_2e_U12.indd 686
Might imbalances in brain chemistry underlie schizophrenia? Scientists have long known that strange behavior can have strange
chemical causes. The saying “mad as a hatter” refers to the psychological deterioration of British hatmakers whose brains, it
was later discovered, were slowly poisoned as they moistened
the brims of mercury-laden felt hats with their tongue and lips
(Smith, 1983). As we saw in Module 25, scientists are clarifying
the mechanism by which chemicals such as LSD produce hallucinations. These discoveries hint that schizophrenia symptoms
might have a biochemical key.
TEACH
1/21/14 9:35 AM
Concept Connections
Link the discussion of dopamine to the Unit III
discussion of neurotransmitters and their functions. Dopamine is similar in chemical makeup
to cocaine, which explains why abusers of the
drug experience schizophrenia-like symptoms. Also linked to dopamine is Parkinson’s
disease, which is believed to result from a lack
of dopamine channels in the brain. Patients
who take medicine to treat schizophrenia will
develop Parkinson’s-like symptoms, such as
hand tremors.
Abnormal Behavior
3/5/14 11:03 AM
MyersAP_SE_2e
Schizophrenia
Module 68
687
Modern brain-scanning techniques reveal that many people with
chronic schizophrenia have abnormal activity in multiple brain areas.
Some have abnormally low brain activity in the frontal lobes, which
are critical for reasoning, planning, and problem solving (Morey et al.,
2005; Pettegrew et al., 1993; Resnick, 1992). People diagnosed with
schizophrenia also display a noticeable decline in the brain waves
that reflect synchronized neural firing in the frontal lobes (Spencer
et al., 2004; Symond et al., 2005). Out-of-sync neurons may disrupt
the integrated functioning of neural networks, possibly contributing
to schizophrenia symptoms.
One study took PET scans of brain activity while people were hallucinating (Silbersweig et al., 1995). When participants heard a voice or saw
something, their brain became vigorously active in several core regions,
including the thalamus, a structure deep in the brain that filters incoming
sensory signals and transmits them to the cortex. Another PET scan study
of people with paranoia found increased activity in the amygdala, a fearprocessing center (Epstein et al., 1998).
Many studies have found enlarged, fluid-filled areas and a corresponding shrinkage
and thinning of cerebral tissue in people with schizophrenia (Goldman et al., 2009; Wright
et al., 2000). Some studies have even found such abnormalities in the brains of people who
would later develop this disorder and in their close relatives (Karlsgodt et al., 2010). The
greater the brain shrinkage, the more severe the thought disorder (Collinson et al., 2003;
Nelson et al., 1998; Shenton, 1992). One smaller-than-normal area is the cortex. Another
is the corpus callosum connection between the two hemispheres (Arnone et al., 2008). Another is the thalamus, which may explain why people with schizophrenia have difficulty
filtering sensory input and focusing attention (Andreasen et al., 1994; Ellison-Wright et al.,
2008). The bottom line of various studies is that schizophrenia involves not one isolated
brain abnormality but problems with several brain regions and their interconnections (Andreasen, 1997, 2001).
Naturally, scientists wonder what causes these abnormalities. Some point to mishaps
during prenatal development or delivery (Fatemi & Folsom, 2009; Walker et al., 2010). Risk
factors for schizophrenia include low birth weight, maternal diabetes, older paternal age,
and oxygen deprivation during delivery (King et al., 2010). Famine may also increase risks.
People conceived during the peak of the Dutch wartime famine later displayed a doubled
rate of schizophrenia, as did those conceived during the famine that occurred from 1959 to
1961 in eastern China (St. Clair et al., 2005; Susser et al., 1996).
Chris Mueller/Redux
ABNORMAL BRAIN ACTIVITY AND ANATOMY
1/21/14 9:35 AM
Are people at increased risk of schizophrenia if, during the middle of their fetal development,
their country experienced a flu epidemic? The repeated answer is Yes (Mednick et al.,
1994; Murray et al., 1992; Wright et al., 1995).
•
Are people born in densely populated areas, where viral diseases spread more readily, at
greater risk for schizophrenia? The answer, confirmed in a study of 1.75 million Danes, is
Yes (Jablensky, 1999; Mortensen, 1999).
•
Are those born during the winter and spring months—after the fall-winter flu season—also
at increased risk? Although the increase is small, just 5 to 8 percent, the answer is again
Yes (Fox, 2010; Torrey et al., 1997, 2002).
MyersAP_SE_2e_Mod68_B.indd 687
Enrichment
Recent research using brain imaging
techniques has shed some light on
the biological basis of schizophrenia.
People who develop schizophrenia
experience abnormal brain activity
before the onset of symptoms,
suggesting that schizophrenia may
be a developmental disorder.
MRI studies show that gray matter
in the brains of people with
schizophrenia is markedly less
dense than that in people without
schizophrenia.
Studies have also shown that
people who have auditory
hallucinations experience temporal
lobe activation, indicating that
they really are hearing voices, even
though the voices are not present.
Studying the neurophysiology
of schizophrenia Psychiatrist E.
Fuller Torrey has collected the brains of
hundreds of those who died as young
adults and suffered disorders such as
schizophrenia and bipolar disorder.
TEACH
Common Pitfalls
Remind students that a genetic predisposition does not guarantee one
will get a disorder such as schizophrenia. Environmental and behavioral
influences help determine whether
someone with a predisposition will
develop the disorder.
MATERNAL VIRUS DURING MIDPREGNANCY
Consider another possible culprit: a midpregnancy viral infection that impairs fetal brain
development (Patterson, 2007). Can you imagine some ways to test this fetal-virus idea?
Scientists have asked the following:
•
ENGAGE
1/21/14 9:35 AM
Schizophrenia
MyersPsyAP_TE_2e_U12.indd 687
Module 68
687
3/5/14 11:04 AM
688
Unit XII Abnormal Behavior
•
In the Southern Hemisphere, where the seasons are the reverse of the Northern Hemisphere,
are the months of above-average schizophrenia births similarly reversed? Again, the answer
is Yes, though somewhat less so. In Australia, for example, people born between
August and October are at greater risk—unless they migrated from the Northern
Hemisphere, in which case their risk is greater if they were born between January and
March (McGrath et al., 1995, 1999).
•
Are mothers who report being sick with influenza during pregnancy more likely to bear
children who develop schizophrenia? In one study of nearly 8000 women, the answer
was Yes. The schizophrenia risk increased from the customary 1 percent to about 2
percent—but only when infections occurred during the second trimester (Brown et al.,
2000). Maternal influenza infection during pregnancy also affects brain development
in monkeys (Short et al., 2010).
•
Does blood drawn from pregnant women whose offspring develop schizophrenia show
higher-than-normal levels of antibodies that suggest a viral infection? In one study of 27
women whose children later developed schizophrenia, the answer was Yes (Buka et al.,
2001). And the answer was again Yes in a huge California study, which collected blood
samples from some 20,000 pregnant women during the 1950s and 1960s (Brown et
al., 2004). Another study found traces of a specific retrovirus (HERV) in nearly half of
people with schizophrenia and virtually none in healthy people (Perron et al., 2008).
These converging lines of evidence suggest that fetal-virus infections play a contributing role in the development of schizophrenia. They also strengthen the recommendation
that “women who will be more than three months pregnant during the flu season” have a
flu shot (CDC, 2003).
Why might a second-trimester maternal flu bout put fetuses at risk? Is it the virus itself? The mother’s immune response to it? Medications taken (Wyatt et al., 2001)? Does the
infection weaken the brain’s supportive glial cells, leading to reduced synaptic connections
(Moises et al., 2002)? In time, answers may become available.
Genetic Factors
68-4
Figure 68.1
Risk of developing
schizophrenia The lifetime risk of
Are there genetic influences on schizophrenia? What factors may be
early warning signs of schizophrenia in children?
Fetal-virus infections do appear to increase the odds that a child will develop schizophrenia. But this theory cannot tell us why only 2 percent of women who catch the flu during
their second trimester of pregnancy bear children who develop schizophrenia. Might people
also inherit a predisposition to this disorder? The evidence strongly suggests that, Yes, some
do. The nearly 1-in-100 odds of any person’s being diagnosed with schizophrenia become
about 1 in 10 among those whose sibling or parent has the disorder, and close to 1 in 2 if the
affected sibling is an identical twin (FIGURE 68.1). Although only a dozen or so such cases
are on record, the co-twin of an identical twin with
schizophrenia retains that 1-in-2 chance even when
Fraternal twins
the twins are reared apart (Plomin et al., 1997).
Identical twins
Remember, though, that identical twins also share
a prenatal environment. About two-thirds also share
a placenta and the blood it supplies; the other onethird have two single placentas. If an identical twin has
schizophrenia, the co-twin’s chances of being similarly
afflicted are 6 in 10 if they shared a placenta. If they had
separate placentas, as do fraternal twins, the chances
Japan
Denmark Finland Germany
U.K.
are only 1 in 10 (Davis et al., 1995a,b; Phelps et al.,
(1996)
(1996)
(1998)
(1998)
(1999)
1997). Twins who share a placenta are more likely to
developing schizophrenia varies with
one’s genetic relatedness to someone
having this disorder. Across countries,
barely more than 1 in 10 fraternal twins,
but some 5 in 10 identical twins, share
a schizophrenia diagnosis. (Adapted
from Gottesman, 2001.)
Schizophrenia risk
for twins of those
diagnosed
with schizophrenia
TEACH
70%
Common Pitfalls
60
50
40
30
Students might be confused to learn
that identical twins may not share a
placenta. Identical twins are defined
by their shared DNA, not by a shared
placenta.
20
10
0
MyersAP_SE_2e_Mod68_B.indd 688
ENGAGE
1/21/14 9:35 AM
Active Learning
Have students examine the heritability of other
mental illnesses. They can find statistics on
how likely a child is to develop a disorder that
his or her parent has. Consider the following
conditions:
688
Unit XII
MyersPsyAP_TE_2e_U12.indd 688
Depression and bipolar disorder
Anxiety disorders
Somatoform disorders
Substance-related disorders
Personality disorders
Abnormal Behavior
3/5/14 11:04 AM
MyersAP_SE_2e
Module 68
689
experience the same prenatal viruses. So it is possible that shared germs as well as shared
genes produce identical twin similarities.
Adoption studies, however, confirm that the genetic link is real (Gottesman, 1991). Children adopted by someone who develops schizophrenia seldom “catch” the disorder. Rather,
adopted children have an elevated risk if a biological parent is diagnosed with schizophrenia.
With the genetic factor established, researchers are now sleuthing specific genes that, in
some combination, might predispose schizophrenia-inducing brain abnormalities (Levinson
et al., 2011; Mitchell & Porteous, 2011; Vacic et al., 2011; Wang et al., 2010). (It is not our genes
but our brains that directly control our behavior.) Some of these genes influence the effects of
dopamine and other neurotransmitters in the brain. Others affect the production of myelin,
a fatty substance that coats the axons of nerve cells and lets impulses travel at high speed
Schizophrenia in identical twins
through neural networks.
When twins differ, only the one afflicted
Although the genetic contribution to schizophrenia is beyond question, the genetic forwith schizophrenia typically has
mula is not as straightforward as the inheritance of eye color. Genome studies of thousands
enlarged, fluid-filled cranial cavities
(right) (Suddath et al., 1990). The
of individuals with and without schizophrenia indicate that schizophrenia is influenced by
difference between the twins implies
many genes, each with very small effects (International Schizophrenia Consortium, 2009;
some nongenetic factor, such as a
Pogue-Geile & Yokley, 2010). Recall from Module 14 that epigenetic (literally “in addition
virus, is also at work.
to genetic”) factors influence gene expression. Like hot water activating the tea bag, environmental factors such as prenatal viral infections,
nutritional deprivation, and maternal stress can “turn on” the genes
that predispose schizophrenia. Identical twins’ differing histories in the
womb and beyond explain why only one of them may show differing
gene expressions (Walker et al., 2010). As we have so often seen, nature and nurture interact. Neither hand claps alone.
Thanks to our expanding understanding of genetic and brain influences on maladies such as schizophrenia, the general public more
and more attributes psychiatric disorders to biological factors (Pescosolido et al., 2010). In 2007, one privately funded new research center
No schizophrenia
Schizophrenia
announced its ambitious aim: “To unambiguously diagnose patients
with psychiatric disorders based on their DNA sequence in 10 years’ time” (Holden, 2007).
In 2010, $120 million in start-up funding launched a bold new effort to study the neuroscience and genetics of schizophrenia and other psychiatric disorders (Kaiser, 2010). So, can
scientists develop genetic tests that reveal who is at risk? If so, will people in the future subject their embryos to genetic testing (and gene repair or abortion) if they are at risk for this
FYI
or some other psychological or physical malady? Might they take their egg and sperm to a
The odds of any four people
genetics lab for screening before combining them to produce an embryo? Or will children
picked at random all being
be tested for genetic risks and given appropriate preventive treatments? In this brave new
diagnosed with schizophrenia are
twenty-first-century world, such questions await answers.
1 in 100 million. But genetically
Psychological Factors
If prenatal viruses and genetic predispositions do not, by themselves, cause schizophrenia,
neither do family or social factors alone. It remains true, as Susan Nicol and Irving Gottesman (1983) noted almost three decades ago, that “no environmental causes have been
discovered that will invariably, or even with moderate probability, produce schizophrenia in
persons who are not related to” a person with schizophrenia.
Hoping to identify environmental triggers of schizophrenia, several investigators are
following the development of “high-risk” children, such as those born to a parent with
schizophrenia or exposed to prenatal risks (Freedman et al., 1998; Olin & Mednick, 1996;
Susser, 1999). One study followed 163 teens and early-twenties adults who had two relatives with schizophrenia. During the 2.5-year study, the 20 percent who developed schizophrenia displayed some tendency to withdraw socially and behave oddly before the onset of
1/21/14 9:35 AM
MyersAP_SE_2e_Mod68_B.indd 689
ENGAGE
Critical Questions
Have students discuss the implications of genetic testing for mental
illness.
Both photos: From Daniel Weinberger, M.D., CBDB, NIMH
Schizophrenia
identical sisters Nora, Iris, Myra,
and Hester Genain all have the
disease. Two of the sisters have
more severe forms of the disorder
than the others, suggesting the
influence of environmental as well
as biological factors.
If a genetic test were available,
would you take it? Why or why
not?
If you tested positive for a mental
illness, what actions would you
want to take?
What would the implications be for
insurance companies? How might
they react?
TEACH
Enrichment
The first and last names of the
quadruplets featured here in the FYI
box were changed to protect their
privacy as they became recurrent
research subjects. They share the
unique distinction of being the only
set of multiples who share this mental
illness. All became schizophrenic by
age 25, but in addition to sharing
the same genetic code, they shared
a nightmarish family life, suffering
abuse at the hands of their father and
ambivalence from their mother. The
least ill of the 4, Myra, seemed to be
her mother’s favorite and managed
to avoid some of her father’s abuse,
reinforcing the idea of the interplay
between heredity and environment.
1/21/14 9:35 AM
Schizophrenia
MyersPsyAP_TE_2e_U12.indd 689
Module 68
689
3/5/14 11:04 AM
690
Unit XII Abnormal Behavior
the disorder (Johnstone et al., 2005). By comparing the experiences of high-risk and low-risk
children who do versus do not develop schizophrenia, researchers have so far pinpointed
the following possible early warning signs:
CLOSE & ASSESS
Exit Assessment
Have students list the symptoms,
major diagnostic criteria, and possible
genetic factors for schizophrenia.
•
A mother whose schizophrenia was severe and long-lasting
•
Birth complications, often involving oxygen deprivation and low birth weight
•
Separation from parents
•
Short attention span and poor muscle coordination
•
Disruptive or withdrawn behavior
•
Emotional unpredictability
•
Poor peer relations and solo play
***
Most of us can relate more easily to the ups and downs of mood disorders than to the
strange thoughts, perceptions, and behaviors of schizophrenia. Sometimes our thoughts do
jump around, but in the absence of disorder we do not talk nonsensically. Occasionally we
feel unjustly suspicious of someone, but we do not fear that the world is plotting against us.
Often our perceptions err, but rarely do we see or hear things that are not there. We have
felt regret after laughing at someone’s misfortune, but we rarely giggle in response to bad
news. At times we just want to be alone, but we do not live in social isolation. However, millions of people around the world do talk strangely, suffer delusions, hear nonexistent voices,
see things that are not there, laugh or cry at inappropriate times, or withdraw into private
imaginary worlds. The quest to solve the cruel puzzle of schizophrenia therefore continues,
and more vigorously than ever.
Before You Move On
䉴 ASK YOURSELF
Do you think the media accurately portray the behavior of people suffering from
schizophrenia? Why or why not?
䉴 TEST YOURSELF
How do researchers believe that biological and environmental factors interact in the onset of
schizophrenia?
Answers to the Test Yourself questions can be found in Appendix E at the end of the book.
MyersAP_SE_2e_Mod68_B.indd 690
690
Unit XII
MyersPsyAP_TE_2e_U12.indd 690
1/21/14 9:35 AM
Abnormal Behavior
3/5/14 11:04 AM
MyersAP_SE_2e
Schizophrenia
Module 68
691
Module 68 Review
68-1
•
Schizophrenia is a disorder that typically strikes during late
adolescence, affects men slightly more than women, and
seems to occur in all cultures.
68-4
Are there genetic influences on
schizophrenia? What factors may be early
warning signs of schizophrenia in children?
•
Twin and adoption studies indicate that the predisposition
to schizophrenia is inherited, and environmental factors
influence gene expression to enable this disorder, which is
found worldwide.
•
Symptoms are disorganized and delusional thinking,
disturbed perceptions, and diminished or inappropriate
emotions.
•
•
Delusions are false beliefs; hallucinations are sensory
experiences without sensory stimulation.
No environmental causes invariably produce
schizophrenia.
•
Possible early warning signs of later development of
schizophrenia include both biological factors (a mother
with severe and long-lasting schizophrenia; oxygen
deprivation and low weight at birth; short attention span
and poor muscle coordination) as well as psychological
factors (disruptive or withdrawn behavior; emotional
unpredictability; poor peer relations and solo play).
68-2
How do chronic and acute schizophrenia
differ?
•
Schizophrenia symptoms may be positive (the presence
of inappropriate behaviors) or negative (the absence of
appropriate behaviors).
•
In chronic (or process) schizophrenia, the disorder
develops gradually and recovery is doubtful.
•
In acute (or reactive) schizophrenia, the onset is sudden,
in reaction to stress, and the prospects for recovery are
brighter.
68-3
1/21/14 9:35 AM
What patterns of thinking, perceiving, and
feeling characterize schizophrenia?
How do brain abnormalities and viral
infections help explain schizophrenia?
•
People with schizophrenia have increased dopamine
receptors, which may intensify brain signals, creating
positive symptoms such as hallucinations and paranoia.
•
Brain abnormalities associated with schizophrenia include
enlarged, fluid-filled cerebral cavities and corresponding
decreases in the cortex.
•
Brain scans reveal abnormal activity in the frontal lobes,
thalamus, and amygdala.
•
Interacting malfunctions in multiple brain regions and
their connections may produce schizophrenia’s symptoms.
•
Possible contributing factors include viral infections or
famine conditions during the mother’s pregnancy and low
weight or oxygen deprivation at birth.
MyersAP_SE_2e_Mod68_B.indd 691
1/21/14 9:35 AM
Schizophrenia
MyersPsyAP_TE_2e_U12.indd 691
Module 68
691
3/5/14 11:04 AM
Unit XII Abnormal Behavior
692
Answers to Multiple-Choice
Questions
1. c
2. c
Multiple-Choice Questions
1. Which of the following is the best term or phrase for a
false belief, often of persecution, that may accompany
psychotic disorders?
3. b
a.
b.
c.
d.
e.
Psychosis
Schizophrenia
Delusion
Split mind
Dissociative identity disorder
3. According to research, which of the following has been
identified as an early warning sign of schizophrenia?
a.
b.
c.
d.
e.
Emotional predictability
Poor peer relations and solo play
Long attention span
Good muscle coordination
High birth weight
2. Which of the following is true?
a. Those born during winter and spring are less likely to
develop schizophrenia later in life.
b. People born in densely populated areas are less likely
to develop schizophrenia later in life.
c. Fetuses exposed to flu virus are more likely to
develop schizophrenia later in life.
d. Maternal influenza during pregnancy does not affect
brain development in monkeys.
e. The retrovirus HERV is found more often in people
who do not develop schizophrenia.
Answer to Practice FRQ 2
Practice FRQs
1. Name three possible warning signs of schizophrenia.
1 point: Dopamine overactivity.
1 point: The brains of those with
schizophrenia may have 6 times the
number of dopamine receptors than
normal.
Answer
• A mother whose schizophrenia was severe and
long-lasting
• Birth complications, often involving oxygen
deprivation and low weight
• Separation from parents
• Short attention span and poor muscle coordination
1 point: Abnormal brain activity
includes, but is not limited to:
low frontal lobe brain wave activity.
less synchronized neural firing in
the frontal lobes.
vigorous thalamus activity.
increased activity in the amygdala.
thinning cerebral tissue.
• Disruptive or withdrawn behavior
• Emotional unpredictability
• Poor peer relations and solo play
MyersAP_SE_2e_Mod68_B.indd 692
692
Unit XII
MyersPsyAP_TE_2e_U12.indd 692
understand schizophrenia.
(4 points)
Score 1 point for any of the following (up to 3) possibilities.
1 point: Abnormal brain activity.
2. Name and explain two brain abnormalities that help us
1/21/14 9:35 AM
Abnormal Behavior
3/5/14 11:04 AM
MyersAP_SE_2e
Other Disorders
Module 69
693
TEACH
Module 69
TR M
TRM
Other Disorders
TEACH
Module Learning Objectives
69-1
Describe somatic symptom and related disorders.
69-2
Describe dissociative disorders, and discuss why they are
controversial.
69-3
68-4
Explain how anorexia nervosa, bulimia nervosa, and binge-eating
disorder demonstrate the influence of psychological and genetic
forces.
Enrichment
H
ENTIET
FOX/TW
C20TH Collection
tt
Evans/
Mary Grant/Evere
Ronald
Somatoform disorders are not as
commonly diagnosed as they once
were. The revisions to DSM-5 highlight
some of the reasons why:
X/
RY FO
CENTU
Contrast the three clusters of personality disorders, and describe
the behaviors and brain activity that characterize the antisocial
personality.
Somatic Symptom and Related Disorders
69-1
1/21/14 9:35 AM
The interplay of psychological and
medical symptoms and diagnoses
The elimination of medically
unexplained symptoms as criteria
for diagnosing a somatoform
disorder
The change in terminology, from
hypochondriasis (a term that
has come to have a negative
connotation) to illness anxiety
disorder
What are somatic symptom and related disorders?
Among the most common problems bringing people into doctors’ offices are “medically
unexplained illnesses” (Johnson, 2008). Ellen becomes dizzy and nauseated in the late afternoon—shortly before she expects her husband home. Neither her primary care physician nor the neurologist he sent her to could identify a physical cause. They suspect her
symptoms have an unconscious psychological origin, possibly triggered by her mixed feelings about her husband. In a somatic symptom disorder such as Ellen’s, the distressing
symptoms take a somatic (bodily) form without apparent physical causes. One person may
have a variety of complaints—vomiting, dizziness, blurred vision, difficulty in swallowing.
Another may experience severe and prolonged pain.
Culture has a big effect on people’s physical complaints and how they explain them
(Kirmayer & Sartorius, 2007). In China, psychological explanations of anxiety and depression are socially less acceptable than in many Western countries, and people less often express the emotional aspects of distress. The Chinese appear more sensitive to—and more
willing to report—the physical symptoms of their distress (Ryder et al., 2008). Mr. Wu, a
36-year-old technician in Hunan, illustrates one of China’s most common psychological
disorders (Spitzer & Skodol, 2000). He finds work difficult because of his insomnia, fatigue,
weakness, and headaches. Chinese herbs and Western medicines provide no relief. To his
Chinese clinician, who treats the bodily symptoms, he seems not so much depressed as
exhausted. Similar, generalized bodily complaints have often been observed in African cultures (Binitie, 1975).
MyersAP_SE_2e_Mod69_B.indd 693
somatic symptom disorder a
psychological disorder in which the
symptoms take a somatic (bodily)
form without apparent physical
cause. (See conversion disorder and
illness anxiety disorder.)
TEACH
Concept Connections
One contributor to somatoform
illnesses may be stress. Remind students of the physical effects of stress
as discussed in Unit VIII. Some of these
symptoms of stress-related illness may
seem familiar as they study somatoform disorders.
1/21/14 9:35 AM
Other Disorders
MyersPsyAP_TE_2e_U12.indd 693
Discussion Starter
Use the Module 69 Fact or Falsehood?
activity from the TRM to introduce the
concepts from this module.
Module 69
693
3/5/14 11:04 AM
694
TEACH
Concept Connections
People with illness anxiety aren’t
faking sickness to get attention. They
truly believe they suffer from an illness
that doctors haven’t yet identified.
They frequently switch doctors, seeking a diagnosis that will confirm their
condition. On the other hand, people
who fake illness are malingering. With
this disorder, patients fake illness to
avoid trouble or to gain something.
Unit XII Abnormal Behavior
conversion disorder a disorder
in which a person experiences very
specific genuine physical symptoms
for which no physiological basis
can be found. (Also called functional
neurological symptom disorder.)
illness anxiety disorder
a disorder in which a person
interprets normal physical
sensations as symptoms of
a disease. (Formerly called
hypochondriasis.)
dissociative disorders disorders
in which conscious awareness
becomes separated (dissociated)
from previous memories, thoughts,
and feelings.
ENGAGE
Enrichment
Before You Move On
People with conversion disorder will
typically suffer problems with parts
of their bodies that directly relate to
the stress they are experiencing. For
example, a quarterback for a football
team might lose sensation in his
throwing hand before the big game.
Otherwise, he may report feeling fine
and not admit to experiencing stress
about the event. La belle indifference,
a French phrase meaning “beautiful
indifference,” is used to describe the
apathy these patients feel about their
condition.
䉴 ASK YOURSELF
Can you recall (as most people can) times when you have fretted needlessly over a normal
bodily sensation?
䉴 TEST YOURSELF
What does somatic mean?
Answers to the Test Yourself questions can be found in Appendix E at the end of the book.
Dissociative Disorders
69-2
Enrichment
Other forms of somatoform disorders
include the following:
Body dysmorphic disorder:
Preoccupation with defects in
one’s body. When faced with an
imperfection, concern about it
becomes excessive.
Pain disorder: Complaints of
severe pain without the presence
of any particular physical
condition; or malingering.
Somatization disorder: Patients
under 30 years of age will exhibit
a variety of unexplained physical
symptoms.
694
Unit XII
MyersPsyAP_TE_2e_U12.indd 694
What are dissociative disorders, and why are they controversial?
Among the most bewildering disorders are the rare dissociative disorders. These are
disorders of consciousness, in which a person appears to experience a sudden loss of memory or change in identity, often in response to an overwhelmingly stressful situation. Chris
Sizemore’s story, told in the book and movie The Three Faces of Eve, gave early visibility to
what is now called dissociative identity disorder. One Vietnam veteran who was haunted by
his comrades’ deaths, and who had left his World Trade Center office shortly before the 9/11
attack, disappeared en route to work one day and was discovered six months later in a Chicago homeless shelter, reportedly with no memory of his identity or family (Stone, 2006). In
such fugue state cases, the person’s conscious awareness is said to dissociate (become separated) from painful memories, thoughts, and feelings. (Note that this explanation presumes
the existence of repressed memories, which, as we noted in Modules 33 and 56, have been
questioned by memory researchers.)
Dissociation itself is not so rare. Now and then, many people may have a sense of
being unreal, of being separated from their body, of watching themselves as if in a movie.
ENGAGE
Even to people in the West, somatic symptoms are familiar. To a lesser extent, we have
all experienced inexplicable physical symptoms under stress. It is little comfort to be told
that the problem is “all in your head.” Although the symptoms may be psychological in
origin, they are nevertheless genuinely felt.
One rare type of disorder, more common in Freud’s day than in ours, is conversion
disorder (also known as functional neurological symptom disorder), so called because anxiety
presumably is converted into a physical symptom. (As we noted in Module 55, Freud’s effort to treat and understand psychological disorders stemmed from his puzzlement over
ailments that had no physiological basis.) A patient with a conversion disorder might, for
example, lose sensation in a way that makes no neurological sense. Yet the physical symptoms would be real; sticking pins in the affected area would produce no response. Other
conversion disorder symptoms might be unexplained paralysis, blindness, or an inability to
swallow. In each case, the person would be strangely indifferent to the problem.
As you can imagine, somatic symptom and related disorders send people not to a psychologist or psychiatrist but to a physician. This is especially true of those who experience
illness anxiety disorder (formerly called hypochondriasis). In this relatively common disorder, people interpret normal sensations (a stomach cramp today, a headache tomorrow)
as symptoms of a dreaded disease. Sympathy or temporary relief from everyday demands
may reinforce such complaints. No amount of reassurance by any physician convinces the
patient that the trivial symptoms do not reflect a serious illness. So the patient moves on to
another physician, seeking and receiving more medical attention—but failing to confront
the disorder’s psychological root.
TEACH
MyersAP_SE_2e_Mod69_B.indd 694
1/21/14 9:35 AM
Common Pitfalls
Students should understand that dissociation is not the same as psychosis. Dissociation
involves breaking away from the sense of
self, either losing one’s memory and identity
or adding personalities. Psychosis involves a
break with reality, believing things that are
untrue or having hallucinations about things
that don’t exist.
Abnormal Behavior
3/5/14 11:04 AM
MyersAP_SE_2e
Other Disorders
Module 69
695
AP/Wide World Photos
Sometimes we may say, “I was not myself at the time.” Perhaps you can recall getting
up to go somewhere and ending up at some unintended location while your mind was
preoccupied elsewhere. Or perhaps you can play a well-practiced tune on a guitar or
piano while talking to someone. Facing trauma, dissociative detachment may actually
protect a person from being overwhelmed by emotion.
Understanding Dissociative Identity Disorder
Skeptics question whether DID is a genuine disorder or an extension of our normal capacity for personality shifts. Nicholas Spanos (1986, 1994, 1996) asked college students to
pretend they were accused murderers being examined by a psychiatrist. Given the same
hypnotic treatment Bianchi received, most spontaneously expressed a second personality.
This discovery made Spanos wonder: Are dissociative identities simply a more extreme
version of our capacity to vary the “selves” we present—as when we display a goofy,
loud self while hanging out with friends, and a subdued, respectful self around grandparents? Are clinicians who discover multiple personalities merely triggering role-playing
by fantasy-prone people? Do these patients, like actors who commonly report “losing
themselves” in their roles, then convince themselves of the authenticity of their own role
enactments? Spanos was no stranger to this line of thinking. In a related research area, he
had also raised these questions about the hypnotic state. Given that most DID patients
are highly hypnotizable, whatever explains one condition—dissociation or role playing—
may help explain the other.
Skeptics also find it suspicious that the disorder is so localized in time and space. Between 1930 and 1960, the number of DID diagnoses in North America was 2 per decade. In
the 1980s, when the DSM contained the first formal code for this disorder, the number of
reported cases had exploded to more than 20,000 (McHugh, 1995a). The average number of
displayed personalities also mushroomed—from 3 to 12 per patient (Goff & Simms, 1993).
Outside North America, the disorder is much less prevalent, although in other cultures some
people are said to be “possessed” by an alien spirit (Aldridge-Morris, 1989; Kluft, 1991). In
Britain, DID—which some have considered “a wacky American fad” (Cohen, 1995)—is rare.
In India and Japan, it is essentially nonexistent (or at least unreported).
1/21/14 9:35 AM
MyersAP_SE_2e_Mod69_B.indd 695
Enrichment
Dissociative identity disorder (DID) is
so controversial partly because the
claims of patients are so difficult to
confirm. Typically, DID is not diagnosed until later in life when alleged
perpetrators might have died or
evidence of past abuse or trauma has
been lost.
Dissociative Identity Disorder
A massive dissociation of self from ordinary consciousness characterizes those with
dissociative identity disorder (DID), in which two or more distinct identities are
said to alternately control the person’s behavior. Each personality has its own voice and
mannerisms. Thus the person may be prim and proper one moment, loud and flirtatious
the next. Typically, the original personality denies any awareness of the other(s).
People diagnosed with DID (formerly called multiple personality disorder) are usually
not violent, but cases have been reported of dissociations into a “good” and a “bad” (or
aggressive) personality—a modest version of the Dr. Jekyll/Mr. Hyde split immortalized
in Robert Louis Stevenson’s story. One unusual case involved Kenneth Bianchi, accused
in the “Hillside Strangler” rapes and murders of 10 California women. During a hypnosis
session with Bianchi, psychologist John Watkins (1984) “called forth” a hidden personality:
“I’ve talked a bit to Ken, but I think that perhaps there might be another part of Ken that . . .
maybe feels somewhat differently from the part that I’ve talked to. . . . Would you talk with me,
Part, by saying, ‘I’m here’?” Bianchi answered “Yes” and then claimed to be “Steve.”
Speaking as Steve, Bianchi stated that he hated Ken because Ken was nice and that he
(Steve), aided by a cousin, had murdered women. He also claimed Ken knew nothing about
Steve’s existence and was innocent of the murders. Was Bianchi’s second personality a ruse,
simply a way of disavowing responsibility for his actions? Indeed, Bianchi—a practiced liar
who had read about multiple personality in psychology books—was later convicted.
ENGAGE
The “Hillside Strangler” Kenneth
Bianchi is shown here at his trial.
dissociative identity disorder
(DID) a rare dissociative disorder
in which a person exhibits two
or more distinct and alternating
personalities. Formerly called
multiple personality disorder.
Does such difficulty in confirming
trauma necessarily refute the
claims of patients?
How do investigators prove claims
of abuse that happened long ago?
TEACH
TR M
TRM
At this point, students should also
learn about another dissociative disorder: dissociative amnesia, whose
patients suffer a complete loss of identity. As a result of trauma, they forget
who they are. They may also experience dissociative fugue, a condition
that leads them to travel away from
home, often turning up as a “John
Doe” or “Jane Doe” in a distant community. Note that dissociative fugue
is no longer a separate diagnosis in
the new DSM-5. It is now a diagnostic
criterion for dissociative amnesia.
Use Student Activity: Questionnaire of Experiences of Dissociation
from the TRM to help students further
explore this topic.
“Pretense may become reality.”
-CHINESE PROVERB
1/21/14 9:35 AM
Other Disorders
MyersPsyAP_TE_2e_U12.indd 695
Teaching Tip
Module 69
695
3/5/14 11:04 AM
TEACH
Common Pitfalls
If you show any of the popular films
that depict dissociative identity
disorder, help students watch with a
critical eye. The goal of movies is not
to depict reality, but to entertain and
tell a compelling story. The movie may
or may not be an accurate depiction
of what happened.
Unit XII Abnormal Behavior
© The New Yorker Collection, 2001, Leo Cullum from
cartoonbank.com. All Rights Reserved.
696
“ Would it be possible to speak with
the personality that pays the bills?”
ENGAGE
Critical Questions
“Though this be madness, yet
there is method in ’t.” -WILLIAM
SHAKESPEARE, HAMLET, 1600
Women’s health issues have only
recently begun to be distinguished
from men’s health issues. Contemporaries of Freud believed that women
who attended college suffered from
shrunken ovaries since they did not
bear as many children as their lesseducated sisters. In the past, mental
illnesses such as dissociative and
somatoform disorders were often only
reported in women. Have students
explore old stereotypes and new
advances in women’s health:
What were some other common
beliefs about women’s health
issues?
What kind of influence did Freud
and others have on the study of
women’s health?
Such findings, skeptics say, point to a cultural phenomenon—a disorder created
by therapists in a particular social context (Merskey, 1992). Rather than being provoked
by trauma, dissociative symptoms tend to be exhibited by suggestible, fantasy-prone
people (Giesbrecht et al., 2008, 2010). Patients do not enter therapy saying “Allow
me to introduce myselves.” Rather, note these skeptics, some therapists go fishing for
multiple personalities: “Have you ever felt like another part of you does things you
can’t control? Does this part of you have a name? Can I talk to the angry part of you?”
Once patients permit a therapist to talk, by name, “to the part of you that says those
angry things,” they begin acting out the fantasy. Like actors who lose themselves in
their roles, vulnerable patients may “become” the parts they are acting out. The result may
be the experience of another self.
Other researchers and clinicians believe DID is a real disorder. They find support for this
view in the distinct brain and body states associated with differing personalities (Putnam,
1991). Handedness, for example, sometimes switches with personality (Henninger, 1992).
Ophthalmologists have detected shifting visual acuity and eye-muscle balance as patients
switched personalities, changes that did not occur among control group members trying to
simulate DID (Miller et al., 1991). Dissociative disorder patients also have exhibited heightened activity in brain areas associated with the control and inhibition of traumatic memories
(Elzinga et al., 2007).
Researchers and clinicians have interpreted DID symptoms from psychodynamic and
learning perspectives. Both views agree that the symptoms are ways of dealing with anxiety.
Psychodynamic theorists see them as defenses against the anxiety caused by the eruption
of unacceptable impulses; a wanton second personality enables the discharge of forbidden
impulses. Learning theorists see dissociative disorders as behaviors reinforced by anxiety
reduction.
Other clinicians include dissociative disorders under the umbrella of posttraumatic
stress disorder—a natural, protective response to “histories of childhood trauma” (Putnam,
1995; Spiegel, 2008). Many DID patients recall suffering physical, sexual, or emotional abuse
as children (Gleaves, 1996; Lilienfeld et al., 1999). In one study of 12 murderers diagnosed
with DID, 11 had suffered severe, torturous child abuse (Lewis et al., 1997). One was set afire
by his parents. Another was used in child pornography and was scarred from being made to
sit on a stove burner. Some critics wonder, however, whether vivid imagination or therapist
suggestion contributes to such recollections (Kihlstrom, 2005).
So the debate continues. On one side are those who believe multiple personalities are
the desperate efforts of the traumatized to detach from a horrific existence. On the other are
the skeptics who think DID is a condition contrived by fantasy-prone, emotionally vulnerable people, and constructed out of the therapist-patient interaction. If the skeptics’ view
wins, predicted psychiatrist Paul McHugh (1995b), “this epidemic will end in the way that
the witch craze ended in Salem. The [multiple personality phenomenon] will be seen as
manufactured.”
Before You Move On
䉴 ASK YOURSELF
In a more normal way, do you ever flip between displays of different aspects of your
personality?
䉴 TEST YOURSELF
When did women’s health issues
start to garner serious attention as
being distinct from men’s health
issues?
The psychodynamic and learning perspectives agree that dissociative identity disorder
symptoms are ways of dealing with anxiety. How do their explanations differ?
Answers to the Test Yourself questions can be found in Appendix E at the end of the book.
What advances have been
made in recent years allowing
women’s health issues to be better
understood?
TEACH
MyersAP_SE_2e_Mod69_B.indd 696
1/21/14 9:35 AM
Concept Connections
Remind students of Elizabeth Loftus’ work
questioning the validity of repressed memories (Unit VII). Many DID patients experience
memory loss with at least one alternate personality. Have your students consider whether
this type of memory loss can be related to
research done by Loftus and others.
696
Unit XII
MyersPsyAP_TE_2e_U12.indd 696
Abnormal Behavior
3/5/14 11:04 AM
MyersAP_SE_2e
Other Disorders
Module 69
697
69-3
Jeff Kravitz/FilmMagic for MTV/Getty Images
Eating Disorders
How do anorexia nervosa, bulimia nervosa, and binge-eating disorder
demonstrate the influence of psychological and genetic forces?
Our bodies are naturally disposed to maintain a steady weight, including stored energy
reserves for times when food becomes unavailable. Yet sometimes psychological influences
overwhelm biological wisdom. This becomes painfully clear in three eating disorders.
•
•
•
Anorexia nervosa typically begins as a weight-loss diet. People with anorexia—
usually adolescents and 9 times out of 10 females—drop significantly below normal
weight. Yet they feel fat, fear being fat, and remain obsessed with losing weight, and
sometimes exercise excessively. About half of those with anorexia display a bingepurge-depression cycle.
Bulimia nervosa may also be triggered by a weight-loss diet, broken by gorging
on forbidden foods. Binge-purge eaters—mostly women in their late teens or early
twenties—eat in spurts, sometimes influenced by friends who are bingeing (Crandall,
1988). In a cycle of repeating episodes, overeating is followed by compensatory
purging (through vomiting or laxative use), fasting, or excessive exercise (Wonderlich
et al., 2007). Preoccupied with food (craving sweet and high-fat foods), and fearful of
becoming overweight, binge-purge eaters experience bouts of depression and anxiety
during and following binges (Hinz & Williamson, 1987; Johnson et al., 2002). Unlike
anorexia, bulimia is marked by weight fluctuations within or above normal ranges,
making the condition easy to hide.
Anorexia nervosa is characterized
by significant weight loss. Usually,
patients must be at or below 85
percent of “normal body weight”
to be diagnosed with this disorder.
Patients with anorexia often do
eat food, but in small quantities or
with low nutritive value. If patients
do eat regular portions of food,
they will often overexercise to
eliminate the calories.
Patients with bulimia nervosa
often stay about 5–10 percent
above “normal body weight.”
Often, patients experience severe
gastrointestinal issues, especially
with the stomach and esophagus,
which can develop ulcers and tears
from repeated vomiting. In order
to be diagnosed with bulimia,
patients must purge somehow—
with laxatives, exercise, or
vomiting—which distinguishes
them from patients with bingeeating disorder.
Dying to be thin Anorexia was
identified and named in the 1870s,
when it appeared among affluent
adolescent girls (Brumberg, 2000).
Many modern-day celebrities, including
Lady Gaga, have struggled publicly
with eating disorders.
•
Mothers of girls with eating disorders tend to focus on their own weight and on their
daughters’ weight and appearance (Pike & Rodin, 1991).
•
Families of bulimia patients have a higher-than-usual incidence of childhood obesity
and negative self-evaluation (Jacobi et al., 2004).
•
Families of anorexia patients tend to be competitive, high-achieving, and protective
(Pate et al., 1992; Yates, 1989, 1990).
Those with eating disorders often have low self-evaluations, set perfectionist standards, fret about falling short of expectations, and are intensely concerned with how others perceive them (Pieters et al., 2007; Polivy & Herman, 2002; Sherry & Hall, 2009). Some
of these factors also predict teen boys’ pursuit of unrealistic muscularity (Ricciardelli &
McCabe, 2004).
Genetics also influence susceptibility to eating disorders. Twins are more likely to share
the disorder if they are identical rather than fraternal (Culbert et al., 2009; Klump et al., 2009;
Root et al., 2010). Scientists are now searching for culprit genes, which may influence the
body’s available serotonin and estrogen (Klump & Culbert, 2007).
But these disorders also have cultural and gender components. Ideal shapes vary
across culture and time. In impoverished areas of the world, including much of Africa—
where plumpness means prosperity and thinness can signal poverty or illness—bigger
MyersAP_SE_2e_Mod69_B.indd 697
anorexia nervosa an eating
disorder in which a person (usually
an adolescent female) maintains
a starvation diet despite being
significantly (15 percent or more)
underweight.
bulimia nervosa an eating
disorder in which a person
alternates binge eating (usually of
high-calorie foods) with purging (by
vomiting or laxative use), excessive
exercise, or fasting.
binge-eating disorder significant
binge-eating episodes, followed by
distress, disgust, or guilt, but without
the compensatory purging or fasting
that marks bulimia nervosa.
Use Student Activity: A Survey of
Eating Habits or Student Activity: A
Week of Food from the TRM to further
explore eating habits and disorders.
1/21/14 9:35 AM
Other Disorders
MyersPsyAP_TE_2e_U12.indd 697
Common Pitfalls
Students frequently have preconceptions about how eating disorders
work. Help them understand the
nuances of these disorders:
Those who do significant binge eating, followed by remorse—but do not purge, fast,
or exercise excessively—are said to have binge-eating disorder.
A national study funded by the U.S. National Institute of Mental Health reported that,
at some point during their lifetime, 0.6 percent of people meet the criteria for anorexia, 1
percent for bulimia, and 2.8 percent for binge-eating disorder (Hudson et al., 2007). So, how
can we explain these disorders?
Eating disorders do not provide (as some have speculated) a telltale sign of childhood
sexual abuse (Smolak & Murnen, 2002; Stice, 2002). The family environment may provide a
fertile ground for the growth of eating disorders in other ways, however.
1/21/14 9:35 AM
TEACH
TR M
TRM
Module 69
697
3/5/14 11:04 AM
698
seems better (Knickmeyer, 2001; Swami et al., 2010). Bigger does
not seem better in Western cultures, where, according to 222
studies of 141,000 people, the rise in eating disorders over the
last 50 years has coincided with a dramatic increase in women
having a poor body image (Feingold & Mazzella, 1998).
Those most vulnerable to eating disorders are also those
(usually women or gay men) who most idealize thinness and
have the greatest body dissatisfaction (Feldman & Meyer, 2010;
Kane, 2010; Stice et al., 2010). Should it surprise us, then, that
when women view real and doctored images of unnaturally thin
models and celebrities, they often feel ashamed, depressed, and
dissatisfied with their own bodies—the very attitudes that predispose eating disorders (Grabe et al., 2008; Myers & Crowther, 2009;
Tiggemann & Miller, 2010)? Researchers tested this modeling idea by
giving some adolescent girls (but not others) a 15-month subscription
“Gee, I had no idea you were married to a supermodel.”
to an American teen-fashion magazine (Stice et al., 2001). Compared with
their counterparts who had not received the
magazine, vulnerable girls—defined as those
“Skeletons on Parade” A
who were already dissatisfied, idealizing thinnewspaper article used this headline
ness, and lacking social support—exhibited inin criticizing the display of superthin
models. Do such models make selfcreased body dissatisfaction and eating disorstarvation fashionable?
der tendencies. But even ultra-thin models do
not reflect the impossible standard of the classic
Barbie doll, who had, when adjusted to a height
of 5 feet 7 inches, a 32–16–29 figure (in centi“Why do women have such low
meters, 82–41–73) (Norton et al., 1996).
self-esteem? There are many
It seems clear that the sickness of today’s
complex psychological and
eating disorders lies in part within our weightsocietal reasons, by which I mean
Barbie.” -DAVE BARRY, 1999
obsessed culture—a culture that says, in countless ways, “Fat is bad,” that motivates millions
of women to be “always dieting,” and that encourages eating binges by pressuring women
to live in a constant state of semistarvation. If
cultural learning contributes to eating behavior,
then might prevention programs increase acceptance of one’s body? Reviews of prevention
studies answer Yes, and especially if the programs are interactive and focused on girls over
age 15 (Stice et al., 2007; Vocks et al., 2010).
© The New Yorker Collection, 1999, Michael Maslin from cartoonbank.com.
All Rights Reserved.
TEACH
Diversity Connections
The nation of Israel banned runway
and fashion models who fell within
a certain range below the recommended female body weight.
What effect might this decision
have had on Israeli women and
girls? How does observational
learning come into play here?
What is an ideal female body
weight? Does this differ by nation
or culture?
How much different is the average
female body weight from that of a
typical American fashion model or
beauty pageant contestant?
What are the implications of such a
difference?
WireImage/Getty Images
Unit XII Abnormal Behavior
Personality Disorders
ENGAGE
69-4
Critical Questions
Personality disorders are difficult to
diagnose and treat, because their
associated behaviors are enduring
and rigid. Have students discuss why
people may be unable to recognize
that someone has a personality
disorder:
Where is the line between being
eccentric, anxious, or odd and
having a personality disorder?
How might treatment be difficult?
personality disorders
psychological disorders
characterized by inflexible and
enduring behavior patterns that
impair social functioning.
What are the three clusters of personality disorders? What behaviors
and brain activity characterize the antisocial personality?
Some dysfunctional behavior patterns impair people’s social functioning without depression
or delusions. Among them are personality disorders—disruptive, inflexible, and enduring behavior patterns that impair one’s social functioning. Anxiety is a feature of one cluster
of these disorders, such as a fearful sensitivity to rejection that predisposes the withdrawn
avoidant personality disorder. A second cluster expresses eccentric or odd behaviors, such as
the emotionless disengagement of the schizoid personality disorder. A third cluster exhibits
dramatic or impulsive behaviors, such as the attention-getting histrionic personality disorder
and the self-focused and self-inflating narcissistic personality disorder.
MyersAP_SE_2e_Mod69_B.indd 698
Enrichment
Students may be curious about the other personality disorders identified in the DSM-5:
698
Unit XII
MyersPsyAP_TE_2e_U12.indd 698
1/21/14 9:35 AM
ENGAGE
Cluster A includes paranoid, schizoid, and
schizotypal personalities. People with
illnesses from this group of disorders
exhibit odd or eccentric behavior.
Cluster B includes antisocial, borderline,
histrionic, and narcissistic personalities.
This group focuses on disorders that
demonstrate dramatic or impulsive
behavior.
Cluster C includes avoidant, dependent, and
obsessive-compulsive personalities. These
are the anxiety-related disorders.
Abnormal Behavior
3/5/14 11:04 AM
MyersAP_SE_2e
EPA/JEFF TUTTLE/Landov
Other Disorders
Module 69
699
No remorse Dennis Rader, known
as the “BTK killer” in Kansas, was
convicted in 2005 of killing 10 people
over a 30-year span. Rader exhibited
the extreme lack of conscience that
marks antisocial personality disorder.
TEACH
Common Pitfalls
The terms psychopath and sociopath
are synonymous. These words are
actually legal terms and not used as
psychological diagnoses.
A P ® E x a m Ti p
Notice how different antisocial
personality disorder is from the
other disorders you have studied
in this unit. Because individuals
with antisocial personality
disorder so often behave badly,
they tend to be viewed differently
from people with disorders such
as depression or phobia.
TR M
TRM
The most troubling and heavily researched personality disorder is the
antisocial personality disorder. The person (sometimes called
a sociopath or a psychopath) is typically a male whose lack of conscience becomes plain before age 15, as he begins to lie, steal, fight,
or display unrestrained sexual behavior (Cale & Lilienfeld, 2002).
About half of such children become antisocial adults—unable to
keep a job, irresponsible as a spouse and parent, and assaultive or
otherwise criminal (Farrington, 1991). When the antisocial personality combines a keen intelligence with amorality, the result may be a
charming and clever con artist, a ruthless corporate executive (Snakes in
Suits is a book on antisocial behavior in business)—or worse.
“Thursday is out. I have jury duty.”
Despite their remorseless and sometimes criminal behavior, criminality is not an essenMany criminals, like this one,
tial component of antisocial behavior (Skeem & Cooke, 2010). Moreover, many criminals do exhibit a sense of conscience and
not fit the description of antisocial personality disorder. Why? Because they actually show responsibility in other areas of
their life, and thus do not exhibit
responsible concern for their friends and family members.
antisocial personality disorder.
Antisocial personalities behave impulsively, and then feel and fear little (Fowles &
Dindo, 2009). The results sometimes are horrifying, as they were in the case of Henry
Lee Lucas. He killed his first victim when he was 13. He felt little regret then or later. He
confessed that, during his 32 years of crime, he had brutally beaten, suffocated, stabbed,
shot, or mutilated some 360 women, men, and children. For the last 6 years of his reign of
terror, Lucas teamed with Elwood Toole, who reportedly slaughtered about 50 people he
“didn’t think was worth living anyhow” (Darrach & Norris, 1984).
Understanding Antisocial Personality Disorder
Antisocial personality disorder is woven of both biological and psychological strands. No
single gene codes for a complex behavior such as crime, but twin and adoption studies
reveal that biological relatives of those with antisocial and unemotional tendencies are at
increased risk for antisocial behavior (Larsson et al., 2007; Livesley & Jang, 2008). Molecular geneticists have identified some specific genes that are more common in those with
antisocial personality disorder (Gunter et al., 2010). The genetic vulnerability of people
1/21/14 9:35 AM
MyersAP_SE_2e_Mod69_B.indd 699
Superficial charm and high
intelligence
Poise, rationality, absence of
neurotic anxiety
Lack of a sense of personal
responsibility
Untruthfulness, insincerity,
callousness, manipulativeness
Antisocial behavior without regret
or shame
Poor judgment and failure to learn
from experience
Inability to establish lasting, close
relationships with others
Lack of insight into personal
motivations
Use Teacher Demonstration: Antisocial
Personality Disorder from the TRM to
help students learn more about this
disorder.
antisocial personality disorder
a personality disorder in which a
person (usually a man) exhibits a
lack of conscience for wrongdoing,
even toward friends and family
members. May be aggressive and
ruthless or a clever con artist.
Cleckley, H. (1976). The mask of sanity
(5th ed.). St. Louis, MO: Mosby.
1/21/14 9:36 AM
Other Disorders
MyersPsyAP_TE_2e_U12.indd 699
Teaching Tip
Harvey Cleckley identifies the following characteristics of antisocial
personality disorder:
© The New Yorker Collection, 2007, Leo Cullum from cartoonbank.com.
All Rights Reserved.
Antisocial Personality Disorder
TEACH
Module 69
699
3/5/14 11:04 AM
700
ENGAGE
Unit XII Abnormal Behavior
with antisocial and unemotional tendencies appears as a fearless approach to life. Awaiting aversive events, such as electric shocks or loud
Adrenaline
noises, they show little autonomic nervous system arousal (Hare, 1975;
excretion (ng/min.)
van Goozen et al., 2007). Long-term studies have shown that their
15
levels of stress hormones were lower than average when they were
youngsters, before committing any crime (FIGURE 69.1). Three-yearolds who are slow to develop conditioned fears are later more likely to
10
commit a crime (Gao et al., 2010).
Other studies have found that preschool boys who later became aggressive or antisocial adolescents tended to be impulsive, uninhibited,
5
unconcerned with social rewards, and low in anxiety (Caspi et al., 1996;
Tremblay et al., 1994). If channeled in more productive directions, such
fearlessness may lead to courageous heroism, adventurism, or star-level
0
athleticism (Poulton & Milne, 2002). Lacking a sense of social responsiStressful
Nonstressful
situation
situation
bility, the same disposition may produce a cool con artist or killer (LykNo criminal conviction
ken, 1995). The genes that put people at risk for antisocial behavior also
Criminal conviction
put people at risk for substance use disorders, which helps explain why
Figure 69.1
these disorders often appear in combination (Dick, 2007).
Cold-blooded arousability
Genetic influences, often in combination with child abuse, help wire the brain (Dodge,
and risk of crime Levels of
2009). Adrian Raine (1999, 2005) compared PET scans of 41 murderers’ brains with those
the stress hormone adrenaline
were measured in two groups of
from people of similar age and sex. Raine found reduced activity in the murderers’ frontal
13-year-old Swedish boys. In both
lobes, an area of the cortex that helps control impulses (FIGURE 69.2). This reduction was
stressful and nonstressful situations,
especially apparent in those who murdered impulsively. In a follow-up study, Raine and his
those who would later be convicted
team (2000) found that violent repeat offenders had 11 percent less frontal lobe tissue than
of a crime as 18- to 26-year-olds
showed relatively low arousal. (From
normal. This helps explain why people with antisocial personality disorder exhibit marked
Magnusson, 1990.)
deficits in frontal lobe cognitive functions, such as planning, organization, and inhibition
(Morgan & Lilienfeld, 2000). Compared with people who feel and display empathy, their
brains also respond less to facial displays of others’ distress (Deeley et al., 2006).
A biologically based fearlessness, as well as early environment, helps explain the reunion of long-separated sisters Joyce Lott, 27, and Mary Jones, 29—in a South Carolina
prison where both were sent on drug charges. After a newspaper story about their reunion,
their long-lost half-brother Frank Strickland called. He explained it would be a while before
he could come see them—because he, too, was in jail, on drug, burglary, and larceny charges
(Shepherd et al., 1990).
Genetics alone is hardly the whole story of antisocial crime, however. A study of crimiFYI
nal tendencies among young Danish men illustrates the usefulness of a complete bioDoes a full Moon trigger
psychosocial approach. Another Adrian Raine-led study (1996) checked criminal records on
“madness” in some people?
James Rotton and I. W. Kelly
nearly 400 men at ages 20 to 22, knowing that these men either had experienced biological
(1985) examined data from 37
risk factors at birth (such as premature birth) or came from family backgrounds marked by
Males with criminal convictions
as adults had lower levels of
arousal as 13-year-olds
Active Learning
While all serial killers are antisocial,
not all those with antisocial disorder
are serial killers. Have students explore
the lives and crimes of these serial killers, all of whom are believed to have
had antisocial personality disorder:
Ted Bundy
Kenneth Bianchi (one of the
Hillside Stranglers)
Jeffrey Dahmer
Aileen Wuornos, America’s only
known female serial killer
studies that related lunar phase
to crime, homicides, crisis calls,
and mental hospital admissions.
Their conclusion: There is
virtually no evidence of “Moon
madness.” Nor does lunar phase
correlate with suicides, assaults,
emergency room visits, or traffic
disasters (Martin et al., 1992;
Raison et al., 1999).
Figure 69.2
Murderous minds Researchers
have found reduced activation in a
murderer’s frontal lobes. This brain
area (shown in a left-facing brain)
helps brake impulsive, aggressive
behavior (Raine, 1999).
Frontal
lobe
MyersAP_SE_2e_Mod69_B.indd 700
700
Unit XII
MyersPsyAP_TE_2e_U12.indd 700
1/21/14 9:36 AM
Abnormal Behavior
3/5/14 11:04 AM
MyersAP_SE_2e
Other Disorders
Module 69
701
TEACH
Figure 69.3
Biopsychosocial roots of
crime Danish male babies whose
Percentage 35%
of criminal
30
offenders
Active Learning
backgrounds were marked both by
obstetrical complications and social
stresses associated with poverty
were twice as likely to be criminal
offenders by ages 20 to 22 as those
in either the biological or social risk
groups. (From Raine et al., 1996.)
25
20
15
10
5
Divide your students into small groups
and provide them with case studies
of people with different personality
disorders. After they read each case
study, ask groups to decide which
disorder is described. Then have students come together to see if they all
agree on the diagnosis. They may also
come up with their own case studies
to see if other students can correctly
diagnose the disorders.
0
Total crime
Childhood
poverty
Obstetrical
complications
Thievery
Violence
Both poverty and
obstetrical complications
poverty and family instability. The researchers then compared each of these two groups with
a third biosocial group whose lives were marked by both the biological and social risk factors. The biosocial group had double the risk of committing a crime (FIGURE 69.3). Similar
findings emerged from a famous study that followed 1037 children for a quarter-century:
Two combined factors—childhood maltreatment and a gene that altered neurotransmitter
balance—predicted antisocial problems (Caspi et al., 2002). Neither “bad” genes alone nor
a “bad” environment alone predisposed later antisocial behavior. Rather, genes predisposed
some children to be more sensitive to maltreatment. Within “genetically vulnerable segments of the population,” environmental influences matter—for better or for worse (Belsky
et al., 2007; Moffitt, 2005).
With antisocial behavior, as with so much else, nature and nurture interact and together
leave their marks on the brain. To explore the neural basis of antisocial behavior, neuroscientists are identifying brain activity differences in criminals who display antisocial personality
disorder. Shown emotionally evocative photographs, such as a man holding a knife to a
woman’s throat, they display lower heart rate and perspiration responses, and less activity in
brain areas that typically respond to emotional stimuli (Harenski et al., 2010; Kiehl & Buckholtz, 2010). They also display a hyper-reactive dopamine reward system that predisposes
their impulsive drive to do something rewarding, despite the consequences (Buckholtz et
al., 2010). Such data provide another reminder: Everything psychological is also biological.
CLOSE & ASSESS
Before You Move On
Exit Assessment
䉴 ASK YOURSELF
Given what we have learned in earlier units about the powers and limits of parental influence,
how much do you think parental training might affect the risk of a child’s developing
antisocial personality disorder?
Have students write the definition of
each of the disorders in this module.
Be sure they include the major diagnostic criteria for each.
䉴 TEST YOURSELF
What contribution do genes make to the development of antisocial personality disorder?
Answers to the Test Yourself questions can be found in Appendix E at the end of the book.
1/21/14 9:36 AM
MyersAP_SE_2e_Mod69_B.indd 701
1/21/14 9:36 AM
Other Disorders
MyersPsyAP_TE_2e_U12.indd 701
Module 69
701
3/5/14 11:04 AM
Unit XII Abnormal Behavior
702
Module 69 Review
69-1
•
Somatic symptom disorder presents a somatic (bodily)
symptom—some physiologically unexplained but
genuinely felt ailment.
•
With conversion disorder (also called functional
neurological symptom disorder), anxiety appears
converted to a physical symptom that has no reasonable
neurological basis.
•
The more common illness anxiety disorder is the
interpretation of normal sensations as a dreaded disorder.
69-2
•
•
Answers to Multiple-Choice
Questions
What are somatic symptom and related
disorders?
What are dissociative disorders, and why
are they controversial?
Dissociative disorders are conditions in which conscious
awareness seems to become separated from previous
memories, thoughts, and feelings.
Skeptics note that dissociative identity disorder, formerly
known as multiple personality disorder, increased
dramatically in the late twentieth century, that it is rarely
found outside North America, and that it may reflect
role-playing by people who are vulnerable to therapists’
suggestions. Others view this disorder as a manifestation
of feelings of anxiety, or as a response learned when
behaviors are reinforced by anxiety-reduction.
1. Adela regularly interprets ordinary physical symptoms
like stomach cramps and headaches as serious medical
problems. Her doctor is unable to convince her that her
problems are not serious. Adela suffers from
a.
b.
c.
d.
e.
illness anxiety disorder.
conversion disorder.
fugue state.
dissociative identity disorder.
anorexia nervosa.
MyersAP_SE_2e_Mod69_B.indd 702
Unit XII
MyersPsyAP_TE_2e_U12.indd 702
How do anorexia nervosa, bulimia nervosa,
and binge-eating disorder demonstrate
the influence of psychological and genetic
forces?
•
In these eating disorders, psychological factors may
overwhelm the homeostatic drive to maintain a balanced
internal state.
•
Despite being significantly underweight, people with
anorexia nervosa (usually adolescent females) continue to
diet because they view themselves as fat.
•
Those with bulimia nervosa (usually females in their teens
and twenties) secretly binge and then compensate by
purging, fasting, or excessively exercising.
•
Those with binge-eating disorder binge but do not follow
bingeing with purging, fasting, or exercise.
•
Cultural pressures, low self-esteem, and negative
emotions interact with stressful life experiences and
genetics to produce eating disorders.
69-4
What are the three clusters of personality
disorders? What behaviors and brain
activity characterize the antisocial
personality?
•
Personality disorders are disruptive, inflexible, and enduring
behavior patterns that impair social functioning.
•
These disorders form clusters, based on three main
characteristics: (1) anxiety; (2) eccentric or odd behaviors;
and (3) dramatic or impulsive behaviors.
•
Antisocial personality disorder is characterized by a lack of
conscience and, sometimes, by aggressive and fearless
behavior. Genetic predispositions may interact with
the environment to produce the altered brain activity
associated with antisocial personality disorder.
Multiple-Choice Questions
1. a
2. e
702
69-3
2. Which of the following is the diagnosis given to people
with multiple personalities?
a.
b.
c.
d.
e.
Schizophrenia
Antisocial personality disorder
Fugue state
Conversion disorder
Dissociative identity disorder
1/21/14 9:36 AM
Abnormal Behavior
3/5/14 11:04 AM
MyersAP_SE_2e
Other Disorders
Module 69
703
3. Which of the following is the defining characteristic of
3. b
antisocial personality disorder?
a.
b.
c.
d.
e.
Violence
Lack of conscience
Mood swings
Unexplained physical symptoms
Committing serial murders
Answer to Practice FRQ 2
Practice FRQs
1. Name and briefly describe three eating disorders.
Answer
1 point: Anorexia nervosa is a disorder in which the
individuals starve themselves despite being significantly
underweight.
1 point: Bulimia nervosa is a disorder in which the
individual alternates between bingeing and purging.
2. Dissociative identity disorder (DID) is among the most
1 point: Dissociative identity disorder
occurs when a person has multiple
personalities.
controversial of all psychological disorders. Briefly
describe the disorder. Then, provide one piece of
evidence that supports the existence of the disorder and
one piece of evidence that would indicate the disorder
might not be genuine.
1 point: Answers may vary, but one
possible explanation supporting the
disorder’s existence is that different body states (like handedness)
may be associated with the different
personalities.
(3 points)
1 point: Binge-eating disorder is a disorder in which the
individual binges without purging.
1 point: Answers may vary, but one
possible explanation indicating the
disorder may not be genuine is that
the huge increase in diagnosis of
the disorder is more likely to reflect
a cultural phenomenon than a real
disorder.
1/21/14 9:36 AM
MyersAP_SE_2e_Mod69_B.indd 703
1/21/14 9:36 AM
Other Disorders
MyersPsyAP_TE_2e_U12.indd 703
Module 69
703
3/5/14 11:04 AM
704
Unit XII Abnormal Behavior
Unit XII Review
Key Terms and Concepts to Remember
psychological disorder, p. 651
posttraumatic stress disorder (PTSD),
p. 664
somatic symptom disorder, p. 693
posttraumatic growth, p. 665
illness anxiety disorder, p. 694
medical model, p. 653
mood disorders, p. 671
dissociative disorders, p. 694
DSM-5, p. 654
major depressive disorder, p. 672
anxiety disorders, p. 661
mania, p. 673
dissociative identity disorder (DID),
p. 695
generalized anxiety disorder, p. 662
bipolar disorder, p. 673
anorexia nervosa, p. 697
panic disorder, p. 662
rumination, p. 679
bulimia nervosa, p. 697
phobia, p. 662
schizophrenia, p. 684
binge-eating disorder, p. 697
social anxiety disorder, p. 662
psychosis, p. 684
personality disorders, p. 698
agoraphobia, p. 663
delusions, p. 684
antisocial personality disorder, p. 699
obsessive-compulsive disorder (OCD),
p. 663
hallucination, p. 685
attention-deficit/hyperactivity disorder
(ADHD), p. 652
conversion disorder, p. 694
AP® Exam Practice Questions
Answers to Multiple-Choice
Questions
1. b
2. c
3. e
4. d
Multiple-Choice Questions
1. Which of the following statements is false?
a. Many behavioral and cognitive changes accompany
depression.
b. Someone suffering from depression will get better
only with therapy or medication.
c. Compared with men, women are nearly twice as
vulnerable to major depression.
d. Stressful events related to work, marriage, and close
relationships often precede depression.
e. With each new generation, depression is striking
earlier and affecting more people.
5. a
2. The risk of major depression and bipolar disorder
dramatically increases if you
a.
b.
c.
d.
e.
have suffered a debilitating injury.
have an adoptive parent that has the disorder.
have a parent or sibling with the disorder.
have a life-threatening illness.
have above-average intelligence.
MyersAP_SE_2e_Mod69_B.indd 704
704
Unit XII
MyersPsyAP_TE_2e_U12.indd 704
3. What do mental health professionals call a clinically
significant disturbance in an individual’s cognition,
emotion regulation, or behavior?
a.
b.
c.
d.
e.
An interaction of nature and nurture
A physiological state
A genetic predisposition
A psychological factor
A psychological disorder
4. Adolescent mood swings might be misdiagnosed as
which psychological disorder?
a.
b.
c.
d.
e.
Schizophrenia
Temper tantrums
Oppositional defiant disorder
Bipolar disorder
ADHD
5. A split from reality that shows itself in disorganized
thinking, disturbed perceptions, and/or diminished
or inappropriate emotions is associated with which
psychological disorder?
a. Schizophrenia
b. Phobias
c. Depression
d. Bipolar disorder
e. Anxiety
1/21/14 9:36 AM
Abnormal Behavior
3/5/14 11:04 AM
MyersAP_SE_2e
Review
6. The nearly 1-in-100 odds of any person being diagnosed
with schizophrenia become about 1 in 10 among those
a.
b.
c.
d.
who also suffer anxiety disorder.
whose sibling or parent has the disorder.
who have been diagnosed with depression.
who live with someone diagnosed with
schizophrenia.
e. whose identical twin has schizophrenia.
7. Which of the following can be characterized as a
compulsion?
a. Worry about exposure to germs or toxins
b. Fear that something terrible is about to happen
c. Concern with making sure things are in symmetrical
order
d. Anxiety when objects are not lined up in an exact
pattern
e. Checking repeatedly to see if doors are locked
8. Sensory experiences without sensory stimulation are
called
a.
b.
c.
d.
e.
word salads.
delusions.
paranoid thoughts.
ruminations.
hallucinations.
9. What is the most common reason people seek mental
health services?
a.
b.
c.
d.
e.
Depression
Bipolar disorder
Posttraumatic stress disorder
Dissociative identity disorder
Illness anxiety disorder
10. Brain-scanning techniques reveal what kinds of
brain activity differences in people with chronic
schizophrenia?
a. Abnormally high brain activity in the frontal lobes
b. An increase in the brain waves that reflect
synchronized neural firing
c. Abnormal activity in multiple brain areas
d. Decreased activity in the amygdala
e. A lack of dopamine receptors
Unit XII
705
11. Which personality disorder is associated with a lack of
regret over violating others’ rights?
a.
b.
c.
d.
e.
6.
7.
8.
9.
Antisocial personality disorder
Avoidant personality disorder
Schizoid personality disorder
Histrionic personality disorder
Narcissistic personality disorder
e
e
e
a
10.
11.
12.
13.
c
a
d
d
14. e
15. a
12. What term refers to thoughts about who or what we
blame for our successes and failures?
a.
b.
c.
d.
e.
Stability
Emotional memory
The social-cognitive perspective
Explanatory style
Dissociative reasoning
13. Although some psychological disorders are culturebound, others are universal. Which of the following
disorders is found in every known culture?
a.
b.
c.
d.
e.
Bulimia nervosa
Anorexia nervosa
Susto
Schizophrenia
Taijin-kyofusho
14. Modern psychologists contend that all behavior,
whether it is called normal or disordered, arises from the
interaction of
a.
b.
c.
d.
e.
genetics and physiology.
children and parents.
experience and wisdom.
inborn tendencies and drives.
nature and nurture.
Rubric for Free-Response
Question 2
1 point: Obsessions are repetitive,
unwanted thoughts or cognitions
that cause anxiety for individuals with
obsessive-compulsive disorder.
pp. 663–664
15. Which of the following are symptoms of generalized
anxiety disorder?
a. Unexplainable and continual tension
b. Sudden episodes of intense dread
c. Irrational and intense fear of a specific object or
situation
d. Repetitive thoughts or actions
e. Nightmares for weeks after a severe, uncontrollable
event
1 point: Compulsions are repetitive behaviors that individuals with
obsessive-compulsive disorder feel
compelled to perform because they
reduce the anxiety created by obsessive thoughts.
pp. 663–664
1 point: Common obsessive thoughts
experienced by individuals who
have obsessive-compulsive disorder
include concerns with dirt, germs, or
toxins; the fear that something terrible
might happen to them or others (fire,
death, illness); or concerns about
maintaining symmetry, order, or
exactness.
pp. 663–664
1/21/14 9:36 AM
MyersAP_SE_2e_Mod69_B.indd 705
1/21/14 9:36 AM
1 point: Common compulsive behaviors exhibited by individuals who
have obsessive-compulsive disorder
include excessive hand washing, bathing, toothbrushing, or grooming. In
addition, some compulsions involve
repeating rituals such as going in and
out of a door; getting up and down
from a chair; or repetitively checking
doors, locks, appliances, car brakes, or
homework assignments.
pp. 663–664
(Rubric continued on page 706.)
Review
MyersPsyAP_TE_2e_U12.indd 705
Unit XII
705
3/5/14 11:04 AM
706
1 point for either of the following:
According to the learning
approach, individuals may
develop compulsions as a result
of reinforcements. After initially
performing a particular behavior or
ritual, the individual finds that his
or her level of anxiety is reduced;
this then motivates the person to
repeat that behavior.
pp. 666–667
Individuals may develop
compulsions as a result of
observational learning. The
behaviors and rituals may develop
as a result of watching these
behaviors in others and adopting
them.
pp. 666–667
1 point for either of the following:
Research suggests that a particular
gene responsible for maintaining
levels of serotonin may be related
to anxiety disorders such as
obsessive-compulsive disorder.
pp. 667–668
Research suggests that genes
involved in the regulation of the
neurotransmitter glutamate may
be related to anxiety disorders.
Excess levels of glutamate might
result in overactivity in regions of
the brain related to alarm, which
results in elevated levels of anxiety.
pp. 667–668
Unit XII Abnormal Behavior
Free-Response Questions
1. After reading her AP® Psychology text, Jane starts
to wonder if all young people have some kind of
psychological disorder. First, briefly explain what you
might say to Jane about the criteria psychologists use
when diagnosing mental illnesses. Then, briefly explain
the symptoms associated with the following diagnoses
and how age might be related to each diagnosis:
• ADHD
2. Describe what the terms obsession and compulsion refer
to in the context of obsessive-compulsive disorder. Then,
briefly explain
• an example of a common obsession experienced by
individuals with obsessive-compulsive disorder.
• an example of a common compulsion experienced by
individuals with obsessive-compulsive disorder.
• how the learning perspective explains compulsions.
• Anorexia nervosa
• how the biological perspective explains compulsions.
• OCD
(6 points)
• Bipolar disorder
Rubric for Free-Response Question 1
1 point: Psychologists define a disorder as a significant
disturbance in thinking, emotion, and/or behavior that is
maladaptive. Jane’s general statement that all young people
have a psychological disorder doesn’t make sense in this
context: It’s not likely that all young people have significant
disturbances that interfere with their day-to-day lives.
page 651
1 point: ADHD: The symptoms of ADHD involve inattention, hyperactivity, and impulsivity. ADHD is diagnosed more
often in young people, but some adults are also diagnosed
page 652
with ADHD.
1 point: Anorexia nervosa: The symptoms of anorexia nervosa involve a starvation diet that results in a person weighing at least 15 percent less than he or she should. Often the
symptoms of anorexia nervosa appear in adolescence since
body image is likely to be a focus during this time of life,
but people of any age might suffer from this disorder.
page 697
3. Psychologists organize psychological disorders into
categories (in publications such as the DSM-5) in order
to communicate commonalities and differences among
psychological disorders, and to imply appropriate
treatment and encourage future research. For each
of the psychological disorders below, explain what
psychological disorder category it should be organized
into, and why it “belongs” in that category.
• Major depressive disorder
• Dissociative identity disorder
• Panic disorder
• Phobias
(4 points)
Multiple-choice self-tests and more may be found at
www.worthpublishers.com/MyersAP2e
1 point: OCD: The symptoms of OCD involve unwanted
repetitive thoughts (obsessions) and actions (compulsions).
This disorder is more commonly diagnosed in teens and
pages 663–664
young adults.
1 point: Bipolar disorder: The symptoms of bipolar disorder
involve alternating between mania and depression. Some
psychologists think that adolescent mood swings might have
caused the increase in this diagnosis among young people in
recent years.
pages 673–674
Rubric for Free-Response
Question 3
1 point: Major depressive disorder is
one of the mood disorders because
the most significant symptom of the
disorder is a disturbing disruption in
mood. Diagnostic criteria involve at
least 5 signs of depression (such as
depressed mood, insomnia, or suicidal
thinking) lasting 2 or more weeks.
pp. 671–672
1 point: Dissociative identity disorder
(DID) is listed in DSM-5 as one of the
dissociative disorders. All dissociative
disorders involve a sudden change in
identity, which describes the primary
symptom of DID: the manifestation
of more than one personality in an
individual.
pp. 695–696
706
Unit XII
MyersPsyAP_TE_2e_U12.indd 706
706
1MyersAP_SE_2e_Mod69_B.indd
point: Panic disorder
is one of the anxiety
disorders. The most disruptive symptom of
panic disorder is an episode of unpredictable
intense dread. Psychologists focus on the
anxiety elements of panic disorder as an emotional experience of anxiety (rather than the
perceived physiological symptoms associated
with the disorder).
pp. 661–662
1 point: Phobias involve an intense focus on
specific objects, activities, or situations, but
the one element that unites all phobias is the
feeling of fear. For this reason, phobias are
organized under the anxiety category, which
directs psychologists to focus on the anxiety
components of phobias rather than the specific “trigger” for the anxiety.
pp. 662–663
1/21/14 9:36 AM
Abnormal Behavior
3/5/14 11:04 AM